바른답 알찬 I 순열과조합 개의숫자중 1 이 3 개, 2 가 2 개있으므로구하는 7 자리자연수의개수는 7! 3!2! ~= 순열 교과서에서뽑은기본문제 pp. 8~ ⑴ 8 ⑵ 3 ⑶

Size: px
Start display at page:

Download "바른답 알찬 I 순열과조합 개의숫자중 1 이 3 개, 2 가 2 개있으므로구하는 7 자리자연수의개수는 7! 3!2! ~= 순열 교과서에서뽑은기본문제 pp. 8~ ⑴ 8 ⑵ 3 ⑶"

Transcription

1 바른답 알찬 확률과통계 345 제 해 (001~027) 일등확통 -01~03 강 ok.indd 오후 3:33

2 바른답 알찬 I 순열과조합 개의숫자중 1 이 3 개, 2 가 2 개있으므로구하는 7 자리자연수의개수는 7! 3!2! ~= 순열 교과서에서뽑은기본문제 pp. 8~ ⑴ 8 ⑵ 3 ⑶ 서로다른두개의주사위를동시에던질때, 나오는두눈의수를각각 a, b 라하면순서쌍 (a, b) 는 r1par a+b=2 인경우, (1, 1) 의 1 가지 r2par a+b=7 인경우, (1, 6), (2, 5), (3, 4), (4, 3), (5, 2), (6, 1) 의 6 가지 r1par, r2par 는동시에일어날수없으므로구하는경우의수는 1+6= = ⑴ _n&p_2=n(n-1) 이므로 n(n-1)=56=8 7.t3 n=8 ⑵ 120=6 5 4 이므로 120=_6&P_3.t3 r=3 ⑶ _8&P_r= 8! 8! ~= (8-r)! 3! ~ 이므로 8-r=3.t3 r= 명중에서 2 명을택하여일렬로나열하는방법의수와같으므로 _15&P_2=15 14= 명의남자가원형의탁자에둘러앉는방법의수는 (3-1)!=2!=2 남자들사이사이의 3 개의자리에여자 3 명을앉히는방법의 수는 _3&P_3=3!=6 따라서구하는방법의수는 2 6= 각자리에는 1, 2, 3, 4, 5 가모두올수있으므로구하는세자리자연수의개수는 5&PAI_3=5^3=125 기출문제 pp. 10~ ⑴ 100 ⑵ 40 ⑶ 나오는눈의수의합이 3 의배수가되는경우는눈의수의합이 3 또는 6 또는 9 또는 12 일때이다. r1par 눈의수의합이 3 인경우는 (1, 2), (2, 1) 의 2 가지 r2par 눈의수의합이 6 인경우는 (1, 5), (2, 4), (3, 3), (4, 2), (5, 1) 의 5 가지 r3par 눈의수의합이 9 인경우는 (3, 6), (4, 5), (5, 4), (6, 3) 의 4 가지 r4par 눈의수의합이 12 인경우는 (6, 6) 의 1 가지 r1par r4par 는동시에일어날수없으므로구하는경우의수는 = 부터 100 까지의자연수중에서 2 의배수는 50 개, 3 의배수는 33 개, 2 와 3 의최소공배수인 6 의배수는 16 개이므로 2 의배수또는 3 의배수의개수는 =67 따라서 2 또는 3 으로나누어떨어지지않는자연수의개수는 =33 2 의배수또는 3 의배수의개수구하기 60% 2 또는 3 으로나누어떨어지지않는자연수의개수 구하기 40% 2 바른답 알찬 해 (001~027) 일등확통 -01~03 강 ok.indd 오후 3:33

3 010 x, y 가양의정수이므로 3-<x+y-<5 를만족시키는경우는 x+y=3 또는 x+y=4 또는 x+y=5 일때이다. r1par x+y=3 일때, 순서쌍 (x, y) 는 (1, 2), (2, 1) 의 2 개 r2par x+y=4 일때, 순서쌍 (x, y) 는 (1, 3), (2, 2), (3, 1) 의 3 개 r3par x+y=5 일때, 순서쌍 (x, y) 는 (1, 4), (2, 3), (3, 2), (4, 1) 의 4 개 r1par ~ r3par 은동시에일어날수없으므로구하는순서쌍의개수는 2+3+4=9 011 x, y, z 가양의정수이므로 x->1, y->1, z->1 x+2y+3z=10 에서 3z-<7, 즉 z -< 7/3 이므로 z=1 또는 z=2 r1par z=1 일때, x+2y+3=10, 즉 x+2y=7 이므로순서쌍 (x, y) 는 (5, 1), (3, 2), (1, 3) 의 3 개 r2par z=2 일때, x+2y+6=10, 즉 x+2y=4 이므로순서쌍 (x, y) 는 (2, 1) 의 1 개 r1par, r2par 는동시에일어날수없으므로구하는순서쌍의개수는 3+1=4 양의정수 z 의경우구하기 30% 의경우에따라순서쌍 (x, y) 의개수구하기 50% 순서쌍 (x, y, z) 의개수구하기 20% x, y, z 중에서계수가가장큰 z 를기준으로경우를나누는것이 편리하다 반의 1 교시는국어, 2 교시는영어, 3 교시는수학, 4 교시는과학인경우에대하여 2 반의시간표를만들어보면다음과 같이 9 가지가있다. 1 교시 2 교시 3 교시 4 교시 국어과학수학 영어 ~ 수학과학국어 과학국어수학 국어과학영어수학국어영어과학영어국어 국어영어수학과학국어영어수학영어국어 같은방법으로 1 반의 1 교시는국어, 2 교시는영어, 3 교시는 과학, 4 교시는수학인경우에대하여 2 반의시간표를만드는 방법도 9 가지이다. 따라서구하는방법의수는 9+9=18 규칙성을찾기어려운경우의수를구할때에는수형도를그리면중복되지않고빠짐없이모든경우를나열할수있다. 013 좌석번호가 1 번인사람이 1 번자리에앉고나머지 4 명은다른번호의좌석에앉는경우를구해보면다음과같이 9 가지 가있다. 1 번 2 번 3 번 4 번 5 번 2 번 5 번 4 번 3 번 4 번 5 번 2 번 5 번 2 번 4 번 2 번 5 번 3 번 1 번 4 번 2 번 3 번 5 번 3 번 2 번 2 번 3 번 4 번 5 번 2 번 3 번 4 번 3 번 2 번 같은방법으로좌석번호가 2 번인사람이 2 번자리에앉는 경우, 좌석번호가 3 번인사람이 3 번자리에앉는경우, 좌석 번호가 4 번인사람이 4 번자리에앉는경우, 좌석번호가 5 번인사람이 5 번자리에앉는경우도각각 9 가지씩있다. 따라서구하는방법의수는 = (a+b+c)(x+y)^2=(a+b+c)(x^2&+2xy+y^2) 에서 a, b, c 에곱해지는항이각각 x&^2, 2xy, y^2 의 3 개이므로구하는항 의개수는 3 3=9 015 A C 로가는방법의수는 2 A B C 로가는방법의수는 3 2=6 따라서구하는방법의수는 2+6=8 동시에갈수없는길이면합의법칙, 이어지는길이면곱의법칙을이용한다. 016 서로다른 3 개의주사위를동시에던질때, 나오는눈의수의곱이홀수인경우는세주사위의눈의수가모두홀수인경 우뿐이므로이경우의수는 3 3 3=27 따라서서로다른 3 개의주사위를동시에던질때, 나오는눈 의수의곱이짝수인경우의수는 = 을소인수분해하면 360=2^3& 3^ 의양의약수의개수는 (3+1)(2+1)(1+1)=24.t3 a=24 01 순열 01. 순열 3 해 (001~027) 일등확통 -01~03 강 ok.indd 오후 3:33

4 바른답 알찬 360 의양의약수의총합은 (1+2+2^2&+2^3)(1+3+3^2&)(1+5)=1170.t3 b=1170.t3 a+b= = 을소인수분해하기 20% a 의값구하기 30% b 의값구하기 30% a+b 의값구하기 20% 자연수 N 이 N=a^p&b^q&c&^r(a, b, c 는서로다른소수, p, q, r 는자연 수 ) 꼴로소인수분해될때 ⑴ N 의양의약수의개수는 (p+1)(q+1)(r+1) ⑵ N 의양의약수의총합은 (1+a+a^2&+.c3+a^p)(1+b+b^2&+.c3+b^q)(1+c+c^2&+.c3+c^r) 원짜리동전 2 개로지불할수있는금액과 1000 원짜리지폐 1 장으로지불할수있는금액이같으므로 1000 원짜리 지폐 2 장을 500 원짜리동전 4 개로바꾸면지불할수있는금 액의수는 500 원짜리동전 7 개, 100 원짜리동전 4 개로지불 할수있는금액의수와같다. 500 원짜리동전으로지불할수있는금액은 0 원, 500 원, 1000 원,.c3, 3500 원의 8 가지 100 원짜리동전으로지불할수있는금액은 0 원, 100 원, 200 원, 300 원, 400 원의 5 가지 이때 0 원을지불하는경우는제외해야하므로구하는금액의 수는 8 5-1= 조건 에서 ~ f(-2)=-f(2), ~f(-1)=-f(1),~ f(0)=0 ~ 이므로 ~f(-2)=1 일때, f(2)=-1 로, ~f(-1)=2 일때, f(1)=-2 로정해진다. 따라서 ~f(-2) 와 ~f(-1) 이될수있는값은각각 -2, -1, 0, 1, 2 의 5 가지이므로구하는함수 ~f 의개수는 5 5= _n&p_2+4_n&p_1=28 에서 n(n-1)+4n=28 n^2&+3n-28=0, (n+7)(n-4)=0.t3 n=4 (.T3 n->2) 021 (n-1)!(n+1)! = 8/7 에서 (n!)^2 (n-1)! n! ` (n+1)! = 8/7 n! 1/n (n+1)=8/7, 7n+7=8n.t3 n=7 022 _n&p_4`:`2_n&p_2=3`:`1 에서 _n&p_4=6_n&p_2 n(n-1)(n-2)(n-3)=6n(n-1) _n&p_4 에서 n->4 이므로양변을 n(n-1) 로나누면 (n-2)(n-3)=6, n^2&-5n=0, n(n-5)=0.t3 n=5 (.T3 n->4).t3 5&P_2=5 4= _n&p_4+35_n-_1p_2-9_n&p_3=0 에서 n(n-1)(n-2)(n-3)+35(n-1)(n-2) -9n(n-1)(n-2)=0 _np_4 에서 n->4 이므로양변을 (n-1)(n-2) 로나누면 n(n-3)+35-9n=0 n^2&-12n+35=0, (n-5)(n-7)=0.t3 n=5 또는 n=7 따라서구하는모든자연수 n 의값의곱은 5 7=35 순열의수를 n 에대한식으로정리하기 40% n 의값구하기 40% 모든 n 의값의곱구하기 20% 024 지혜가 3 등을하는경우의수는지혜를제외한 4 명의학생을 1, 2, 4, 5 등에일렬로세우는경우의수와같으므로구하는 경우의수는 4!= = a, e 를한문자로보고 4 개의문자를일렬로나열하는방법의수는 4!=24 a 와 e 의자리를바꾸는방법의수는 2!=2 따라서구하는방법의수는 24 2= r1par 남자 3 명이앞줄에서옆으로나란히서로이웃하여서는방법의수는 3! 4!=144 r2par 남자 3 명이뒷줄에서옆으로나란히서로이웃하여서는 방법의수는 _4&P_3& (2 3!)=288 r1par, r2par 에서구하는방법의수는 =432 남자 3 명이앞줄에서는방법의수구하기 40% 남자 3 명이뒷줄에서는방법의수구하기 40% 남자 3 명이앞줄또는뒷줄에서는방법의수구하기 20% 027 남학생 3 명이앉을 3 개의의자와빈의자 1 개, 총 4 개의의자를일렬로나열하는방법의수는 4!=24 4 개의의자사이사이와양끝의 5 개의자리중 2 개를택하여 여학생이앉을의자를놓는방법의수는 5&P_2=20 따라서구하는방법의수는 24 20=480 다른 6 개의의자에 5 명이앉는방법의수는 _6P5 6 개의의자에여학생이이웃하여앉는방법의수는 5! 2! 따라서구하는방법의수는 _6P5&-5! 2!=480 4 바른답 알찬 해 (001~027) 일등확통 -01~03 강 ok.indd 오후 3:33

5 028 6개의문자를일렬로나열하는방법의수는 6!=720 모음은 O, E의 2개이므로양끝에모두모음이오도록나열하는방법의수는 2! 4!=48 따라서구하는방법의수는 =672 1등급비법 ( 사건 A가적어도한번일어나는경우의수 ) =( 모든경우의수 )-( 사건 A가일어나지않는경우의수 ) r3par C nemonemonemonemo 꼴인문자열의개수는 4!=24 r4par DA nemonemonemo 꼴인문자열의개수는 3!=6 r5par DB nemonemonemo 꼴인문자열의개수는 3!=6 이때 A로시작하는문자열부터 DB로시작하는문자열까지총개수는 =84이므로 DCABE, DCAEB, DCBAE, DCBEA, DCEAB,.c3 에서 89번째문자열은 DCEAB이다. 따라서 89번째문자열의마지막문자는 B이다. 032 서로다른한자리의자연수 6 개를일렬로나열하는방법의수는 6!= 순열 029 여섯개의숫자 0, 1, 2, 3, 4, 5 에서서로다른 3 개를택하여만든세자리자연수가짝수이려면일의자리의숫자가 0 또 는 2 또는 4 이어야한다. r1par 일의자리의숫자가 0 인경우 백의자리, 십의자리에는 0 을제외한 5 개의숫자중에서 2 개의숫자가올수있으므로그경우의수는 5&P_2=20 r2par 일의자리의숫자가 2 인경우 백의자리에는 0 과 2 를제외한 4 개의숫자가올수있고, 십의자리에는백의자리의숫자와 2 를제외한 4 개의숫 자가올수있으므로그경우의수는 4 4=16 r3par 일의자리의숫자가 4 인경우 백의자리에는 0 과 4 를제외한 4 개의숫자가올수있고, 십의자리에는백의자리의숫자와 4 를제외한 4 개의숫 자가올수있으므로그경우의수는 4 4=16 이상에서구하는짝수의개수는 =52 일의자리의숫자가 0인짝수의개수구하기 30% 일의자리의숫자가 2인짝수의개수구하기 30% 일의자리의숫자가 4인짝수의개수구하기 30% 세자리자연수중짝수의개수구하기 10% 보다큰수는 24 nemonemonemo, 25 nemonemonemo, 3 nemonemonemonemo, 4 nemonemonemonemo, 5 nemonemonemonemo 꼴이다. r1par 24 nemonemonemo 꼴인자연수의개수는 3!=6 r2par 25 nemonemonemo 꼴인자연수의개수는 3!=6 r3par 3 nemonemonemonemo 꼴인자연수의개수는 4!=24 r4par 4 nemonemonemonemo 꼴인자연수의개수는 4!=24 r5par 5 nemonemonemonemo 꼴인자연수의개수는 4!=24 이상에서구하는자연수의개수는 = r1par A nemonemonemonemo 꼴인문자열의개수는 4!=24 r2par B nemonemonemonemo 꼴인문자열의개수는 4!=24 서로다른한자리의자연수 6 개중에서짝수의개수를 n 이 라하면양끝에모두짝수가오도록나열하는방법의수는 _n&p_2& 4!=_nP_2 24 이때적어도한쪽끝에홀수가오도록나열하는방법의수가 432 이므로 720-_n&P_2& 24=432, _n&p_2& 24=288 즉, n(n-1)=4 3 이므로 n=4.t3 _n&p_2&=12 따라서짝수의개수가 4 이므로홀수의개수는 6-4=2 짝수의개수를 n 이라하고, 양끝에모두짝수가오도 록나열하는방법의수구하기 30% 주어진조건을이용하여 n 의값구하기 50% 홀수의개수구하기 20% 033 운전석에는아버지또는어머니만앉을수있으므로운전석에앉는방법의수는 _2P_1=2 할아버지와할머니는가운데줄에만앉을수있으므로그방 법의수는 _3P_2=6 나머지 4 명의가족이빈자리에앉는방법의수는 4!=24 따라서구하는방법의수는 = 부모가마주보도록원형의탁자에앉은다음나머지네자리에 4 명이앉으면되므로구하는방법의수는 4!=24 다른 아버지의자리가결정되면어머니의자리는마주 보는자리에고정되므로구하는방법의수는아버지와나머 지 4 명의가족, 즉 5 명이원형의탁자에둘러앉는방법의수 와같다..t3 (5-1)!=4!= A, B 를한묶음으로생각하여 5 개의용기를원형의실험기구에넣는경우의수는 (5-1)!=4!=24 A 와 B 의자리를바꾸는경우의수는 2!=2 따라서구하는방법의수는 24 2= 순열 5 해 (001~027) 일등확통 -01~03 강 ok.indd 오후 3:33

6 바른답 알찬 036 남학생 5명이원형의탁자에둘러앉는방법의수는 (5-1)!=4!=24 남학생사이사이의 5개의자리에여학생 3명이앉는방법의수는 5&P_3=60 따라서구하는방법의수는 24 60= 가운데사각형을칠하는방법의수는 9 이고, 나머지 8 개의사각형을칠하는방법의수는가운데사각형에칠한색을제 외한 8 가지색을원형으로배열하는방법의수와같으므로 (8-1)!=7! 이때원형으로배열하는한가지방법에대하여사각형모양 에서는다음그림과같이 2 가지의서로다른경우가존재한 다. 남학생 5명이앉는방법의수구하기 40% 여학생 3명이앉는방법의수구하기 40% 여학생끼리이웃하지않게앉는방법의수구하기 20% 명이원형의탁자에둘러앉는방법의수는 (6-1)!=5!=120 이때원형의탁자에둘러앉는한가지방법에대하여직사각 형모양의탁자에서는다음그림과같이 3 가지의서로다른 경우가존재한다 따라서구하는방법의수는 120 3= 다각형모양의탁자에둘러앉는방법의수를구할때에는원형으로배열하는방법의수와다각형으로배열할때서로다른경우의수를구하여곱한다 명이원형의탁자에둘러앉는방법의수는 (10-1)!=9! 이때원형의탁자에둘러앉는한가지방법에대하여정오각 형모양의탁자에서는다음그림과같이 2 가지의서로다른 경우가존재한다 따라서구하는방법의수는 2 9! 가운데삼각형을칠하는방법의수는 4 이고, 나머지 3 개의삼각형을칠하는방법의수는가운데삼각형에칠한색을제 외한 3 가지색을원형으로배열하는방법의수와같으므로 (3-1)!=2!=2 따라서구하는방법의수는 4 2= 따라서구하는방법의수는 9 7! 2=18 7! 041 서로다른 3 개의동아리에서 5 개를택하는중복순열의수와같으므로 _3&PAI5=3^5= ⑴ 백의자리에는 0 이올수없으므로백의자리에올수있는숫자는 1, 2, 3, 4 의 4 개이다. 십의자리, 일의자리에올수있는숫자의개수는 0, 1, 2, 3, 4 의 5 개의숫자에서 2 개를택하는중복순열의수와 같으므로 5&PAI_2=5^2=25 따라서구하는세자리자연수의개수는 4 25=100 ⑵ 백의자리에는 0이올수없으므로백의자리에올수있는숫자는 1, 2, 3, 4의 4개이다. 십의자리에올수있는숫자는 0, 1, 2, 3, 4 의 5 개이다. 일의자리에올수있는숫자는 1, 3 의 2 개이다. 따라서구하는홀수의개수는 4 5 2=40 ⑶ 세자리자연수중짝수의개수는 =60 세자리자연수의개수구하기 40% 세자리자연수중홀수의개수구하기 40% 세자리자연수중짝수의개수구하기 20% 043 천의자리에올수있는숫자는 0 을제외한 1, 2, 3, 4, 5, 6 의 6 개이고, 백의자리, 십의자리, 일의자리중에서 0 이오 는자리를정하는경우의수는 3 이다. 남은두자리에올수있는숫자의개수는 1, 2, 3, 4, 5, 6 의 6 개에서 2 개를택하는중복순열의수와같으므로 _6&PAI_2=6^2=36 따라서구하는자연수의개수는 =648 6 바른답 알찬 해 (001~027) 일등확통 -01~03 강 ok.indd 오후 3:33

7 보다작은자연수의개수는 999 이고, 세자리이하의자연수에서 4 와 5 가들어가지않는수의개 수는 0, 1, 2, 3, 6, 7, 8, 9 의 8 개에서 3 개를택하는중복순 열의수에서 000 의 1 개를제외한수의개수와같으므로 _8&PAI_3&-1=8^3&-1=511 따라서구하는수의개수는 = f(1)+f(2)=3인경우는 f(1)=1, f(2)=2 또는 ~f(1)=2, f(2)=1의 2가지 f(3), f(4), f(5) 의값을정하는방법의수는 1, 2, 3, 4, 5 의 5개에서 3개를택하는중복순열의수와같으므로 5&PAI_3=5^3=125 따라서구하는함수 ~f의개수는 다른 m과 w를제외한 6개의문자 t, o, o, r, r, o를일렬로나열하는방법의수는 6! 3!2! =60 6개의문자사이사이와양끝의 7개의자리중 2개를택하여 m과 w를나열하는방법의수는 _7&P_2=42 따라서구하는방법의수는 60 42= a, d와 c, e의순서가각각정해져있으므로 a, d를모두 A 로, c, e를모두 B로생각하여 7개의문자 A, b, B, A, B, f, g를일렬로나열한후, 첫번째 A는 a, 두번째 A는 d로, 첫번째 B는 c, 두번째 B는 e로바꾸면된다. 따라서구하는방법의수는 7! 2!2! = 순열 2 125=250 f(1), f(2) 의값을정하는방법의수구하기 30% f(3), f(4), f(5) 의값을정하는방법의수구하기 50% 함수 ~f의개수구하기 20% 046 f(3)=3이므로 A : ~ f(1), f(2), f(4) 의값을정하는방법의수는 1, 2, 3, 4의 4개에서 3개를택하는중복순열의수와같으므로 _4&PAI_3=4^3=64 B : ~f(1), f(2), f(4) 의값을정하는방법의수는 1, 2, 4의 3개의숫자를일렬로나열하는방법의수와같으므로 3!=6 C : ~f(1), f(2), f(4) 의값도 3이어야하므로이방법의수는 1이다..t3 A-2B+C= = b와 d를제외한 5개의문자 a, a, a, c, c를일렬로나열하는방법의수는 5! 3!2! =10 양끝에 b와 d를나열하는방법의수는 2!=2 따라서구하는방법의수는 10 2= r1par 8개의문자 t, o, m, o, r, r, o, w를일렬로나열하는방법의수는 8! 3!2! =3360 r2par m과 w를한문자로생각하여 7개의문자를일렬로나열하는방법의수는 7! 3!2! =420 이때 m과 w의자리를바꾸는방법의수는 2!=2 따라서 m과 w가이웃하도록나열하는방법의수는 420 2= , 4와 1, 3, 5의순서가각각정해져있으므로 2, 4를모두 A로, 1, 3, 5를모두 B로생각하여 6장의카드 B, A, B, A, B, 6을일렬로나열한후, 첫번째 A는 2, 두번째 A는 4로, 첫번째 B는 1, 두번째 B는 3, 세번째 B는 5로바꾸면된다. 따라서구하는방법의수는 6! 2!3! = 각자리숫자의합이 9인경우는 (0, 0, 9), (0, 1, 8), (0, 2, 7), (0, 3, 6), (0, 4, 5), (1, 1, 7), (1, 2, 6), (1, 3, 5), (1, 4, 4), (2, 2, 5), (2, 3, 4), (3, 3, 3) r1par (0, 0, 9) 로만들수있는세자리자연수는 900의 1개 r2par (0, 1, 8), (0, 2, 7), (0, 3, 6), (0, 4, 5) 로만들수있는세자리자연수의개수는 4 (2 2!)=16 r3par (1, 1, 7), (1, 4, 4), (2, 2, 5) 로만들수있는세자리자연수의개수는 3 3! 2! =9 r4par (1, 2, 6), (1, 3, 5), (2, 3, 4) 로만들수있는세자리자연수의개수는 3 3!=18 r5par (3, 3, 3) 으로만들수있는세자리자연수는 333의 1개 이상에서구하는자연수의개수는 =45 각자리의숫자의합이 9인경우구하기 30% 각경우의세자리자연수의개수구하기 50% r1par, r2par 에서구하는방법의수는 =2520 각자리의숫자의합이 9인세자리자연수의개수구하기 20% 01. 순열 7 해 (001~027) 일등확통 -01~03 강 ok.indd 오후 3:33

8 바른답 알찬 052 A 에서 P 까지최단거리로가는방법의수는 5! 4! =5 P에서 B까지최단거리로가는방법의수는 6! 5! =6 따라서 A 에서출발하여 P 를거쳐 B 까지최단거리로가는 방법의수는 5 6=30 a, b는주사위의눈의수이므로 a-b의값은정수이다. r1par a-b=-1일때, 순서쌍 (a, b) 는 (1, 2), (2, 3), (3, 4), (4, 5), (5, 6) 의 5개 r2par a-b=0일때, 순서쌍 (a, b) 는 (1, 1), (2, 2), (3, 3), (4, 4), (5, 5), (6, 6) 의 6개 r3par a-b=1일때, 순서쌍 (a, b) 는 (2, 1), (3, 2), (4, 3), (5, 4), (6, 5) 의 5개 이상에서구하는순서쌍 (a, b) 의개수는 5+6+5=16 이차방정식의판별식을이용하여 a-b 의값의범위 구하기 30% a-b 의값에따라순서쌍 (a, b) 의개수구하기 50% 순서쌍 (a, b) 의개수구하기 20% 1 등급문제 합의법칙 전략수형도를그려서방법의수를구한다. pp. 21~23 A 학생이 C 학생의과제를확인하는경우를구해보면다음과 같이 8 가지가있다. A B C D E C D E A E B B E A E A B B A A B D B A D D 같은방법으로 A 학생이 D 학생과 E 학생의과제를확인하는 경우도각각 8 가지씩있다. 따라서구하는방법의수는 8+8+8= 합의법칙전략이차함수 y=f(x) 의그래프가 x축과만나지않으려면방정식 f(x)=0의판별식 D가 D<0이어야함을이용한다. 이차함수 y=x^2&-(a+b)x+ab+1 의그래프와 x 축이만나 지않으려면이차방정식 x^2&-(a+b)x+ab+1=0 의판별 A 식을 D 라할때, D<0 이어야하므로 D=(a+b)^2&-4(ab+1)<0 (a-b)^2&-4<0 (a-b+2)(a-b-2)<0.t3-2<a-b<2 B B A 055 곱의법칙 전략지불할수있는방법의수는곱의법칙을이용하여구한다음 0 원을지불하는경우를제외하고, 지불할수있는금액의수는 500 원짜 리동전과 100 원짜리동전을 50 원짜리동전으로바꾸어생각한다. r1par 지불할수있는방법의수 500 원짜리동전 1 개로지불할수있는방법은 0 개, 1 개의 2 가지 100 원짜리동전 5 개로지불할수있는방법은 0 개, 1 개,.c3, 5 개의 6 가지 50 원짜리동전 10 개로지불할수있는방법은 0 개, 1 개,.c3, 10 개의 11 가지 이때 0 원을지불하는경우를제외해야하므로구하는방 법의수는 =131 r2par 지불할수있는금액의수 500 원짜리동전 1 개로지불할수있는금액과 100 원짜리 동전 5 개로지불할수있는금액이같고, 100 원짜리동전 1 개로지불할수있는금액과 50 원짜리동전 2 개로지불 할수있는금액이같다. 따라서 500 원짜리동전 1 개를 50 원짜리 10 개, 100 원짜리 동전 5 개를 50 원짜리동전 10 개로바꾸면지불할수있는 금액의수는 50 원짜리동전 30 개로지불할수있는금액 의수와같으므로 0 원, 50 원, 100 원, 150 원,.c3, 1450 원, 1500 원의 31 가지 이때 0 원을지불하는경우는제외해야하므로구하는금 액의수는 31-1=30 r1par, r2par 에서 m=131, n=30 이므로 m-n=131-30= _n&p_r 의계산 전략 _np_r= n! (0-<r-<n) 을이용한다. (n-r)! 8 바른답 알찬 해 (001~027) 일등확통 -01~03 강 ok.indd 오후 3:33

9 _n&-_1p_r&+r _n-_1&p_r-_1 &= (n-1)! (n-1)! +r (n-r-1)! (n-r)! = (n-1)! (n-r) (n-1)! +r (n-r-1)! (n-r) (n-r)! = (n-1)! (n-r)! {(n-r)+r} = (n-1)! (n-r)! ~ n = n! (n-r)! =_n&p_r.t3 (n-r-1)!, (n-r)!, n, n! 060 원순열전략남학생 4명을먼저자리에앉히고, 여학생을앉히는방법의수를구한다. 남학생 4명이정사각형모양의탁자의각변에 1명씩앉을때, 각각오른쪽또는왼쪽의자를선택하여앉을수있으므로그방법의수는 (4-1)! =96 남은네개의의자에여학생 4명이앉는방법의수는 4!=24 따라서구하는방법의수는 96 24= 순열 057 순열의수 전략 254보다큰짝수는백의자리의숫자가 3 이상이고일의자리의 숫자가짝수임을이용한다. 254보다큰짝수는 3 nemo 0, 3 nemo 2, 3 nemo 4, 4 nemo 0, 4 nemo 2, 5 nemo 0, 5 nemo 2, 5 nemo 4 꼴이다. r1par 3 nemo 0, 3 nemo 2, 3 nemo 4 꼴인자연수의개수는 3 _4&P_1=12 r2par 4 nemo 0, 4 nemo 2 꼴인자연수의개수는 2 _4&P_1=8 r3par 5 nemo 0, 5 nemo 2, 5 nemo 4 꼴인자연수의개수는 3 _4&P_1=12 이상에서구하는자연수의개수는 = 순열의수 전략 아버지와어머니가 A열에앉을경우와 B열에앉을경우로나 누어구한다. r1par 아버지와어머니가 A열에이웃하여앉는방법의수는 2 2! 3!=24 r2par 아버지와어머니가 B열에이웃하여앉는방법의수는 2! 3!=12 r1par, r2par 에서구하는방법의수는 24+12=36 아버지와어머니가 A열에앉는방법의수구하기 40% 아버지와어머니가 B열에앉는방법의수구하기 40% 아버지와어머니가같은열에이웃하여앉는방법의 20% 수구하기 059 원순열 전략 주어진규칙에따라남학생과여학생의자리를정하고, 정해진 자리에 5명의남학생과 4명의여학생이앉는방법의수를구한다. 오른쪽그림에서선생님 (A) 을 기준으로교실을나가는순서는 A B C D E A이다. 따라서구하는경우의수는 A, B, C, C D D, E의자리에선생님과여학생이앉고남은다섯자리에남학생이앉는방법의수와같으므로 (5-1)! 5!=2880 E B 정사각형모양의탁자의한변에는의자가 2개씩이므로남학생 4 명이자리에앉을때, 4명모두의자를고르는것까지고려해야한다. 061 중복순열전략세자리자연수의개수에서 3이들어가지않는자연수의개수를뺀다. 0, 1, 2, 3의 4개의숫자에서중복을허용하여만들수있는세자리자연수의개수는 3 _4&PAI_2=48 3을제외한 0, 1, 2의 3개의숫자에서중복을허용하여만들수있는세자리자연수의개수는 2 _3&PAI_2=18 따라서구하는자연수의개수는 48-18=30 세자리자연수의개수구하기 40% 3이들어가지않는자연수의개수구하기 40% 적어도한번은 3이들어가는자연수의개수구하기 20% 062 같은것이있는순열전략가로로한칸가는것을 a, 세로로한칸가는것을 b라하고, A 에서 B까지최단거리로갈수있는지점을찾는다. 오른쪽그림과같이네지점 C B C, D, E, F를잡으면 D F r1par A C B로가는방법의 P 수는 1 E r2par A D B로가는방법의 A 수는 5! 4! 4! 3! =20 r3par A E F B로가는방법의수는 (P는거치지않는다.) ^( 4! 3! ~-1^) 1 ^( 2!2! 2! ~-1^)=10 이상에서구하는방법의수는 01. 순열 9 해 (001~027) 일등확통 -01~03 강 ok.indd 오후 3:33

10 바른답 알찬 =31 1등급비법 A에서 B로갈때, 장애물이있는경우에는반드시거쳐야하는점을잡아최단거리로가는방법의수를구한다. 02 조합 교과서에서뽑은기본문제 pp. 24~ 순열의수 1단계조건, 를만족시키는경우의수를구한다. 조건 에서야구공은연속하여꺼낼수없으므로테니스공 4개를꺼내는사이사이나앞뒤에야구공을꺼내야한다. 065 ⑴ 9 ⑵ 8 ⑶ ⑴ 84 ⑵ ⑴ 3 ⑵ ⑴ 2 ⑵ ⑴ _n&c_2=upn(n-1) 2 `이므로 upn(n-1) 2 `=36에서 n(n-1)=72=9 8 위의그림에서조건 를만족시키는야구공 2 개의위치는 (1, 3), (2, 3), (2, 4), (3, 4), (3, 5) 의 5 가지 이다. 2 단계순열의수를이용하여서로다른테니스공 4 개와서로다른야 구공 2 개를일렬로나열하는방법의수를구한다. 서로다른테니스공 4 개를일렬로나열하는방법의수는 4!=24 위의그림에서정해진 2 개의위치에서로다른야구공 2 개를 일렬로나열하는방법의수는 2!=2 3 단계곱의법칙을이용하여 6 개의공을꺼내는방법의수를구한다. 조건, 를모두만족시키면서 6 개의공을상자에서모두 꺼내는방법의수는 = 같은것이있는순열 1 단계작은정육면체에서가로로 1 칸가는것을 a, 세로로 1 칸가는 것을 b, 위로 1 칸가는것을 c 로생각하여꼭짓점 A 에서꼭짓점 B 까지 최단거리로가는방법의수를구한다. 꼭짓점 A 에서꼭짓점 B 로가려면가로, 세로, 높이의방향 으로각각 2 번씩이동해야하므로그방법의수는 6! 2!2!2! =90 2 단계꼭짓점 A 에서점 P 를거쳐꼭짓점 B 까지최단거리로가는방 법의수를구한다 꼭짓점 A 에서점 P 로가려면가로, 세로, 높이의방향으로 각각 1 번씩이동해야하므로그방법의수는 3! 1!1!1! =6 같은방법으로점 P 에서꼭짓점 B 까지최단거리로가는방 법의수도 6 이다. 따라서꼭짓점 A 에서점 P 를거쳐꼭짓점 B 까지최단거리 로가는방법의수는 6 6=36 3 단계꼭짓점 A 에서점 P 를거치지않고꼭짓점 B 까지최단거리로 가는방법의수를구한다. 꼭짓점 A 에서점 P 를거치지않고꼭짓점 B 까지최단거리 로가는방법의수는 90-36=54.t3 n=9 ⑵ _n&c_3=_n&c_n-_3 이므로 _n&c_3=_n&c5 에서 n-3=5.t3 n=8 ⑶ r1par _10&C_r=_10&C_r-_4 에서 r=r-4 이식을만족시키는 r 의값은존재하지않는다. r2par _10&C_r=_10&C_1_0_-_r 이므로 _10&C_1_0_-_r=_10&C_r-_4 에서 10-r=r-4.t3 r=7 r1par, r2par 에서 r=7 066 ⑴ 구하는방법의수는 9 명중에서 3 명을택하는방법의수 와같으므로 _9&C_3= =84 ⑵ 남자 5명중에서 2명을뽑는방법의수는 5&C_2= =10 여자 4명중에서 1명을뽑는방법의수는 _4&C_1=4 따라서구하는방법의수는 10 4= 구하는방법의수는서로다른 4 개중에서중복을허용하여 10 개를택하는중복조합의수와같으므로 _4&H_10=_4+_10-_1&C_10=_1_3&C_10=_1_3&C_3= = ⑴ 두집합의원소가각각 1 개, 2 개인경우의수는 _3&C_1 _2&C_2=3 1=3 이므로 S(3, 2)=3 ⑵ 세집합의원소가각각 1 개, 1 개, 2 개인경우의수는 _4&C_1 _3&C_1 _2&C_2 low2! ~~ 1 = /2=6 이므로 S(4, 3)=6 069 ⑴ 5=3+1+1=2+2+1 이므로 P(5, 3)=2 ⑵ 6= = 이므로 P(6, 4)=2 070 ^(x^2&+ 2 ^)^^5의전개식의일반항은 x 5&C_r(x^2)^5-^r^( 2 ^)^^r=5&c_r 2^r x^10-^3^r x 10 바른답 알찬 해 (001~027) 일등확통 -01~03 강 ok.indd 오후 3:33

11 x^4 항은 10-3r=4 일때이므로 r=2 따라서 x^4 의계수는 5&C_2 2^2=10 4= _n&c0&+_n&c_1&+_n&c_2&+.c3+_n&c_n=2^n 에서 _n&c_1&+_n&c_2&+.c3+_n&c_n=2^n&-1 이때 2^n&-1=127 이므로 2^n=128=2^7.t3 n=7 기출문제 ⑴ 210 ⑵ 15 ⑶ _1_2&C_2_r+_1&=_1_2&C_7-_r& 에서 2r+1=7-r 또는 (2r+1)+(7-r)=12 r1par 2r+1=7-r 일때, 3r=6.t3 r=2 r2par (2r+1)+(7-r)=12 일때, r+8=12.t3 r=4& r1par, r2par 에서구하는모든자연수 r 의값의곱은 2 4=8 073 조건 에서 r-1=3r+1 또는 (r-1)+(3r+1)=8 r1par r-1=3r+1 일때, 2r=-2.t3 r=-1 pp. 26~36 그런데조건을만족시키는자연수 r 의값은존재하지않 는다. r2par (r-1)+(3r+1)=8 일때, 4r=8.t3 r=2 r1par, r2par 에서 r=2 조건 에서 _n&c_2&+_n&c_3&=2 _2_n&C_1 _n&c_2&+_n&c_3=_n+_1&c_3 이므로 _n+_1&c_3&=2 _2_n&C_1& (n+1) n (n-1) =2 2n n^3&-n=24n, n^3&-25n=0 n(n-5)(n+5)=0.t3 n=0 또는 n=5 또는 n=-5 이때 n 은자연수이므로 n=5.t3 nr=2 5= 이차방정식 3x^2&-3_n&C_r&x-5_n&P_r=0의두근이 -2, 5이므로이차방정식의근과계수의관계에의하여 -2+5= 3_n&C_r 5_n&P_r, (-2) 5=- 3 3.t3 _n&c_r=3, _n&p_r&=6 이때 _n&p_r=r!_n&c_r& 이므로 6=r! 3 r!=2.t3 r=2 _n&p_2&=6에서 n(n-1)=6=3 2이므로 n=3.t3 n+r=3+2=5& 075 _n&c_1=n, _n&c_2= n(n-1), _n&c_3= n(n-1)(n-2) 가이순서대로등차수열을이루므로등차중항의성질에의 하여 2 n(n-1) =n+ n(n-1)(n-2) n^2&-n=n+1/6&(n^3&-3n^2&+2n) n^3&-9n^2&+14n=0 n(n-2)(n-7)=0.t3 n=0 또는 n=2 또는 n=7 n>3 이므로 n=7 등차중항의성질을이용하여 n 에대한관계식구하기 50% n 의값구하기 50% 076 ⑴ 전체 10 명중에서 4 명을뽑는방법의수는 _10&C_4=up `~ `~&=210 ⑵ 남자 6 명중에서 4 명을뽑는방법의수는 _6&C_4=_6&C_2=up &`=15 ⑶ =195 4 명을뽑는방법의수구하기 35% 남자 4 명을뽑는방법의수구하기 35% 여자가적어도한명포함되도록하는방법의수구하기 30% 077 철수를포함하여 4 명을뽑는경우의수는철수를제외한 9 명중에서 3 명을뽑는경우의수와같으므로 a=_9&c_3 철수를포함하지않고 4 명을뽑는경우의수는철수를제외 한 9 명중에서 4 명을뽑는경우의수와같으므로 b=_9&c_4 02 조합 02. 조합 11 해 (001~027) 일등확통 -01~03 강 ok.indd 오후 3:33

12 바른답 알찬.t3 a+b=_9&c_3&+_9c_4=_10c_4 1 서로다른 n 개에서특정한 k 개를포함하여 r 개를뽑는방법의 수는 (n-k) 개에서 (r-k) 개를뽑는방법의수와같다. _n_-_k&c_r_-_k 2 서로다른 n 개에서특정한 k 개를제외하고 r 개를뽑는방법의 수는 (n-k) 개에서 r 개를뽑는방법의수와같다. _n_-_k&c_r 권의교과서중에서 2 권을뽑는방법의수는 _5&C_2=10 3 권의문제집중에서 2 권을뽑는방법의수는 _3&C_2=_3&C_1=3 4 권의책을일렬로꽂는방법의수는 4!=24 따라서구하는방법의수는 =720 단순히뽑는것은조합이고, 뽑은다음일렬로나열하는것은순열이므로뽑아서나열하는경우의수는조합의수와순열의수를각각구한후이들을곱하여구한다. 079 nemo nemo nemo nemo nemo nemo에 2부터 7까지 6개의자연수를주어진조건에맞게나열한다고할때, 3, 5가나열되는두자리를선택하는경우의수는 _6C_2=15 이때선택한두자리의왼쪽에 3, 남은자리에 5를나열하면된다. 남은네자리에 2, 4, 6이나열되는세자리를선택하는경우의수는 _4C_3=_4C_1=4 이때선택한세자리의왼쪽부터작은수를차례로나열하고남은한자리에 7을나열하면된다. 따라서구하는경우의수는 = 가로방향의 4개의평행선에서 2개, 세로방향의 6개의평행선에서 2개를택하면한개의평행사변형을만들수있으므로구하는평행사변형의개수는 _4&C_2 _6&C_2=6 15=90 7개의점중에서 3개를택하는방법의수구하기 35% 한직선위에있는 4개의점중에서 3개를택하는방 35% 법의수구하기 삼각형의개수구하기 30% 1등급비법한직선위에있는서로다른 n개의점으로는삼각형을만들수없으므로이런경우는반드시제외해야한다 개의점중에서 3개를택하는방법의수는 _7&C_3=35 이때한직선위에있는 3개의점으로는삼각형을만들수없으므로구하는삼각형의개수는 35-3= r1par 원에내접하는직사각형의두대각선의교점은원의중심이고, 오른쪽그림과같이원위에같은간격으로놓인 12개의점을원의중심이지나도록연결한선분은 6개이므로 12개의점중에서 4개를연결하여만들수있는직사각형의개수는원의중심을지나는 6개의선분중 2개를택하는방법의수와같으므로 m=_6&c_2=15 r2par 원에내접하는직각삼각형의빗변의중점은원의중심이므로 12개의점중에서 3개를연결하여만들수있는직각삼각형의개수는원의중심을지나는 6개의선분중 1개를택하고남은 10개의점중 1개를택하는방법의수와같으므로 n=_6&c_1 _10&C_1=6 10=60 r1par, r2par 에서 m+n=75 1등급비법원의지름에대한원주각의크기는 90 이므로 1 6개의지름중 2개를택하면그지름을대각선으로갖는직사각형을만들수있다. 2 6개의지름중 1개를택하면그지름을빗변으로갖는직각삼각형을만들수있다 개의점중에서 3개를택하는방법의수는 _7&C_3=35 한직선위에있는 4개의점중에서 3개를택하는방법의수는 _4&C_3=4 이때한직선위에있는 3개의점으로는삼각형을만들수없으므로구하는삼각형의개수는 35-4= 집합 A의모든원소 a에대하여 ~f(a)-<a를만족시키는 f(a) 의값이될수있는것은다음과같다. r1par a=1일때, ~f(1)-<1에서 f(1) 의값은 1의 1가지 r2par a=2일때, ~f(2)-<2에서 f(2) 의값은 1, 2의 2개에서 1개를택하면되므로 _2&C_1=2( 가지 ) 12 바른답 알찬 해 (001~027)-01~03 강 ok.indd 오후 2:14

13 r3par a=3 일때,~ f(3)-<3 에서 f(3) 의값은 1, 2, 3 의 3 개에서 1 개를택하면되므로 _3&C_1=3( 가지 ) r4par a=4 일때,~ f(4)-<4 에서 f(4) 의값은 1, 2, 3, 4 의 4 개에서 1 개를택하면되므로 _4&C_1=4( 가지 ) 이상에서구하는함수 ~ f 의개수는 = X={2, 3, 5, 7}, Y={1, 2, 3, 4, 5, 6, 7, 8} 이므로조건, 에서 ~f(2) 의값이될수있는수는 1 또는 2 f(5), f(7) 의값이될수있는수는 4 또는 5 또는 6 또는 7 또는 8 이때 ~f(5)<f(7) 이어야하므로 4, 5, 6, 7, 8 의 5 개에서 2 개 를택하여그값이작은것부터차례로 ~f(5), f(7) 에대응시 키면된다. 따라서구하는함수 ~ f 의개수는 _2&C_1 5&C_2=2 10= 조건 에서 f(3) 의값이될수있는수는 6 또는 7 r1par ~f(3)=6 일때, 조건 에서 ~ f(1), ~f(2) 의값이될수있는수는 3 또는 4 또는 5 이때 ~f(1)<f(2) 이어야하므로 3, 4, 5 의 3 개에서 2 개를 택하여그값이작은것부터차례로 ~f(1), f(2) 에대응시 키면된다. 또, ~f(4), f(5) 의값이될수있는수는 7 또는 8 또는 9 마찬가지로 ~f(4)<f(5) 이어야하므로 7, 8, 9 의 3 개에서 2 개를택하여작은것부터차례로 ~ f(4), f(5) 에대응시키 면된다. 087 r1par 조건 에서함수 ~f 의개수는집합 Y 의원소 1, 2, 3, 4, 5, 6, 7 의 7 개에서 5 개를택하여그값이작은것부터차 례로 ~ f(-2), f(-1), f(0), f(1),~ f(2) 에대응시키는 방법의수와같으므로 m=_7&c5=_7&c_2=21 r2par 조건 에서 ~ f(-2)=f(2),~ f(-1)=f(1) 함수 ~ f 의개수는집합 Y 의원소 1, 2, 3, 4, 5, 6, 7 의 7 개 에서 3 개를택하는중복순열의수와같으므로 n=_7&pai_3=7^3&=343 r1par, r2par 에서 m+n= 서로다른 3 개에서 7 개를택하는중복조합의수와같으므로 _3&H_7=_9&C_7=_9&C_2= 서로다른 4 개에서 5 개를택하는중복조합의수와같으므로 _4&H5=_8&C_5=_8&C_3= 서로다른 3 개에서 m 개를택하는중복조합의수가 36 이므로 _3&H_m=_m+_2&C_m=_m+_2&C_2=up(m+2)(m+1) `~2 `~=36 (m+2)(m+1)=72=9 8.t3 m=7 따라서고구마피자, 새우피자, 불고기피자를적어도하나씩 포함하여 7 개를주문하는경우의수는 3 종류의피자를 1 개씩 주문하면 4 개의피자를더주문해야하므로서로다른 3 개에 서 4 개를택하는중복조합의수와같다..t3 _3&H_4=_6&C_4=_6&C_2=15 n 명에게같은물건 r 개를나누어줄때, 한명에게적어도한개 를나누어주는방법의수는 _n&h_r-_n 이다. ( 단, n-<r) 02 조합 따라서함수 ~ f의개수는 ~ _3&C_2 _3&C_2=3 3=9 r2par f(3)=7일때, 조건 에서 ~ f(1), f(2) 의값이될수있는수는 3 또는 4 또는 5 또는 6 이때 ~f(1)<f(2) 이어야하므로 3, 4, 5, 6의 4개에서 2 개를택하여그값이작은것부터차례로 ~f(1), f(2) 에대응시키면된다. 또, ~f(4), ~f(5) 의값이될수있는수는 8 또는 9이므로 f(4)=8, ~f(5)=9 따라서함수 ~f의개수는 _4&C_2 1=6 r1par, r2par 에서구하는함수 ~f의개수는 9+6=15 f(3)=6일때, 함수 ~f의개수구하기 40% f(3)=7일때, 함수 ~f의개수구하기 40% 함수 ~f의개수구하기 20% 091 주어진조건에서 ~f(1)-<~f(2)-<~f(3) 따라서구하는함수 ~f 의개수는집합 Y 의원소 1, 2, 3, 4, 5, 6 의 6 개에서 3 개를택하는중복조합의수와같다..t3 _6&H_3=_8&C_3=56 두집합 X, Y 에대하여 n(x)=m, n(y)=n 이라하면함수 f`:`x?c Y 중에서 (x_1, x_2&\<x) 1 x_1<x_2 이면 ~ f(x_1)<f(x_2) 를만족시키는함수 f 의개수 _n&c_m 2 x_1<x_2 이면 ~ f(x_1)-<f(x_2) 를만족시키는함수 f 의개수 _n&h_m 092 방정식 x+y+z=9 를만족시키는음이아닌정수 x, y, z 의순서쌍 (x, y, z) 의개수는서로다른 3 개의문자에서 9 개를 택하는중복조합의수와같으므로 a=_3&h_9=_1_1&c_2=55 x-1=a, y-1=b, z-1=c 로놓고 x+y+z=9 에대입 하면 02. 조합 13 해 (001~027) 일등확통 -01~03 강 ok.indd 오후 3:33

14 바른답 알찬 (A+1)+(B+1)+(C+1)=9.t3 A+B+C=6 즉, 방정식 x+y+z=9 를만족시키는양의정수 x, y, z 의 순서쌍 (x, y, z) 의개수는방정식 A+B+C=6 을만족시 키는음이아닌정수 A, B, C 의순서쌍 (A, B, C) 의개수 와같다. 따라서서로다른 3 개의문자에서 6 개를택하는중복조합의 수와같으므로 b=_3&h_6=_8&c_6=_8&c_2=28.t3 a-b=55-28= ^a&2^b&2^c&=1024 에서 2^a^+^b^+^c&=2^10 이므로 a+b+c=10 이때 a-1=a, b-1=b, c-1=c 로놓고 a+b+c=10 에대입하면 (A+1)+(B+1)+(C+1)=10.t3 A+B+C=7 즉, 구하는순서쌍의개수는방정식 A+B+C=7 을만족시 키는음이아닌정수 A, B, C 의순서쌍 (A, B, C) 의개수 와같다. 따라서서로다른 3 개의문자에서 7 개를택하는중복조합의 수와같으므로 _3&H_7=_9&C_7=_9&C_2= x=2x+1, y=2y+1, z=2z+1 로놓고 x+y+z=19 에대입하면 (2X+1)+(2Y+1)+(2Z+1)=19 2X+2Y+2Z=16.t3 X+Y+Z=8 즉, 구하는순서쌍의개수는방정식 X+Y+Z=8 을만족 시키는음이아닌정수 X, Y, Z 의순서쌍 (X, Y, Z) 의개 수와같다. 따라서서로다른 3 개의문자에서 8 개를택하는중복조합의 수와같으므로 _3&H_8=_10&C_8=_10&C_2= x, y, z, w 가양의정수이므로 x->1, y->1, z->1, w->1 따라서 x+y+z+w->4 이므로부등식 4-<x+y+z+w-<6 을만족시키는양의정수 x, y, z, w 의순서쌍 (x, y, z, w) 의개수를구하면된다. x-1=x, y-1=y, z-1=z, w-1=w 로놓고 4-<x+y+z+w-<6 에대입하면 4-<(X+1)+(Y+1)+(Z+1)+(W+1)-<6.t3 0-<X+Y+Z+W-<2 r1par X+Y+Z+W=0 을만족시키는음이아닌정수 X, Y, Z, W 의순서쌍 (X, Y, Z, W) 의개수는 _4&H0=_3&C0=1 r2par X+Y+Z+W=1 을만족시키는음이아닌정수 X, Y, Z, W 의순서쌍 (X, Y, Z, W) 의개수는 _4&H_1=_4&C_1=4 r3par X+Y+Z+W=2 를만족시키는음이아닌정수 X, Y, Z, W 의순서쌍 (X, Y, Z, W) 의개수는 _4&H_2=5&C_2=10 이상에서구하는순서쌍의개수는 =15 x+y+z+w=4일때, 순서쌍의개수구하기 30% x+y+z+w=5일때, 순서쌍의개수구하기 30% x+y+z+w=6일때, 순서쌍의개수구하기 30% 순서쌍의개수구하기 10% 를소인수분해하면 105=3\5\7이므로구하는방법의수는집합 {3, 5, 7} 을원소가각각 1개, 2개인두집합으로분할하는방법의수와같다..t3 _3&C_1 _2&C_2=3 1=3 097 r1par 두집합의원소가각각 1개, 4개인경우의수는 5&C_1 _4&C_4=5 1=5 r2par 두집합의원소가각각 2개, 3개인경우의수는 5&C_2 _3&C_3=10 1=10 r1par, r2par 에서구하는방법의수는 5+10=15 두집합의원소가각각 1개, 4개인경우의수구하기 40% 두집합의원소가각각 2개, 3개인경우의수구하기 40% 주어진집합을 2개의집합으로분할하는방법의수구하기 20% 098 서로다른 7 권의책을 3 권, 2 권, 2 권으로나누는방법의수는 _7&C_3 _4&C_2 _2&C_2 low2! `1 = /2=105 1등급비법 서로다른 n개를 p개, q개, r개 (p+q+r=n) 로분할하는방법의수는 1 p, q, r가모두다른수일때 : _n&c_p _n-_p&c_q _r&c_r 2 p, q, r 중에서어느두수가같을때 : _n&c_p _n-_p&c_q _r&c_r low2! `1 3 p, q, r가모두같은수일때 : _n&c_p _n-_p&c_q _r&c_r low3! `1 099 여자 9 명중 1 명이남자 3 명과한조를이루면되므로여자 9 명을 4 명, 4 명, 1 명으로나누면된다. 따라서구하는방법의수는 _9&C_4 5&C_4 _1&C_1 low2! `1 = /2= =1+3=2+2이므로 4명을두조로나누는방법은다음과같다. r1par 1명, 3명으로나누는방법의수는 _4&C_1 _3&C_3=4 1=4 r2par 2명, 2명으로나누는방법의수는 _4&C_2 _2&C_2 low2! `1``=6 1 1/2=3 14 바른답 알찬 해 (001~027) 일등확통 -01~03 강 ok.indd 오후 3:33

15 r1par, r2par 에서 4 명을두조로나누는방법의수는 4+3=7 이때두조를 2 대의보트에배정하는방법의수는 2!=2 이 므로구하는방법의수는 7 2= r1par 서로다른 6 개의구슬을 3 개, 2 개, 1 개로나누는방법의수는 p=_6&c_3 _3&C_2 _1&C_1=60 r2par 서로다른 6 개의구슬을 3 개, 2 개, 1 개로나누어 3 명에게 나누어주는방법의수는 q=_6&c_3 _3&C_2 _1&C_1 3!=60 6=360 r1par, r2par 에서 p+q=420 p 의값구하기 40% q 의값구하기 40% p+q 의값구하기 20% 명을 3 명, 3 명의두조로나누는방법의수는 _6&C_3 _3&C_3 low2! `1``=20 1 1/2=10 각조에서부전승으로올라가는 1 명을택하는방법의수는 _3&C_1 _3&C_1=3 3=9 따라서구하는방법의수는 10 9= 개의팀을 4개, 3개의팀으로나누는방법의수는 _7&C_4 _3&C_3=35 1=35 4개의팀을 2개, 2개의팀으로나누는방법의수는 _4&C_2 _2&C_2 low2! `1``=6 1 1/2=3 3개의팀을 2개, 1개의팀으로나누는방법의수는 _3&C_2 _1&C_1=3 따라서구하는방법의수는 = = = = = = = = 따라서구하는방법의수는 P(9, 6)+P(9, 7)+P(9, 8)+P(9, 9)= = = c3+1 1이 11개 = c 이 9개 = c 이 8개 = c3+1+4= c 이 7개 1이 7개 k k k k k = c3+1+5= c 이 6개 1이 6개 =1+1+.c3+1+6=1+1+.c 이 5개 1이 5개 =1+1+.c =1+1+.c 이 5개 1이 5개 = 가 5개따라서구하는분할의개수는 12이다. 106 구하는방법의수는 9 를 4 개의자연수로분할하는방법의수와같다. 9= = = = = = t3 P(9, 4)=6 107 구하는방법의수는 7 을 3 개이하의자연수로분할하는방법의수와같다. 7=6+1=5+2=4+3 =5+1+1=4+2+1=3+3+1=3+2+2.t3 P(7, 1)+P(7, 2)+P(7, 3)=1+3+4=8 참고 k k k k 빈접시가 0 개, 1 개, 2 개인방법의수는차례로 P(7, 3), P(7, 2), P(7, 1) 이다. k 똑같은공 n 개를똑같은상자 k 개에나누어담을때, 1 빈상자가없도록나누어담는방법의수 P(n, k) 2 빈상자가있어도되는방법의수 P(n, 1)+P(n, 2)+.c3+P(n, k) 108 (x-3y)^5의전개식의일반항은 5&C_r& x&^5-^r(-3y)^r=5&c_r(-3)^r& x^5-^r&y^r x^4&y항은 r=1일때이므로 x^4&y의계수는 5&C_1 (-3)= ^(&ax-1/x&^)^^4 의전개식의일반항은 k k _4&C_r(ax)^4-^r^(-1/x&^)^^r=_4&C_r&a^4-^r(-1)^r&x^4-^2^r 상수항은 4-2r=0일때이므로 r=2 이때상수항이 54이므로 _4&C_2&a^2(-1)^2=54, a^2=9.t3 a=3 (.T3 a>0) 1 ~ 항은 4-2r=-2일때이므로 r=3 x^2 따라서 1 ~ 의계수는 _4&C_3 3 (-1)^3=-12 x^2.c3.c3 전개식의일반항구하기 30% a의값구하기 30% 1 의계수구하기 40% x^2 02 조합 02. 조합 15 해 (001~027) 일등확통 -01~03 강 ok.indd 오후 3:33

16 바른답 알찬 110 (x+a)^5 의전개식의일반항은 5C_r& x^5-^r&a^r x^3 항은 r=2 일때이므로 x^3 의계수는 5C_2&a^2=10a^2 x^4 항은 r=1 일때이므로 x^4 의계수는 5C_1&a=5a x^3 의계수와 x^4 의계수가같으므로 10a^2=5a.t3 60a=60 1/2=30.t3 a=1/2 (.T3 a>0) 111 ^(x+ 1 &^)^^4의전개식의일반항은 x _4&C_r&x^4-^r^( 1 x &^)^^r=_4&c_r&x^4-^2^r.c3.c3 ᄀ 이때 (2x^2&+3x+4)^(x+ 1 &^)^^4의전개식에서상수항은 x 2x^2과ᄀ의 1 x^2 ~ 항, 3x와ᄀ의 1 ~ 항, 4와ᄀ의상수항 x 이곱해질때나타난다. r1par ᄀ의 1 ~ 항은 4-2r=-2일때이므로 r=3 x^2.t3 _4&C_3 &x&-^2= 4 x^2 ~ r2par ᄀ의 & 1/x 항은 4-2r=-1 일때이므로 r=5/2 그런데 r 는 0-<r-<4 인정수이므로 1/x 항은존재하지않는다. r3par ᄀ의상수항은 4-2r=0일때이므로 r=2.t3 _4&C_2=6 이상에서구하는상수항은 2x^2 4 x^2 +4 6= (1+x)+(1+x)^2&+.c3+(1+x)^1^4.c3.c3 ᄀᄀ은첫째항이 1+x, 공비가 1+x, 항의개수가 14 인등비 수열의합이므로 up(1+x){(1+x)^1^4-1} `(1+x)-1 ` =up(1+x)^1^5-(1+x) x.c3.c3 ᄂ ᄀ의전개식에서 x^6 의계수는ᄂ의 (1+x)^1^5 의전개식에서 x^7 의계수와같다. (1+x)^1^5 의전개식의일반항은 _15&C_r& x^r x^7 항은 r=7 일때이므로 x^7 의계수는 _15&C_7=_15&C_8 따라서 n+r 의최댓값은 15+8=23 등비수열의합을이용하여주어진식간단히하기 30% (1+x)^1^5 의전개식에서 x^7 의계수구하기 40% 114 _n&c_1&+_n&c_3&+_n&c5&+.c3+_n&c_n=2^n-^1 이므로 _2_1C_1&+_2_1&C_3&+_2_1&C5&+.c3+_2_1&C_2_1=2^20.t3 log_2~(_2_1&c_1&+_2_1&c_3&+_2_1&c5&+.c3+_2_1&c_2_1) =log_2 2^20= _n&c0&+_n&c_1&+_n&c_2&+.c3+_n&c_n=2^n 이므로 _n&c_1&+_n&c_2&+.c3+_n&c_n=2^n&-1 500<_n&C_1&+_n&C_2&+.c3+_n&C_n<1000 에서 500<2^n&-1<1000.t3 501<2^n<1001 이때 2^8=256, 2^9=512, 2^10=1024 이므로 n=9 _n&c_1&+_n&c_2&+.c3+_n&c_n 을간단히하기 40% 2^n 의값의범위구하기 30% n 의값구하기 30% 116 _2C0&+_3C_1&+_4C_2&+.c3+_10C_8 =(_3C0&+_3C_1)+_4C_2&+.c3+_10C_8 (.T3 _2C0=_3C0=1) =(_4&C_1&+_4&C_2)+.c3+_1_0&C_8 =5C_2&+&.c3+_10C_8=&.c3 =_10C_7&+_10C_8=_1_1C_8=_1_1C_3=165 파스칼의삼각형에서 1 이항계수의배열은좌우대칭이므로 _n&c_r=_n&c_n-_r 2 각단계의수는그윗단계의이웃하는두수의합과같으므로 _n-_1&c_r-_1&+_n-_1&c_r=_n&c_r 117 r1par _n&c_n=1이므로 _1C_1&+_2C_2&+_3C_3&+&.c3+_10C_10=10 r2par _1C0&+_2C_1&+_3C_2&+.c3+_10C_9 =(_2C0&+_2C_1)+_3C_2&+.c3+_10C_9 (.T3 _1C0=_2C0=1) =(_3C_1&+_3C_2)+.c3+_10C_9 =_4C_2&+.c3+_10C_9=.c3 =_10C_8&+_10C_9=_1_1C_9=_1_1C_2=55 r1par, r2par 에서색칠한부분의모든수의합은 10+55=65 1 등급문제 pp. 37~39 n+r 의최댓값구하기 30% 113 _n&c0&+_n&c_1&+_n&c_2&+.c3+_n&c_n=2^n 이므로 5&C0&+5&C_1&+5&C_2&+5&C_3&+5&C_4&+5&C5=2^5= 조합의수전략모든경우의수에서남학생만뽑는경우의수와여학생만뽑는경우의수를뺀다. 16 바른답 알찬 해 (001~027) 일등확통 -01~03 강 ok.indd 오후 3:33

17 전체 13 명중에서 3 명을뽑는방법의수는 _1_3C_3=286 남학생만 3 명을뽑는방법의수는 _9C_3=84 여학생만 3 명을뽑는방법의수는 _4C_3=4 따라서구하는방법의수는 286-(84+4)= 조합의수 전략 다섯자리자연수이면서 7 끼리는이웃하지않으려면 7 은최대 3 개까지포함할수있다. 다섯자리자연수를만들때, 7 을 2 개이상포함하고 7 끼리는이웃하지않도록하려면 7 은 2 개또는 3 개이어야한다. r1par 7 이 2 개인경우 nemo nemo nemo 꼴에서 nemo 의자리에는 1, 2, 3, 5, 9 의 5 개의숫자에서 3 개를뽑아일렬로나열하고, 나머지네개 의 의자리에서 2 개를택하여 7 을배열하면되므로 5P_3 _4C_2=60 6=360 r2par 7 이 3 개인경우 7 nemo 7 nemo 7 꼴에서 nemo 의자리에 1, 2, 3, 5, 9 의 5 개의숫자 에서 2 개를뽑아일렬로나열하면되므로 5P_2=20 r1par, r2par 에서구하는다섯자리자연수의개수는 =380 7 이 2 개인경우의자연수의개수구하기 40% 7 이 3 개인경우의자연수의개수구하기 40% 다섯자리자연수의개수구하기 20% 120 조합의수 ; 도형의개수 전략 주어진조건을만족시키는점 (x, y) 를좌표평면위에나타내 고, 사각형을만들수없는경우를제외한다. 조건, 를모두만족시키 는점 (x, y) 를좌표평면위에나타 내면오른쪽그림과같다. 13 개의점중에서 4 개를택하는방법 의수는 _1_3C_4=715 r1par 한직선위에있는 3 개의점과다 른한점을택할때, 한직선위에 3 개의점이있는직선은 10 개, 한직선위에 5 개의점이있는직선은 2 개이므로 _3&C_3 _10&C_1 10+5&C_3 _8&C_1 2=260 r2par 한직선위에있는 5 개의점중에서 4 개를택할때, 한직선위에 5 개의점이있는직선은 2 개이므로 5&C_4 2=10 그런데한직선위에있는 3 개의점과다른한점또는한직 선위에있는 4 개의점으로는사각형을만들수없으므로구 하는사각형의개수는 715-(260+10)= 중복조합 전략중복조합의수를이용한다 y O 1 2 x 규칙, 에의하여 6종류의원판중에서중복을허용하여 3개의원판을택하면쌓는방법은한가지로정해진다. 따라서구하는방법의수는서로다른 6개에서 3개를택하는중복조합의수와같으므로 _6H_3=_8C_3= 중복조합 ; 정수해의개수전략 z=1, z=2, z=3일때로나누어주어진방정식을만족시키는양의정수 x, y, z의순서쌍 (x, y, z) 의개수를구한다. r1par z=1일때, z^2=1이므로주어진방정식은 x+y+1=12.t3 x+y=11 이때양의정수 x, y의순서쌍 (x, y) 의개수는서로다른 2개의문자에서 9개를택하는중복조합의수와같으므로 _2H_9=_10C_9=_10C_1=10 r2par z=2일때, z^2=4이므로주어진방정식은 x+y+4=12.t3 x+y=8 이때양의정수 x, y의순서쌍 (x, y) 의개수는서로다른 2개의문자에서 6개를택하는중복조합의수와같으므로 _2H_6=_7C_6=_7C_1=7 r3par z=3일때, z^2=9이므로주어진방정식은 x+y+9=12.t3 x+y=3 이때양의정수 x, y의순서쌍 (x, y) 의개수는서로다른 2개의문자에서 1개를택하는중복조합의수와같으므로 _2H_1=_2C_1=2 이상에서구하는순서쌍의개수는 =19 z=1일때, 순서쌍 (x, y) 의개수구하기 30% z=2일때, 순서쌍 (x, y) 의개수구하기 30% z=3일때, 순서쌍 (x, y) 의개수구하기 30% 순서쌍 (x, y, z) 의개수구하기 10% 123 집합의분할전략각조의인원이 2명이상이어야하므로 (2명, 6명 ), (3명, 5명 ), (4명, 4명 ) 으로나누어구한다. r1par 2명, 6명으로나눌때, 선생님 1명, 학생 1명을 2명인조에배정하는방법의수는 (_4&C_1 _4&C_1) _6&C_6=16 r2par 3명, 5명으로나눌때, 선생님 1명, 학생 2명을 3명인조에배정하는방법의수는 (_4&C_1 _4&C_2) 5&C5=24 선생님 2명, 학생 1명을 3명인조에배정하는방법의수는 (_4&C_2 _4&C_1) 5&C5=24 이므로 24+24=48 r3par 4명, 4명으로나눌때, 선생님 1명, 학생 3명을한조에배정하는방법의수는 02 조합 02. 조합 17 해 (001~027) 일등확통 -01~03 강 ok.indd 오후 3:33

18 바른답 알찬 (_4&C_1 _4&C_3) _4&C_4 low2! `1 =8 선생님 2 명, 학생 2 명을한조에배정하는방법의수는 (_4&C_2 _4&C_2) _4&C_4 low2! `1 =18 선생님 3 명, 학생 1 명을한조에배정하는방법의수는 (_4&C_3 _4&C_1) _4&C_4 low2! `1 =8 이므로 =34 이상에서구하는방법의수는 = 자연수의분할 전략 을생각한다. 구슬을각상자에 1 개씩넣은후, 나머지구슬을나누어담는방법 구슬을각상자에 1 개씩넣은후, 남은 7 개의구슬을 3 개의상자에빈상자가없도록나누어담으면된다. 이때 7=5+1+1=4+2+1=3+3+1=3+2+2 이므로 구하는방법의수는 P(7, 3)=4 다른 각상자에구슬을 2 개씩넣은후, 남은 4 개의구슬 을 3 개이하의상자에넣으면된다. 이때 4=3+1=2+2=2+1+1 이므로구하는방법의수는 P(4, 1)+P(4, 2)+P(4, 3)=1+2+1=4 125 이항정리 전략 21^1^1=(1+20)^1^1 이므로이항정리를이용한다. 21^1^1 =(1+20)^1^1 =_1_1&C0&+_1_1&C_1& 20+_1_1&C_2& 20^2&+.c3+_1_1&C_1_1& 20^1^1 = _1_1&C0&+_1_1&C_1& ^2(_1_1&C_2&+_1_1&C_3& 20+.c3+_1_1&C_1_1& 20^9) 이때 20^2(_1_1&C_2&+_1_1&C_3& 20+.c3+_1_1&C_1_1& 20^9) 은 400 으로나 누어떨어지므로 21^1^1 을 400 으로나누었을때의나머지는 _1_1&C0&+_1_1&C_1& 20 을 400 으로나누었을때의나머지와같다. _1_1&C0&+_1_1&C_1& 20=1+220=221 따라서구하는나머지는 221 이다. 126 이항정리 이항정리를이용하여 21^1^1=(1+20)^1^1 을전개하기 40% 의전개식에서 400 으로나누어떨어지는항을찾 고, 21^1^1 을 400 으로나누었을때의나머지구하기 전략 (1+i)^1^6 을전개한후, 복소수의성질을이용한다. (1+i)^1^6= _1_6&C0&+_1_6&C_1&i+_1_6&C_2&i ^2&+_1_6&C_3&i ^3&+_1_6&C_4&i ^4 +.c3+_1_6&c_1_6&i ^1^6 에서 i ^4^n=1, i ^4^n^+^1=i, i ^4^n^+^2=-1, i ^4^n^+^3=-i 이므로 (1+i)^1^6= (_1_6&C0&-_1_6&C_2&+_1_6&C_4&-_1_6&C_6&+.c3+_1_6&C_1_6) +i(_1_6&c_1&-_1_6&c_3&+_1_6&c5&-.c3-_1_6&c_15) 이때 (1+i)^1^6={(1+i)^2}^8=(2i)^8=2^8=256 이고, _1_6&C_1=_1_6C_15, _1_6C_3=_1_6C_1_3, _1_6C5=_1_6C_1_1, _1_6C_7=_1_6C_9 이므로 _1_6C_1&-_1_6C_3&+_1_6C5&-.c3-_1_6C_15=0.t3 _1_6&C0&-_1_6&C_2&+_1_6&C_4&-_1_6&C_6&+ +_1_6&C_1_6=256 60% 127 이항계수의성질 전략 수로나타난다. 원소의개수가 2, 4, 6, 8, 10 인부분집합의개수를각각조합의 원소의개수가 2 인부분집합의개수는 1, 2, 3,, 10 의 10 개에서 2 개를택하는조합의수와같으므로 _10&C_2 같은방법으로원소의개수가 4, 6, 8, 10 인부분집합의개수 는각각 _10&C_4, _10&C_6, _10&C_8, _10&C_10 따라서구하는집합의개수는 _10&C_2&+_10&C_4&+_10&C_6&+_10&C_8&+_10&C_10=2^9&-1=511 n이짝수일때, _n&c0&+_n&c_2&+_n&c_4&+.c3+_n&c_n&=2^n-^1이므로 _n&c_2&+_n&c_4&+_n&c_6&+.c3+_n&c_n&=2^n-^1&-_n&c0&=2^n-^1& 조합의수 ; 함수의개수 1단계 f(1)<f(2)<f(3)<f(4)<f(5) 인함수 f의개수를구한다. f(1)<f(2)<f(3)<f(4)<f(5) 인함수 ~ f의개수는 Y의원소 1, 2, 3, 4, 5, 6의 6개중에서 5개를뽑는방법의수와같으므로 _6&C5=_6&C_1=6 2단계 f(1)=f(2)<f(3)<f(4)<f(5) 또는 f(1)<f(2)<f(3)=f(4)<f(5) 인함수 f의개수를구한다. f(1)=f(2)<f(3)<f(4)<f(5) 또는 f(1)<f(2)<f(3)=f(4)<f(5) 인함수 ~ f의개수는 Y의원소 6개중에서 4개를뽑는방법의수와같으므로 2 _6&C_4=2 _6&C_2=30 3단계 f(1)=f(2)<f(3)=f(4)<f(5) 인함수 f의개수를구한다. f(1)=f(2)<f(3)=f(4)<f(5) 인함수 f의개수는 Y의원소 6개중에서 3개를뽑는방법의수와같으므로 _6&C_3=20 4단계 f(1)-<f(2)<f(3)-<f(4)<f(5) 를만족시키는함수 f의개수를구한다. 따라서구하는함수 f의개수는 = 이항정리 1단계 a(x+4)^n의전개식에서 x^n-^1의계수를구한다. a(x+4)^n의전개식의일반항은 a_n&c_r& x^n-^r&4^r=a_n&c_r&~4^r&x^n-^r x^n-^1항은 n-r=n-1일때이므로 r=1 따라서 x^n-^1의계수는 a_n&c_1 4=4an 2단계 (x-1)(x-a)^n의전개식에서 x^n-^1의계수를구한다. (x-a)^n의전개식의일반항은 _n&c_r&x^n-^r(-a)^r=_nc_r(-1)^r&a^r&x^n-^r.c3.c3 ᄀ이때 (x-1)(x-a)^n의전개식에서 x^n-^1항은 x와ᄀ의 x^n-^2항, -1과ᄀ의 x^n-^1항이곱해질때나타난다. r1par ᄀ의 x^n-^2항은 n-r=n-2일때이므로 r=2.t3 _n&c_2(-1)^2&a^2&x^n-^2=upa^2n(n-1) ~2 `x^n-^2 18 바른답 알찬 해 (001~027) 일등확통 -01~03 강 ok.indd 오후 3:33

19 r2par ᄀ의 x^n-^1 항은 n-r=n-1 일때이므로 r=1.t3 _n&c_1 (-1)ax^n-^1=-anx^n-^1 r1par, r2par 에서 x^n-^1 항은 x upa^2n(n-1) ~2 x^n-^2&+(-1) (-anx^n-^1) =^{upa^2n(n-1) ~2 +an^}x^n-^1 이므로 x^n-^1의계수는 upa^2n(n-1) ~2 +an 3 단계두계수가같게되는모든순서쌍 (a, n) 에대하여 an 의최댓 값을구한다. 4an=upa^2n(n-1) ~2 +an에서 8an=a^2&n(n-1)+2an, n(n-1)a^2&-6an=0 a(n-1)-6=0 (.T3 a, n 은자연수 ).t3 a(n-1)=6 a, n 이될수있는수를표로나타내면다음과같다. n a , 즉 n a 따라서 an 의최댓값은 a=6, n=2 일때, 6 2= 같은것이있는순열전략 7개의케이크중 4개를택하는경우를모두구하고, 같은것이있는순열을이용하여각각의경우의수를구한다. 3가지종류의케이크를각각 a, b, c라하면 a, a, b, b, b, c, c의 7개중에서 4개를택하는경우는 aabb, aabc, aacc, abbb, abbc, abcc, bbcc, bbbc 이고각경우의케이크세트의개수는 low2!2! ~`~4! `=6, up4! 2!`=12, low2!2! ~`~4! `=6, up4! 3!`=4, up4! 2!`=12, up4! 2!`=12, low2!2! ~`~4! `=6, up4! 3!`=4 따라서구하는케이크세트의개수는 = 같은것이있는순열전략 C 지점과 D 지점을지나지않으면서 A 지점에서 B 지점까지최단거리로갈때, 반드시지나야하는점을찾아같은것이있는순열을이용한다. A P Q R D B C 위의그림과같이세지점 P, Q, R를잡으면 C 지점과 D 지점을지나지않고 A 지점에서 B 지점까지최단거리로가는방법은 A P Q R B이므로구하는경우의수는 실전대비평가문제 원순열 전략 I. 순열과조합 pp. 40~41 윗면과아랫면을칠하는방법의수를구하고, 원순열을이용하 여옆면을칠하는방법의수를구한후곱한다. _6P_2=30 사각뿔대의윗면과아랫면을칠하는방법의수는 윗면과아랫면에칠한색을제외한 4 가지색을사용하여옆 면을칠하는방법의수는 (4-1)!=3!=6 따라서구하는방법의수는 30 6= 중복순열전략중복순열을이용하여 a_n 을구한후, 등비수열의합을이용한다. a_n 은 3가지색의깃발중 n개를택하는중복순열의수와같으므로 a_n=_3pai_n=3^n.t3 sign=1 ^5``a_n=SIGn=1 ^5``3^n=up3(3^5-1) 3-1 =363 up4! 3!` up3! 2!` 1 2!= 중복조합 전략 수를이용한다. 공역의두원소의합이 10 이되는경우를구하고, 중복조합의 조건 에서공역의두원소의합이 10 이되는경우는 3+7=4+6=5+5=10 이므로이를순서쌍으로나타내면 (3, 7), (4, 6), (5, 5) 이 다. 이때함수 ~f~ 는 3 개의순서쌍중중복을허용하여 3 개를택한 후, 조건 를만족시키도록 ~ f(1), f(3), f(5), f(7) 에대응 시키면된다. 따라서구하는함수 ~f 의개수는 _3H_3=5C_3=5C_2= 중복조합 ; 정수해의개수 전략 노란장미, 분홍장미, 빨간장미의개수를각각 x, y, z 라하고 x+y+z=10 에서조건을만족시키는 x, y, z 의순서쌍 (x, y, z) 의개 수를구한다. 노란장미, 분홍장미, 빨간장미의개수를각각 x, y, z 라하면 x+y+z=10 이때 x->3, y->2, z->1 이므로 x=x+3, y=y+2, z=z+1 로놓고 x+y+z=10 에대 입하면 실전대비 Ⅰ 단원평가문제 19 해 (001~027) 일등확통 -01~03 강 ok.indd 오후 3:33

20 바른답 알찬 (X+3)+(Y+2)+(Z+1)=10.t3 X+Y+Z=4 따라서서로다른 3 개의문자에서 4 개를택하는중복조합의 수와같으므로 _3H_4=_6C_4=_6C_2= 집합의분할 전략 6 명을 2 개의조로나누어 3 개의정류장 A, B, C 중에서 2 개의 정류장에분배한다. 3 개의정류장 A, B, C 중에서승객이내리는 2 개의 정류장을택하는방법의수는 _3C_2=3 6 명의승객을 2 개의조로나눌때, 각조의인원수는 (1 명, 5 명 ) 또는 (2 명, 4 명 ) 또는 (3 명, 3 명 ) r1par 승객을 1 명, 5 명으로나누는방법의수는 _6C_1 5C5=6 r2par 승객을 2 명, 4 명으로나누는방법의수는 _6C_2 _4C_4=15 r3par 승객을 3 명, 3 명으로나누는방법의수는 _6C_3 _3C_3 low2! `1 =10 이상에서승객을 2 개의조로나누는방법의수는 =31 2 개의조를 2 개의정류장에분배하는방법의수는 2!=2 따라서구하는방법의수는 = 이항정리 6명의승객을 2개의조로나누어 2개의정류장에분배해야하므로승객을나누는방법의수에정류장에분배하는방법의수를반드시곱해야한다. 전략 한다. ^(x+1/x^)^^n+1 의전개식의일반항을이용하여 x^n-^3 의계수를구 ^(x+ 1 ^)^^n+1의전개식의일반항은 x _n+_1c_r x^n^+^1-^r^( 1 ^)^^r=_n+_1c_r x^n^+^1-^2^r x x^n-^3항은 n+1-2r=n-3일때이므로 r=2 따라서 x^n-^3의계수는 _n+_1c_2=up(n+1)n 2 `이므로 a_n= n(n+1) 2 1.t3 sign=1 ^10`&~ a_n =sign=1 ^10`lown(n+1) ``2 `=2sign=1 ^10`^(1/n - lown+1 `1 ^) =2^{^(1-1/2^)+^(1/2-1/3^)+.c3+^(1/10-1/11^)^} =2^(1-1/11^)=20/11 II 확률 03 확률의뜻과활용 교과서에서뽑은기본문제 138 ⑴ S={1, 2, 3, 4, 5, 6} ⑵ A={1, 3, 5} ⑶ B={1, 5} 139 1/ / ⑴ 0 ⑵ ⑴ 1 2/2 5 ⑵ 6/2 5 ` 143 7/8 138 ⑴ S={1, 2, 3, 4, 5, 6} ⑵ 홀수는 1, 3, 5이므로 A={1, 3, 5} ⑶ 5의약수는 1, 5이므로 B={1, 5} 139 두눈의수의합이 7이되는경우는 (1, 6), (2, 5), (3, 4), (4, 3), (5, 2), (6, 1) 의 6가지이므로구하는확률은 6/3 6 = 1/6 pp. 44~ 명중에서항체가생긴사람이 906 명이므로어떤사람에게이예방접종을하였을때, 항체가생길확률은 low1000 `906``= 453/ ⑴ 검은공은 2개이므로검은공이 3개나오는사건은절대로일어날수없다. 따라서구하는확률은 0이다. ⑵ 검은공이 2개이므로 3개의공을꺼낼때, 흰공이 1개이상나오는사건은반드시일어난다. 따라서구하는확률은 1이다. 142 ⑴ 카드에적힌수가 3의배수인사건을 A, 4의배수인사건을 B라하면카드에적힌수가 3의배수또는 4의배수인사건은 AhapB, 3의배수이면서 4의배수, 즉 12의배수인사건은 AcupB이다. n(a)=16, n(b)=12, n(acupb)=4이므로 P(A)=16/50, P(B)=12/50, P(AcupB)=4/50 따라서확률의덧셈정리에의하여구하는확률은 P(AhapB)=P(A)+P(B)-P(AcupB) = 1 6/ /5 0-4/5 0 = 1 2/2 5 ⑵ 카드에적힌수가 7의배수인사건을 A, 9의배수인사건을 B라하면카드에적힌수가 7의배수또는 9의배수인사건은 AhapB이다. 20 바른답 알찬 해 (001~027) 일등확통 -01~03 강 ok.indd 오후 3:33

21 n(a)=7, n(b)=5 이므로 n^3=64.t3 n=4 (.T3 n 은자연수 ) P(A)=7/50, P(B)=5/50 두사건 A 와 B 는서로배반사건이므로확률의덧셈정리 에의하여구하는확률은 P(AhapB)=P(A)+P(B) = 7/ /5 0 = 6/ 앞면이적어도한번나오는사건을 A 라하면 A^C 는세번모두뒷면이나오는사건이므로 P(A^C)=#1/@2^3 $=1/8 두시행에서모든경우의수각각구하기 40% n에대한식세우기 30% n의값구하기 30% 147 서로다른세개의동전을동시에던질때, 모든경우의수는 2 2 2=8 앞면이 2개인경우는 ( 앞면, 앞면, 뒷면 ), ( 앞면, 뒷면, 앞면 ), ( 뒷면, 앞면, 앞면 ) 의 3가지이다. 03 확률의뜻과활용 따라서구하는확률은 P(A)=1-P(A^C)=1-1/8=7/8 따라서구하는확률은 3/ 장의카드에서 2 장을뽑는방법의수는 _4C_2=6 이고, 이중에서 한 과 국 이적힌카드를뽑는방법의수는 1 이다. 따라서확률은 1/6 이므로 p=6, q=1 기출문제 pp. 46~52.t3 10p+q=10 6+1= / / / / ⑴ 1/2 0 ⑵ 1/5 5 ⑶ 1/2 2 ⑷ 3/ / 표본공간을 S라하면 S={1, 2, 3, 4, 5} A={2, 4}, B={2, 3, 5} 2 B^C={1, 4} 3 AhapB={2, 3, 4, 5} 5 A^C={1, 3, 5} 따라서옳은것은 4이다. 145 표본공간을 S라하면 S={1, 2, 3, 4, 5, 6} A={2, 4, 6}, B={2, 3, 5}, C={1, 3}, D={3, 6} 이므로 AcupB={2}, AcupC=, BcupC={3} BcupD={3}, CcupD={3} 따라서사건 A와사건 C는서로배반사건이다 부터 n까지의자연수가각각적힌정n면체 3개를던지는시행에서모든경우의수는 n^3이고, 동전 1개를던지는시행에서모든경우의수는 2이다. 이때표본공간의원소의개수가 128이므로 2n^3= 한개의주사위를 2번던질때, 모든경우의수는 6 6=36 r1par 두눈의수의합이 8인경우 (2, 6), (3, 5), (4, 4), (5, 3), (6, 2) 의 5가지 r2par 두눈의수의합이 9인경우 (3, 6), (4, 5), (5, 4), (6, 3) 의 4가지 r3par 두눈의수의합이 10인경우 (4, 6), (5, 5), (6, 4) 의 3가지 r4par 두눈의수의합이 11인경우 (5, 6), (6, 5) 의 2가지 r5par 두눈의수의합이 12인경우 (6, 6) 의 1가지이상에서두눈의수의합이 8 이상인경우의수는 =15 따라서구하는확률은 1 5/3 6 & = 5/1 2 모든경우의수구하기 30% 두눈의수의합이 8 이상인경우의수구하기 50% 두눈의수의합이 8 이상일확률구하기 20% 150 1부터 100까지의자연수중에서 6과서로소인수의개수는 100-{(2의배수의개수 )+(3의배수의개수 ) -(6의배수의개수 )} =100-( )=33 따라서구하는확률은 3/ 개의알파벳 a, b, c, d, e를일렬로나열하는경우의수는 5!&= 확률의뜻과활용 21 해 (001~027) 일등확통 -01~03 강 ok.indd 오후 3:33

22 바른답 알찬 a, b를양끝에배정하고 c, d, e를그사이에일렬로나열하는경우의수는 2!& 3!&=2 6=12 따라서구하는확률은 12/120 = 1/1 0 1등급비법서로다른 n개를 p개, q개, r개, s개 (p+q+r+s=n) 로분할하는방법의수는 p, q, r, s가모두같은수일때, _nc_p _n-_pc_q _n-_p-_qc_r _sc_s low4! `1 152 A, B, C, D, E 의 5 명중 2 명의대표를뽑는방법의수는 5C_2=10 B 가대표로뽑히는방법의수는 B 를제외한나머지 4 명중 1 명을뽑는방법의수와같으므로 _4C_1=4 따라서구하는확률은 4/1 0 = 2/5 서로다른 n 개에서특정한 k 개를포함하여 r 개를뽑는방법의수 는 (n-k) 개에서 (r-k) 개를뽑는방법의수와같으므로 _n-_kc_r-_k 이다. 153 김치 2 종류와나물 4 종류중에서임의로 3 종류의반찬을선택하는방법의수는 _6C_3=20 김치 1 종류와나물 2 종류를선택하는방법의수는 _2C_1 _4C_2=2 6=12 따라서 p=12/20=3/5 이므로 20p=20 3/5=12 3 종류의반찬을선택하는방법의수구하기 30% 김치 1 종류와나물 2 종류를선택하는방법의수구하기 30% 20p 의값구하기 40% 154 정육면체의 8 개의꼭짓점중에서서로다른두꼭짓점을택하는방법의수는 _8C_2=28 이중에서선분의길이가 rt2 이상인경우는 rt2 또는 rt3 r1par 선분의길이가 rt2~ 인경우의수는 각면의대각선의개수와같으므로 2 6=12 r2par 선분의길이가 rt3~ 인경우의수는 정육면체의대각선의개수와같으므로 4 r1par, r2par 에서선분의길이가 rt2 이상인경우의수는 12+4=16 따라서구하는확률은 1 6/2 8 = 4/7 155 전체 8 명이임의로 2 명씩짝을짓는방법의수는 _8C_2 _6C_2 _4C_2 _2C_2 1 4! =105 남학생은남학생끼리, 여학생은여학생끼리 2 명씩짝을짓는 방법의수는 _4C_2 _2C_2 1 2! _4C_2 _2C_2 1 2! =9 따라서구하는확률은 9/105 = 3/ 주사위 1개와동전 5개를동시에던질때, 모든경우의수는 6 2^5=192 a=3b인경우는 a=3, b=1 또는 a=6, b=2 r1par a=3, b=1인경우의수는 1 up5! 4!`=1 5=5 r2par a=6, b=2 인경우의수는 1 low2!3! 5! `=1 10=10 r1par, r2par 에서 a=3b 인경우의수는 5+10=15 따라서구하는확률은 1 5/19 2 = 5/ 서로다른세개의주사위를동시에던질때, 모든경우의수는 6 6 6=216 한주사위의눈의수가다른두개의주사위의눈의수의곱 이되는경우는 (1, 1, 1), (1, 2, 2), (1, 3, 3), (1, 4, 4), (1, 5, 5), (1, 6, 6), (2, 2, 4), (2, 3, 6) r1par (1, 1, 1) 인경우의수는 1 r2par (1, 2, 2), (1, 3, 3), (1, 4, 4), (1, 5, 5), (1, 6, 6), (2, 2, 4) 인경우의수는 ``2!` 6=3 6=18 up3! r3par (2, 3, 6) 인경우의수는 3&!=6 이상에서한주사위의눈의수가다른두개의주사위의눈의수의곱이되는경우의수는 =25 따라서구하는확률은 2 5/ 씨앗이싹이틀확률은 low1200 `925``= 3 7/4 8 따라서 a=37, b=48 이므로 a+b= 단계까지통과한사람은 명이고 5 단계까지통과한사람은 명이므로 p=low `= 5/2 1.t3 42p=42 5/21= 점 P가 ^-AB^- 를지름으로하는반원위 D C 에있을때, semoapb는직각삼각형이 된다. P 따라서오른쪽그림의색칠한부분에 점 P 를잡으면 semo&apb 가둔각삼각형 A B 22 바른답 알찬 해 (001~027) 일등확통 -01~03 강 ok.indd 오후 3:33

23 이되므로구하는확률은 ㄴ. 0-<P(A)-<1, 0-<P(B)-<1 이므로 ( 색칠한부분의넓이 1/2 pai 1^2 ) (nemoabcd의넓이 ) = = pai/8 4 1등급비법반원에대한원주각의크기는 90 이므로원의지름의양끝점과다른한점을택하면직각삼각형을만들수있다. A P O B 0-<P(A)+P(B)-<2 ㄷ. /<(AhapB)/<S이므로 P( )-<P(AhapB)-<P(S).t3 0-<P(AhapB)-<1 이상에서옳은것은ㄱ, ㄴ이다. 164 두사건 A, B가서로배반이므로 P(AcupB)=0 P(AhapB)=P(A)+P(B)-P(AcupB) 에서 03 확률의뜻과활용 161 세선분의길이를각각 x, y, a-x-y라하면 0<x<a, 0<y<a, 0<a-x-y<a.t3 0<x<a, 0<y<a, 0<x+y<a.c3.c3 ᄀ ᄀ을모두만족시키는부등식의영 역은오른쪽그림의색칠한부분과 같다. ( 단, 경계선제외 ) 이때세선분이삼각형의세변이 되려면 x+y>a-x-y, x+(a-x-y)>y, y+(a-x-y)>x.t3 x+y>a/2, y < a/2, x < a/2.c3.c3 ᄂ ᄂ을모두만족시키는부등식의영역은위의그림의빗금친 부분과같다. ( 단, 경계선제외 ) 따라서구하는확률은 ( 빗금친부분의넓이 1/2 a/2 a/2 & ) ( 색칠한부분의넓이 ) = = 1/4 1/2 a a 세선분의길이의범위구하기 30% 세선분이삼각형의세변이될조건구하기 40% 세선분이삼각형의세변이될확률구하기 30% 삼각형의두변의길이의합은나머지한변의길이보다크다. 즉, 오른쪽그림의 semoabc에서 a+b>c, b+c>a, c+a>b 162 표본공간 S={2, 4, 6, 8, 10} 에서홀수는존재하지않으므로 P(A)=0 또, 집합 S 의모든원소는 2 의배수이므로 P(B)=1.t3 P(A)+P(B)=1 163 ㄱ. 확률의기본성질에의하여 0-<P(A)-<1 y a a - 2 O B a a - 2 c a A b C x 3/5 = 1/4 + P(B).t3 P(B)=7/ 이적힌카드를뽑는사건을 A, 15 가적힌카드를뽑는사건을 B 라하면 P(A)= _1_4C_1 _15C_2 = 1 4/10 5 P(B)= _1_4C_1 _15C_2 = 1 4/10 5 P(AcupB)= 1/10 5 따라서구하는확률은 P(AhapB)=P(A)+P(B)-P(AcupB) =14/105+14/105-1/105=9/35 P(A) 는 7이적힌카드를제외한 14장의카드에서 1장을뽑을확률과같고, P(B) 는 15가적힌카드를제외한 14장의카드에서 1 장을뽑을확률과같다. 166 f(3)=5인사건을 A, f(5)=7인사건을 B라하면 P(A)= _8PAI_4 =# /@8^5!8^^4 $=1/8 _8PAI5.c3.c3 P(B)= _8PAI_4 =# /@8^5!8^^4 $=1/8 _8PAI5.c3.c3 P(AcupB)= _8PAI_3 =# /@8^5!8^^3 $=1/64 _8PAI5.c3.c3 따라서구하는확률은 P(AhapB)=P(A)+P(B)-P(AcupB) =1/8+1/8-1/64=15/64 f(3)=5, f(5)=7인사건을각각 A, B라하고 P(A) 의값구하기 30% P(B) 의값구하기 30% P(AcupB) 의값구하기 30% P(AhapB) 의값구하기 10% 03. 확률의뜻과활용 23 해 (001~027) 일등확통 -01~03 강 ok.indd 오후 3:33

24 바른답 알찬 1 f(3)=5 인함수 f 의개수는 Y 의원소 8 개에서중복을허용하 여 4 개를택하여 X 의원소 1, 2, 4, 5 에대응시키는방법의수 와같다. 2 f(3)=5, f(5)=7 인함수 f 의개수는 Y 의원소 8 개에서중복 을허용하여 3 개를택하여 X 의원소 1, 2, 4 에대응시키는방 법의수와같다. 167 꺼낸공 2 개가모두흰공인사건을 A, 모두검은공인사건을 B 라하면 P(A)= _2C_2 5C_2 = 1/1 0 P(B)= _3C_2 5C_2 = 3/1 0 두사건 A, B는서로배반사건이므로구하는확률은 P(AhapB)=P(A)+P(B) 168 ⑴ _3C_3 _1_2C_3 = 1/2 0 =1/10+3/10=2/5 ⑵ _4C_3 _1_2C_3 =4/20&=1/55 ⑶ 5C_3 _1_2C_3 =10/20=1/22 ⑷ 대표 3명이모두 1학년인사건을 A, 2학년인사건을 B, 3학년인사건을 C라하면세사건 A, B, C는서로배반사건이므로구하는확률은 P(AhapBhapC)=P(A)+P(B)+P(C) =1/20+1/55+1/22=3/44 대표 3 명이모두 1 학년일확률구하기 30% 대표 3 명이모두 2 학년일확률구하기 30% 대표 3 명이모두 3 학년일확률구하기 30% 대표 3 명이모두같은학년일확률구하기 10% 169 P(AhapB)=P(A)+P(B)-P(AcupB) = 5/6 + 1/2-2/3 = 2/3 A^CcupB^C=(AhapB)^C이므로 P(A^C&cupB^C)=P((AhapB)^C)=1-P(AhapB) =1-2/3=1/3 170 적어도한개가검은공인사건을 A라하면 A^C는 4개가모두흰공인사건이므로 P(A^C)= 5C_4 _9C_4 = 5/12 6.t3 P(A)=1-P(A^C)=1-5/126=121/ 두눈의수의곱이소수가아닌사건을 A 라하면 A^C는두눈의수의곱이소수인사건이므로 A^C={(1, 2), (1, 3), (1, 5), (2, 1), (3, 1), (5, 1)}.t3 P(A^C)=6/36=1/6.t3 P(A)=1-P(A^C)=1-1/6=5/6 172 두학생 A, B가서로이웃하지않는사건을 A라하면 A^C는 A, B가서로이웃하는사건이므로 P(A^C)=up4! 2! ` 5! ``= 2/5.t3 P(A)=1-P(A^C)=1-2/5=3/5 173 A, B 중적어도한명이기차를타는사건을 A라하면 A^C는 A, B가모두버스를타는사건이므로 P(A^C)= _4C_1 ^(_6C_3 _3C_3 low2! `1 ^) 2! =4/20=1/5.t3 P(A)=1-P(A^C)=1-1/5=4/5 P(A^C)= (A, B를제외한 4명중버스에탈 1명을뽑는방법의수 ) (6명을세명씩두조로나누고두조를기차와버스에배정하는방법의수 ) 174 당첨제비의개수를 n, 적어도 1 개의당첨제비를뽑는사건을 A 라하면 A^C 는 3 개모두당첨제비를뽑지않는사건이 므로 P(A^C)= _9-_nC_3 =up(9-n)(8-n)(7-n) ~9 8 7 _9C_3 이때 P(A)=20/21 이므로 P(A^C)=1-P(A)=1-&20/21=1/21 up(9-n)(8-n)(7-n) ~9 8 7 ~=1/21& 에서 (9-n)(8-n)(7-n)=24=4 3 2 이므로 9-n=4.t3 n=5 따라서당첨제비의개수는 5 이다. 175 A 가문제를맞히는사건을 A, B 가문제를맞히는사건을 B 라하자. 두명중한명만문제를맞힐확률이 0.6 이므로 P(AhapB)-P(AcupB)=0.6 이때 P(AcupB)=0.3 이므로 P(AhapB)-0.3=0.6.t3 P(AhapB)=0.9 A, B 모두문제를틀리는사건은 A^CcupB^C 이므로 구하는확률은 P(A^CcupB^C) =P((AhapB)^C) =1-P(AhapB) =1-0.9= 바른답 알찬 해 (001~027) 일등확통 -01~03 강 ok.indd 오후 3:33

25 1 등급문제 pp. 53~ / / / 수학적확률 전략 판별식을이용하여이차방정식이실근을가질조건을구한다. 이차방정식 ax^2&-8x+b=0이실근을가지려면이이 차방정식의판별식을 D라할때, ;D/4 :=(-4)^2&-ab->0, 즉 ab-<16이어야한다. 주사위를두번던질때, 모든경우의수는 6 6=36 ab-<16인경우는 r1par a=1일때, b=1, 2, 3, 4, 5, 6의 6가지 r2par a=2일때, b=1, 2, 3, 4, 5, 6의 6가지 r3par a=3일때, b=1, 2, 3, 4, 5의 5가지 r4par a=4일때, b=1, 2, 3, 4의 4가지 r5par a=5일때, b=1, 2, 3의 3가지 r6par a=6일때, b=1, 2의 2가지 이상에서 ab-<16인경우의수는 =26 따라서구하는확률은 2 6/3 = 3/ 수학적확률 전략 두수의곱이 -1이되는경우를생각한다. 주사위를두번던질때, 모든경우의수는 6 6=36 i ^m (-1)^n=-1인경우는 i ^m=-1, (-1)^n=1 또는 i ^m=1, (-1)^n=-1 r1par i ^m=-1, (-1)^n=1인경우 m=2 또는 m=6이고 n=2 또는 n=4 또는 n=6이므로 순서쌍 (m, n) 은 (2, 2), (2, 4), (2, 6), (6, 2), (6, 4), (6, 6) 의 6가지 r2par i ^m=1, (-1)^n=-1인경우 m=4이고, n=1 또는 n=3 또는 n=5이므로 순서쌍 (m, n) 은 (4, 1), (4, 3), (4, 5) 의 3가지 r1par, r2par 에서 i ^m (-1)^n=-1인경우의수는 6+3=9 따라서구하는확률은 9/3 6 = 1/4 모든경우의수구하기 20% i~^m (-1)^n=-1인경우의수구하기 60% 확률구하기 20% 음이아닌정수 k에대하여 i~^4^k=1, i~^4^k^+^1=i, i~^4^k^+^2=-1, i~^4^k^+^3=-i 이므로 i~^m에서 m이 2의배수인경우에대해서생각한다. 178 수학적확률전략수형도를이용하여한학생만자신의성적표를선택하는경우를구한다. 5명의학생이 5개의성적표를한장씩선택하는모든경우의수는 5&!&=120 5명의학생을 A, B, C, D, E라하고 A학생만자신의성적표를선택하고나머지네학생은다른학생의성적표를선택하는경우를구해보면다음과같이 9가지이다. A B C D E B E D C D E B E B D B E C A D B C E C B B C D E B C D C B 같은방법으로 B, C, D, E학생이각각자신의성적표를선택하고나머지네학생이다른학생의성적표를선택하는경우도각각 9가지씩이다. 따라서한사람만자신의성적표를선택하는경우의수는 9 5=45이므로구하는확률은 45/120 = 3/8 1등급비법수형도를이용하면중복되지않고빠짐없이모든경우를구할수있다. 179 수학적확률전략어느두사람도이웃하지않으려면먼저빈의자 4개를놓고그사이사이에세사람이앉으면된다. 7개의의자에세사람이앉는경우의수는한사람이임의의한의자에앉고, 나머지두사람이차례로의자에앉는경우의수와같으므로 1 6 5=30 이때세사람중어느두사람도이웃하지않게앉는경우의수는오른쪽그림과같이빈의자 4개사이의 4개의공간에세사람이앉는경우의수와같으므로 03 확률의뜻과활용 03. 확률의뜻과활용 25 해 (001~027) 일등확통 -01~03 강 ok.indd 오후 3:33

26 바른답 알찬 1 3 2=6 따라서구하는확률은 6/3 0 = 1/5 180 수학적확률 전략 먼저 3 학년학생 3 명을일렬로세운후같은학년학생끼리서 로이웃하지않도록세우는방법을생각한다. 6 명을일렬로세우는방법의수는 6!=720 r1par 3 학년학생 3 명을일렬로세우고, 3 학년학생사이사이에 2 학년학생 2 명을세운후, 5 명사이사이와양끝에 1 학 년학생 1 명을세우는방법의수는 3! 2! _6C_1=6 2 6=72 r2par 3 학년학생 3 명을일렬로세우고, 3 학년학생사이사이에 2 학년학생 1 명과 1 학년학생 1 명을세운후, 5 명의양끝 에 2 학년학생 1 명을세우는방법의수는 3! (_2C_1 2!) _2C_1=6 4 2=48 r1par, r2par 에서같은학년학생끼리이웃하지않도록세우는방 법의수는 72+48=120 따라서구하는확률은 120/720 = 1/6 181 수학적확률 전략중복순열과중복조합을이용한다. X 에서 Y 로의함수의개수는 Y 의원소 1, 2, 3, 4 의 4 개에서중복을허용하여 3 개를택하는중복순열의수와같 으므로 _4PAI_3=4^3=64 주어진조건을만족시키는함수 ~ f~ 는 Y 의원소 4 개에서중복 을허용하여 3 개를택한후, 작은수부터차례로 ~f(1), f(2), f(3) 에대응시키면된다. 이때함수 f 의개수는 4 개에서중복을허용하여 3 개를택하 는중복조합의수와같으므로 _4H_3=_6C_3=20 따라서구하는확률은 2 0/6 4 = 5/1 6 이므로 p=16, q=5 182 확률의덧셈정리.t3 p+q=21 전략 P(AhapB)=P(A)+P(B)-P(AcupB) 임을이용한다. 나온수의최솟값이 4 인사건을 A, 최댓값이 8 인사 건을 B 라하면 P(A)= _8C_3 _1_2C_4 = 5 6/49 5 P(B)= _7C_3 _1_2C_4 =35/495=7/99 P(AcupB)= _3C_2 _1_2C_4 =3/495=1/165 따라서구하는확률은 P(AhapB)=P(A)+P(B)-P(AcupB) =56/495+7/99-1/165=8/45 나온수의최솟값이 4일확률구하기 20% 나온수의최댓값이 8일확률구하기 20% 나온수의최솟값이 4이고, 최댓값이 8일확률구하기 20% 나온수의최솟값이 4이거나최댓값이 8일확률구하기 40% (5부터 12까지의 8개에서 3개를뽑는방법의수 ) P(A)= (1부터 12까지의 12개에서 4개를뽑는방법의수 ) (1부터 7까지의 7개에서 3개를뽑는방법의수 ) P(B)= (1부터 12까지의 12개에서 4개를뽑는방법의수 ) (5, 6, 7의 3개에서 2개를뽑는방법의수 ) P(AcupB)= (1부터 12까지의 12개에서 4개를뽑는방법의수 ) 183 확률의덧셈정리전략세수의합이짝수가되는경우를구하고, 확률의덧셈정리를이용한다. 세수의합이짝수가되는경우는 ( 홀수, 홀수, 짝수 ), ( 짝수, 짝수, 짝수 ) 의두가지경우이다. 홀수, 홀수, 짝수가나오는사건을 A, 짝수, 짝수, 짝수가나오는사건을 B라하면 P(A)= 5C_2 _4C_1 = 4 0/8 = 1 0/2 _9C_3 P(B)= _4C_3 = 4/8 4 = 1/2 1 _9C_3 두사건 A, B는서로배반사건이므로구하는확률은 P(AhapB)=P(A)+P(B)=10/21+1/21=11/ 확률의덧셈정리전략상자 A에서빨간공 1개, 검은공 1개를꺼내는경우와검은공 2 개를꺼낸후빨간공 1개, 검은공 1개를꺼내는경우로나누어생각한다. r1par [ 실행 1] 에의하여상자 B에있는빨간공의개수가 1인경우 A에서 2개의공을꺼내는경우의수는 _8C_2=28 A에서빨간공 1개와검은공 1개를꺼내는경우의수는 _3C_1& 5C_1=15 이므로그확률은 1 5/2 8 r2par [ 실행 2] 에의하여상자 B에있는빨간공의개수가 1인경우 A에서 2개의공을꺼내고, 다시 2개의공을꺼내는경우의수는 _8C_2& _6C_2=420 A에서검은공 2개를꺼내고, 다시빨간공 1개와검은공 1개를꺼내는경우의수는 5C_2& _3C_1& _3C_1=90 이므로그확률은 90/420 = 3/1 4 r1par, r2par 는서로배반사건이므로구하는확률은 1 5/ /1 4 = 3/4 26 바른답 알찬 해 (001~027) 일등확통 -01~03 강 ok.indd 오후 3:33

27 185 여사건의확률 전략 P(A)=1-P(A^C) 임을이용한다. (a-b)(b-c)(c-a)=0 a=b 또는 b=c 또는 c=a a, b, c 중적어도 2 개의수가같다. a, b, c 중적어도 2 개의수가같은사건을 A 라하면 A^C 는 a, b, c 가모두다른수인사건이므로 P(A^C)= _6P_3 = 120/216 = 5/ t3 P(A)=1-P(A^C)=1-5/9=4/9 주어진조건의의미파악하기 30% 여사건의확률구하기 40% 확률구하기 30% 186 확률의덧셈정리 + 여사건의확률 전략 P(AhapB)=1-P(A^CcupB^C) 임을이용한다. 적어도한명의 1 학년학생이뽑히는사건을 A, 적어도한명의여학생이뽑히는사건을 B 라하면 A^C 는 2 명모두 2 학년이뽑히는사건이고 B^C 는 2 명모두남학생이뽑히는사건이므로 P(A^C)= _10C_2 _10C_2 = 45/120 = 3/8, P(B^C)= _1_6C_2 _1_6C_2 = 3/8.t3 P(A)=1-P(A^C)=1-3/8=5/8 2단계택한 2개의부분집합중하나가다른하나의부분집합인경우의수를구한다. 둘중하나가다른하나의부분집합인경우는 r1par 둘중하나가원소가 4개인부분집합일때, 다른하나는이집합의부분집합이므로경우의수는 _4C_4 2^4=16 r2par 둘중하나가원소가 3개인부분집합일때, 같은방법으로경우의수는 _4C_3 2^3=32 r3par 둘중하나가원소가 2개인부분집합일때, 같은방법으로경우의수는 _4C_2 2^2=24 r4par 둘중하나가원소가 1개인부분집합일때, 같은방법으로경우의수는 _4C_1 2^1=8 r5par 둘중하나가공집합일때, 경우의수는 _4C0 1=1 이상에서둘중하나가다른하나의부분집합인경우의수는 =81 3단계확률을구하고, p+q의값을구한다. 따라서둘중하나가다른하나의부분집합일확률은 8 1/13 6 이므로 p=136, q=81.t3 p+q= 수학적확률 1단계 8개의의자에앉는방법의수를구한다. 03 확률의뜻과활용 P(B)=1-P(B^C)=1-3/8=5/8 이때 A^CcupB^C 는 2 명모두 2 학년인남학생이뽑히는사건이 므로 P(A^CcupB^C)= _6C_2 = 15/120 = 1/8 _1_6C_2.t3 P(AhapB)=1-P((AhapB)^C)=1-P(A^CcupB^C) =1-1/8=7/8 따라서구하는확률은 P(AcupB) 이므로 P(AhapB)=P(A)+P(B)-P(AcupB) 에서 7/8 = 5/8 + 5/8 - P(AcupB).t3 P(AcupB)=3/8 187 수학적확률 1 단계집합 {1, 2, 3, 4} 의부분집합중에서서로같은집합을포함하 여 2 개의집합을택하는경우의수를구한다. 집합 {1, 2, 3, 4} 의부분집합의개수는 2^4=16 16 개의부분집합중에서서로같은집합을포함하여 2 개의 집합을택하는경우의수는 16 개중에서중복을허용하여 2 개를택하는중복조합의수와같으므로 _1_6H_2=_1_7C_2=136 두개의원형의탁자가서로구별되지않으므로맨처음사람 이의자에앉는방법의수는 4 이고나머지 7 명이차례로의 자에앉는방법의수는 7! 이다. 따라서 8 개의의자에앉는방법의수는 4 7! 2 단계어두운의자에는남자가앉고, 부부끼리는같은원형의탁자에 서마주보도록앉는방법의수를구한다. 4 명의남자중 2 명을택하여어두운의자에앉히는방법의 수는 _4C_2 이고, 부부끼리는같은원형의탁자에서마주보고 앉으므로남은의자는 4 개이다. 또, 남은의자중하나의의자에나머지 4 명중 1 명을앉히는방 법의수는 4 이고맞은편의자는채워지므로남은의자는 2 개 이다. 나머지 2 명을 2 개의의자에앉히는방법의수는 2! 이다. 따라서어두운의자에는남자가앉고, 부부끼리는같은원형 의탁자에서마주보도록앉는방법의수는 _4C_2 4 2! 3 단계 5! p 의값을구한다. 따라서 p= _4C_2 4 2! 이므로 4 7! 5! p=5! _4C_2 4 2! = 2/7 4 7! 03. 확률의뜻과활용 27 해 (001~027) 일등확통 -01~03 강 ok.indd 오후 3:33

28 바른답 알찬 04 조건부확률 교과서에서뽑은기본문제 p ⑴ 2/5 ⑵ 3/ ⑴ 1/6 ⑵ 5/6 ⑶ 1/6 ⑷ 1/ /1 6 P(B A)= P(AcupB) P(A) 1/1 6 = = 1/9 9/1 6 1등급비법사건 A가일어났을때사건 B가일어날조건부확률은 P(B A)= P(AcupB) `이므로주어진조건을이용하여분모, 분 P(A) 자의확률을각각구한다. 189 ⑴ P(A B)= P(A B) P(B) 1/5 = = 2/5 1/2 ⑵ P(B A)= P(A B) 1/5 P(A) = = 3/1 0 2/3 190 ⑴ P(A B)=P(A)P(B)=1/3 1/2=1/6 ⑵ P(A^C&hapB^C)=P((A B)^C)=1-P(A B) =1-1/6=5/6 ⑶ P(A B^C)=P(A)P(B^C)=P(A){1-P(B)} = 1/3 1/2 = 1/6 ⑷ P(A^C B)=P(A^C)P(B)={1-P(A)}P(B) = 2/3 1/2 = 1/3 191 한개의동전을던질때앞면이나올확률은 1/2 이므로동전 을 5 번던질때, 앞면이 3 번나올확률은 _5&C_3(1/2)^^3&(1/2)^^2&=10 1/3 2 = 5/ P(A B)= P(AcupB) 이므로 P(B) P(B)= P(AcupB) 1/8 P(A B) = = 5/8 1/5 P(AhapB)=P(A)+P(B)-P(AcupB) = 1/3 + 5/8-1/8 = 5/6 P(B^C A^C)= P(B^CcupA^C) = P((AhapB)^C) P(A^C) P(A^C) = 1-P(AhapB) 1-P(A) 1-5/6 = 1-1/3 1/6 = =1/4 2/3 P(B) 의값구하기 30% P(AhapB) 의값구하기 30% P(B^C A^C) 의값구하기 40% 기출문제 / / / /4 206 ⑴ 0.32 ⑵ 3/ / / / P(AhapB)=P(A)+P(B)-P(AcupB) 이므로 P(AcupB)=P(A)+P(B)-P(AhapB) =9/16+1/4-3/4=1/16 pp. 57~ P(B A)= P(AcupB) P(A) = P(AcupB) =1 0/7 &~P(AcupB) 이므로 0.7 P(AcupB) 의값이최대일때, P(B A) 의값이최대가되고, P(AcupB) 의값이최소일때, P(B A) 의값이최소가된다. r1par B/<A이면 P(AcupB) 의값이최대이고, P(AcupB)=P(B)=0.5.t3 P(B A)=10/7\0.5= 5/7.t3 M = 5/7 r2par P(AhapB)=P(A)+P(B)-P(AcupB)=1이면 P(AcupB) 의값이최소이고, P(AcupB)=P(A)+P(B)-1= =0.2.t3 P(B A)=10/7\0.2= 2/7.t3 m = 2/7 r1par, r2par 에서 M+m=5/7+2/7=1 28 바른답 알찬 해 (028~056) 일등확통 -04~06 강 ok.indd 오후 3:33

29 195 P(A)=0.6, P(AcupB)=0.35 이므로사건 A 가일어났을때, 사건 B 가일어날확률은 P(B A)= P(AcupB) = 0.35 P(A) 0.6 = 7/ 주사위를던져서짝수의눈이나오는사건을 A, 소수의눈이 갑과을이이웃하여서있는사건을 A, 을과병이이웃하여서있는사건을 B라하고 P(A) 의값구하기 30% P(AcupB) 의값구하기 30% P(B A) 의값구하기 30% p+q의값구하기 10% 나오는사건을 B 라하면 P(A)=1/2 짝수의눈중에서소수의눈은 2 뿐이므로 P(AcupB)=1/6 따라서구하는확률은 P(B A)= P(AcupB) P(A) 1/6 = = 1/3 1/2 197 상자에서빨간카드를뽑는사건을 A, 홀수가적힌카드를뽑는사건을 B 라하면 P(A)=5/9, P(AcupB)=3/9=1/3 200 첫번째에흰공이나오는사건을 A, 두번째에검은공이나오는사건을 B 라하면첫번째에흰공, 두번째에검은공 이나올확률은 P(AcupB)=P(A)P(B A)=3/7 4/6=2/7 201 갑이당첨제비를뽑는사건을 A, 을이당첨제비를뽑는사건을 B 라하면갑이당첨제비를뽑지못하고을만당첨제 비를뽑을확률은 P(A^CcupB)=P(A^C)P(B A^C)=9/12 3/11=9/44 04 조건부확률 따라서구하는확률은 P(B A)= P(AcupB) P(A) 1/3 = = 3/5 5/9 198 A 또는 B 가회장으로뽑히는사건을 M, F 가부회장으로뽑히는사건을 N 이라하면 P(M)=2/6=1/3 P(McupN)=1/6 1/5 + 1/6 1/5 = 1/15 따라서구하는확률은 P(N M)= P(McupN) 1/1 5 = = 1/5 P(M) 1/3 199 갑과을이이웃하여서있는사건을 A, 을과병이이웃하여서있는사건을 B 라하면 갑, 을두명을하나로묶어서생각하여 9 명을한줄로세우 는경우의수는 9! 이고, 갑과을이자리를바꾸는경우는 2 가지이므로 P(A)= 9! 2 = 1/5 10! 갑, 을, 병세명을하나로묶어서생각하여 8 명을한줄로 세우는경우의수는 8! 이고, 갑과을이이웃하면서을과병 이이웃하여서는경우는 2 가지이므로 P(AcupB)= 8! 2 = 1/4 5 10! P(B A)= P(AcupB) 1/4 5 = = 1/9 P(A) 1/5 따라서 p=9, q=1 이므로 p+q= r1par B 의주사위에서나온눈의수가 3 인경우 A 의주사위에서나온눈의수가 2, C 의주사위에서나온 눈의수가 1 일때, B 가우승하므로그확률은 3/6 2/6 3/6 = 1/12 r2par B 의주사위에서나온눈의수가 4 인경우 A 의주사위에서나온눈의수가 2, C 의주사위에서나온 눈의수가 1 일때, B 가우승하므로그확률은 3/6 2/6 3/6 = 1/12 r1par, r2par 에서 B 가우승할확률은 1/ /1 2 = 1/6 203 A 가소수가적힌공을뽑는사건을 A, B 가소수가적힌공을뽑는사건을 B 라하면 r1par A 가소수가적힌공을뽑고, B 도소수가적힌공을뽑을 확률은 P(AcupB)=P(A)P(B A)=4/10 3/9=2/15 r2par A 가소수가아닌수가적힌공을뽑고, B 는소수가적힌 공을뽑을확률은 P(A^CcupB)=P(A^C)P(B A^C)=6/10 4/9=4/15 r1par, r2par 에서 B 가소수가적힌공을뽑을확률은 P(B)=P(AcupB)+P(A^CcupB)=2/15+4/15=2/5 A 가소수가적힌공을뽑고, B 도소수가적힌공을 뽑을확률구하기 A 가소수가아닌수가적힌공을뽑고, B 는소수가 적힌공을뽑을확률구하기 40% 40% B 가소수가적힌공을뽑을확률구하기 20% 04. 조건부확률 29 해 (028~056) 일등확통 -04~06 강 ok.indd 오후 3:33

30 바른답 알찬 204 상자 A, B 를택하는사건을각각 A, B 라하고, 초록색구슬이나오는사건을 E 라하면 P(AcupE)=P(A)P(E A)=1/2 4/10=1/5 P(BcupE)=P(B)P(E B)=1/2 5/8=5/16.t3 P(E)=P(AcupE)+P(BcupE)=1/5+5/16=41/80 따라서구하는확률은 P(A E)=~ P(AcupE) P(E) 1/5 ~=~ ~= 1 6/4 4 1/8 0 사건 E 가일어났을때, 사건 A 의조건부확률은 P(A E)= P(AcupE) P(E) P(AcupE) = P(AcupE)+P(A^CcupE) 205 기계 A 에서생산된제품을택하는사건을 A, 기계 B 에서생산된제품을택하는사건을 B, 불량품인사건을 E 라하면 P(AcupE)=P(A)P(E A)=0.4\0.01=0.004 P(BcupE)=P(B)P(E B)=0.6\0.02=0.012.t3 P(E)=P(AcupE)+P(BcupE)=0.016 따라서구하는확률은 P(B E)= P(BcupE) =up0.012 ~ 0.016`= 3/4 P(E) 206 ⑴ r1par 갑이가위를내서이길확률은 0.3\0.4=0.12 r2par 갑이바위를내서이길확률은 0.4\0.2=0.08 r3par 갑이보를내서이길확률은 0.3\0.4=0.12 이상에서갑이이길확률은 =0.32~ ⑵ 갑이이기는사건을 A, 갑이가위를내는사건을 B라하면구하는확률은 P(B A)= P(AcupB) =up0.12 ~0.32`= 3/8 P(A) 갑이이길확률구하기 50% 갑이이겼을때, 가위를내서이겼을확률구하기 50% 207 흰공을꺼내는사건을 A, 검은공을꺼내는사건을 B, 검은공이라고대답하는사건을 E 라하면 P(A)=2/5, P(B)=3/5, P(E A)=0.4, P(E B)=0.6 따라서구하는확률은 P(B E)= P(BcupE) P(BcupE) = P(E) P(AcupE)+P(BcupE) P(B)P(E B) = P(A)P(E A)+P(B)P(E B) 3/5 \ 0.6 = = 9/1 3 2/5 \ /5 \ 내일비가오는사건을 A, 내일경기에서이기는사건을 E 라하면내일비가오지않는사건은 A^C 이므로 P(A)=0.4, P(A^C)=0.6 P(E A)=0.7, P(E A^C)=0.5 따라서내일경기에서이길확률은 P(E)=P(AcupE)+P(A^CcupE) =P(A)P(E A)+P(A^C)P(E A^C) =0.4\ \0.5 =0.58 = 2 9/5 0 이므로 a=50, b=29.t3 a+b= 두사건 A, B 가서로독립이므로 P(AcupB)=P(A)P(B)=1/3&P(B) P(A^CcupB^C)=P((AhapB)^C)=1-P(AhapB)=1/4 이므로 P(AhapB)=3/4 P(AhapB)=P(A)+P(B)-P(AcupB) 이므로 3/4 = 1/3 + P(B)-1/3&P(B).t3 P(B)=5/8 다른두사건 A, B가서로독립이면두사건 A^C, B^C도서로독립이므로 P(A^CcupB^C)=P(A^C)P(B^C)={1-P(A)}{1-P(B)} 1-P(B)=3/8 =2/3&{1-P(B)}=1/4.t3 P(B)=5/8 210 {1, 2, 4, 6} 이일어나는사건을 A 라하면 P(A)=4/6=2/3 1 {1, 3, 5} 가일어나는사건을 B 라하면 P(B)=3/6=1/2, P(AcupB)=1/6.t3 P(AcupB)not=P(A)P(B) 2 {1, 4, 5} 가일어나는사건을 B라하면 P(B)=3/6=1/2, P(AcupB)=2/6=1/3.t3 P(AcupB)=P(A)P(B) 3 {2, 4, 6} 이일어나는사건을 B라하면 P(B)=3/6=1/2, P(AcupB)=3/6=1/2.t3 P(AcupB)not=P(A)P(B) 4 {1, 2, 3, 4, 5} 가일어나는사건을 B라하면 P(B)=5/6, P(AcupB)=3/6=1/2.t3 P(AcupB)not=P(A)P(B) 5 {1, 3, 4, 5, 6} 이일어나는사건을 B라하면 30 바른답 알찬 해 (028~056) 일등확통 -04~06 강 ok.indd 오후 3:33

31 P(B)=5/6, P(AcupB)=3/6=1/2.t3 P(AcupB)not=P(A)P(B) 따라서 {1, 2, 4, 6} 과서로독립인사건은 2이다. 1등급비법두사건 A, B가독립인지확인하려면 P(A), P(B), P(AcupB) 를각각구한후, P(AcupB)=P(A)P(B) 가성립하는지조사하면된다. 이때 P(AcupB)=P(A)P(B) 이면서로독립이고, P(AcupB)not=P(A)P(B) 이면서로종속이다. 211 A={2, 4, 6}, B={1, 2, 3}, C={3, 4} 이므로 P(A)=1/2, P(B)=1/2, P(C)=1/3 P(AcupB)=1/6, P(BcupC)=1/6, P(CcupA)=1/6 ㄱ. P(A)P(B)not=P(AcupB) 이므로두사건 A와 B는서로종속이다. ㄴ. P(B)P(C)=P(BcupC) 이므로두사건 B와 C는서로독립이다. ㄷ. P(C)P(A)=P(CcupA) 이므로두사건 C와 A는서로독립이다. 이상에서서로독립인것은ㄴ, ㄷ이다. a 의값구하기 40% b 의값구하기 40% a+b 의값구하기 20% 214 ㄱ. A 와 B 가서로배반사건이면 AcupB= 따라서 P(AcupB^C)=P(A) 이므로 P(B^C A)= P(AcupB^C) = P(A) P(A) P(A) =1 ㄴ. A와 B^C가서로독립이면 A와 B도서로독립이므로 P(AcupB)=P(A)P(B) ㄷ. A와 B가서로독립이면 A와 B^C, A^C와 B, A^C와 B^C 모두서로독립이므로 P(A^C B^C)= P(A^CcupB^C) P(B^C) = P(A^C)P(B^C) =P(A^C) P(B^C) 1-P(A^C B)=1-P(A^C)=P(A).t3 P(A^C B^C)not=1-P(A^C B) 이상에서옳은것은ㄱ, ㄴ이다. 참고 A, B가서로독립이면 A와 B^C, A^C와 B, A^C와 B^C 도서로독립이다. 04 조건부확률 212 3개의동전을동시에던져서나오는면을순서쌍으로나타내면 A= {( 앞, 뒤, 뒤 ), ( 뒤, 앞, 뒤 ), ( 뒤, 뒤, 앞 ), ( 뒤, 뒤, 뒤 )} B={( 앞, 앞, 앞 ), ( 뒤, 뒤, 뒤 )} ㄱ. P(A)=#4/@2^3 $=1/2 ㄴ. P(AcupB)=#1/@2^3 $=1/8 ㄷ. P(A)=1/2, P(B)=#2/@2^3 $=1/4, P(AcupB)=1/8 이므로 P(A)P(B)=P(AcupB) 따라서두사건 A와 B는서로독립이다. 이상에서ㄱ, ㄴ, ㄷ모두옳다. 213 A, B가서로배반사건이면 P(AcupB)=0 P(AhapB)=P(A)+P(B)-P(AcupB) 에서 3/4 = P(A)+2/3.t3 P(A)=1/12 배반사건과독립사건의관계는다음과같다. P(A)>0, P(B)>0인두사건 A, B에대하여 1 A, B가서로배반이면 A, B는서로종속이다. 2 A, B가서로독립이면 A, B는서로배반이아니다. 215 ㄱ. A_2={2, 4, 6}, A_3={3, 6}, A_2cupA_3={6} 이므로 P(A_2)=3/6=1/2, P(A_3)=2/6=1/3, P(A_2cupA_3)=1/6 따라서 P(A_2cupA_3)=P(A_2)P(A_3) 이므로 A_2 와 A_3 은 서로독립이다. ㄴ. A_2={2, 4, 6}, A_4={4}, A_2cupA_4={4} 이므로 P(A_4 A_2)= P(A_4cupA_2) P(A_2) 1/6 = = 1/3 1/2 ㄷ. A_2={2, 4, 6}, A_5={5} 이므로 A_2cupA_5= 따라서 A_2 와 A_5 는서로배반사건이다. 이상에서ㄱ, ㄴ, ㄷ모두옳다..t3 a = 1/1 2 A, B가서로독립사건이면 P(AcupB)=P(A)P(B)=2/3&P(A) P(AhapB)=P(A)+P(B)-P(AcupB) 에서 3/4 = P(A)+2/3-2/3&P(A).t3 P(A)=1/4.t3 b = 1/4.t3 a+b=1/12+1/4=1/3 216 자유투를한번던져성공할확률이 2/3 이므로실패할확률은 1/3 이다. 따라서 5번의자유투를던져 3번성공할확률은 _5C_3&(&2/3&)^^3&(&1/3&)^^2= 10 2^3 = 8 0/24 3 3^5 217 한개의주사위를한번던질때, 3 이상의눈이나올확률은 4/6 = 2/3 r1par 3 이상의눈이 3 번나올확률은 04. 조건부확률 31 해 (028~056) 일등확통 -04~06 강 ok.indd 오후 3:33

32 바른답 알찬 _4C_3&(&2/3&)^^3(&1/3&)^^1=32/81 r2par 3 이상의눈이 4 번나올확률은 _4C_4&(&2/3&)^^4=16/81 r1par, r2par 에서구하는확률은 3 2/ /8 1 = 1 6/ 불량품이두개또는세개포함될확률은 _3C_2(0.2)^2(0.8)^1&+_3C_3(0.2)^3=3 (&1/5&)^^2(&4/5&)^^1&+(&1/5&)^^3 =12/125+1/125=13/ a+b의값이 6이되는경우의확률은다음과같다. r1par a=3, b=3인경우 _4C_3&(&1/3&)^^3&(&2/3&)^^1 _3C_3&(&1/3&)^^3=#8/@3^7 $ r2par a=4, b=2 인경우 _4C_4&(&1/3&)^^4& _3C_2&(&1/3&)^^2&(&2/3&)^^1=#6/@3^7 $ r1par, r2par 에서구하는확률은 #8/@3^7 $+#6/@3^7 $=#14/@3^7 $ \0.8=8( 명 ) 이므로 8명이상참석해야동호회행사를진행할수있다. r1par 8명이참석할확률은 _10C_8(&1/2&)^^8(&1/2&)^^2= 45 2^1^0 r2par 9명이참석할확률은 _10C_9(&1/2&)^^9(&1/2&)^^1= 10 2^1^0 r3par 10명이참석할확률은 _10C_10(&1/2&)^^1^^0= 1 2^1^0 r1par, r2par, r3par 에서동호회모임이진행될확률은 45 2^1^ ^1^ ^1^0 = 56 2^1^0 = 7 2^7.t3 n=7 승부가결정되는경우알기 20% A 선수가우승할확률구하기 30% B 선수가우승할확률구하기 30% 승부가결정될확률구하기 20% 222 한개의주사위를 5 번던져 A 지점에있던바둑돌이 B 지점에있으려면오른쪽으로 2 칸, 왼쪽으로 1 칸, 위쪽으로 2 칸이동 하면된다. 1 또는 2 의눈이나올확률은 1/3, 3 의눈이나올확률은 1/6, 4 이상의눈이나올확률은 1/2 이므로구하는확률은 low2!2! 5!```(&1/3&)^^2&(&1/6&)^^1&(&1/2&)^^2=5/36 1 회의시행에서사건 A 가일어날확률을 a, 사건 B 가일어날확률 을 b, 사건 C 가일어날확률을 c 라할때, 이시행을독립적으로 n 회반복하는시행에서 A 가 p 회, B 가 q 회, C 가 r 회일어날확률은 lowp!q!r! ~~~n! `a^p&b&^q&c^r ( 단, a+b+c=1, p+q+r=n) 223 P(4) 는 10 회시행중흰구슬이 4 번나올확률이므로 P(4)=_10C_4&(&3/5&)^^4(&2/5&)^^6 P(6) 는 10 회시행중흰구슬이 6 번나올확률이므로 P(6)=_10C_6&(&3/5&)^^6(&2/5&)^^4 P(6) _10&C_6&^(&3/5&^)^^6^(&2/5&^)^^4 ^(&3/5&^)^^2 P(4) = = = 9/4 _10&C_4&^(&3/5&^)^^4^(&2/5&^)^^6 ^(&2/5&^)^^2 221 A 선수가이길확률이 3/5 이므로 B 선수가이길확률은 1 등급문제 pp. 64~65 1-3/5 = 2/5 다섯번째경기에서승부가결정되려면우승하는선수는네 번째경기까지 3 번이기고 1 번진후, 다섯번째경기에서이 겨야한다. r1par A 선수가우승할확률은 _4C_3&(&3/5&)^^3(&2/5&)^^1 3/5= 648 5^5 r2par B 선수가우승할확률은 _4C_3&(&2/5&)^^3(&3/5&)^^1& 2/5= 192 5^5 r1par, r2par 에서구하는확률은 648 5^ ^5 = 840 5^5 = A, C, B / ⑴ 4/9 ⑵ 4/9 ⑶ 서로독립이아니다 / 조건부확률전략 A, B, C집에서우산을분실한사건에대한조건부확률을각각구하여크기를비교한다. A, B, C집에가는사건을각각 A, B, C라하고, 우산을분실하는사건을 E라하면 P(A E)= P(AcupE) P(E) 1/4 = P(E) 32 바른답 알찬 해 (028~056) 일등확통 -04~06 강 ok.indd 오후 3:33

33 P(B E)= P(BcupE) P(E) 3/4 1/5 3/2 0 = P(E) = P(E) P(C E)= P(CcupE) 3/4 4/5 2/5 6/2 5 = P(E) P(E) = P(E) P(A E)>P(C E)>P(B E) 이므로 A, C, B 의순서로 가는것이합리적이다. 참고 우산을분실할확률은 P(E)=P(AcupE)+P(BcupE)+P(CcupE) =1/4+3/20+6/25=16/25 A, B, C 집순으로방문하여우산을분실하였므로 B 집에서우산 을분실했다면 A 집에서는우산을분실하지않은것이다. 따라서 P(BcupE) 를구할때 A 집에서우산을분실하지않을확 률 3/4 을반드시곱해야한다. 같은방법으로 C 집에서우산을분실했다면 A, B 집에서는우산을 분실하지않은것이므로 P(CcupE) 를구할때 3/4 4/5 를반드시곱 해야한다. 따라서구하는확률은 P(A E)= P(AcupE) P(E) P(AcupE) = P(AcupE)+P(BcupE)+P(CcupE) 5/2 7 = = 5/1 9 5/ / /3 227 확률의곱셈정리전략세번째검사에서검사가끝나려면두번째검사까지불량품이 1개만나와야함을이용한다. 두번째검사에서검사가끝나려면불량품 2개를모두꺼내야하므로 p_1=2/10 1/9=1/45 세번째검사에서검사가끝나려면두번째검사까지불량품 이 1 개, 정상품이 1 개나오고세번째검사에서불량품이나 와야하므로 p_2=(&2/10 8/9+8/10 2/9&) 1/8=2/45 04 조건부확률 225 확률의곱셈정리와조건부확률 전략주어진세사건을 A, B, E 로놓고, P(E)=P(AcupE)+P(BcupE) 임을이용한다. 상자 A, B 를택하는사건을각각 A, B, 꺼낸공이 검은공인사건을 E 라하면구하는확률은 P(B E)= P(BcupE) P(E) P(BcupE) = P(AcupE)+P(BcupE) P(B)P(B E) = P(A)P(E A)+P(B)P(E B) 1/2 1/3 = = 2/5 1/2 2/4 + 1/2 1/3 226 확률의곱셈정리와조건부확률 전략 주사위 A, B, C 에서같은수가적힌면이나오는사건에대한 조건부확률을각각구한다. 주사위 A, B, C 를선택하는사건을각각 A, B, C 라 하고, 주사위를두번던졌을때두번모두같은수가나오 는사건을 E 라하면 P(AcupE)=P(A)P(E A) =1/3 (&1/3 1/3+2/3 2/3&)=5/27 P(BcupE)=P(B)P(E B) =1/3 (&1/3 1/3+2/3 2/3&)=5/27 P(CcupE)=P(C)P(E C) =1/3 (1 1)=1/3.t3 p_1&+p_2=1/45+2/45=1/15 1등급비법제품을 1개씩꺼낼때두번째불량품이나오는순간검사가끝나므로 n번째검사에서검사를끝내기위해서는 (n-1) 번째검사까지는불량품이 1개만나오고 n번째검사에서두번째불량품이나오면된다. 228 독립과종속전략뽑은두개의공에적힌수가모두짝수이거나모두홀수이면두수의합이짝수가된다. ⑴ r1par 두개의공에적힌수가모두짝수일확률은 _5&C_2 ~ _10&C_2 = 2/9 r2par 두개의공에적힌수가모두홀수일확률은 _5&C_2 ~ _10&C_2 = 2/9 r1par, r2par 에서 P(A)=2/9+2/9=4/9 ⑵ r1par 모두흰공을뽑을확률은 _5&C_2 ~ _10&C_2 = 2/9 r2par 모두검은공을뽑을확률은 _5&C_2 ~ _10&C_2 = 2/9 r1par, r2par 에서 P(B)=2/9+2/9=4/9 ⑶ r1par 두개의공에적힌수가모두짝수이면서모두흰공일확률은 04. 조건부확률 33 해 (028~056) 일등확통 -04~06 강 ok.indd 오후 3:33

34 바른답 알찬 _2&C_2 ~ _10&C_2 = 1/4 5 r2par 두개의공에적힌수가모두짝수이면서모두검은공일확률은 _3&C_2 ~ _10&C_2 = 1/1 5 r3par 두개의공에적힌수가모두홀수이면서모두흰공일확률은 _3&C_2 ~ _10&C_2 = 1/1 5 r4par 두개의공에적힌수가모두홀수이면서모두검은공일확률은 _2&C_2 ~ _10&C_2 = 1/4 5 이상에서 P(AcupB)=1/45+1/15+1/15+1/45=8/45 그런데 P(A)P(B)=4/9 4/9=16/81 이므로 P(A)P(B)not=P(AcupB) 따라서 A, B는서로독립이아니다. P(A) 의값구하기 30% P(B) 의값구하기 30% A, B가서로독립인지판별하기 40% 또, P(AhapB)=P(A)+P(B)-P(AcupB) 이므로 5/6=(&x+1/3&)+(y&+1/3&)-1/3.t3 x+y=1/2 ᄂ을ᄀ에대입하면.c3.c3 ᄂ xy+1/3 1/2+1/9=1/3.t3 xy=1/18.c3.c3 ᄃ 2단계 1단계에서세운두식을연립하여풀어 P(AcupB^C), P(A^CcupB) 의값을구한다. ᄂ, ᄃ을연립하면 x(&1/2-x&)&=1/18, x^2&-1/2&x+1/18=0 18x^2&-9x+1=0, (6x-1)(3x-1)=0.t3 x = 1/6 또는 x = 1/3 x = 1/6 ^^{~ y = 1/3 x = 1/3 또는 ^^{~ y = 1/6 그런데 P(A)<P(B) 이므로 x = 1/6, y=1/3& 즉, P(AcupB^C)=1/6, P(A^CcupB)=1/3 3 단계조건부확률을이용하여 P(B^C A) 의값을구한다. P(B^C A)= P(AcupB^C) P(A) 1/6 1/6 = = = 1/3 1/3 + 1/6 1/2 229 독립시행의확률전략 B팀이 7차전에서우승하려면 6번째경기까지 3승 3패가되고 7 번째경기에서 B팀이승리해야함을이용한다. 첫번째경기에서 A팀이승리하였으므로 2번째경기부터 6번째경기까지 5번의경기중에서 B팀이 3번승리하고 7번째경기에서 B팀이승리하면된다. 따라서구하는확률은 _5C_3&(&1/2&)^^3(&1/2&)^^2& 1/2=5/ 조건부확률 + 독립과종속 1단계 P(AcupB^C)=x, P(A^CcupB)=y로놓고두사건 A, B가서로독립임을이용하여 x, y에대한식을세운다. 두사건 A, B가서로독립이므로 P(AcupB)=P(A)P(B) P(AcupB^C)=x, P(A^CcupB)=y라하면 P(A)=P(AcupB)+P(AcupB^C)=1/3+x, P(B)=P(AcupB)+P(A^CcupB)=1/3+y 이므로 P(AcupB)=P(A)P(B) 에서 (&x+1/3&)(y&+1/3&)=1/3 231 독립시행의확률 1단계말이 4행 5열로이동하려면각방향으로몇칸씩이동해야하는지구한다. 말이 1행 1열에서 4행 5열로이동하려면오른쪽으로 4칸, 아래로 3칸, 총 7칸을이동해야하고, 5번의가위바위보로 7칸을이동하기위해서는 [ 규칙 3] 에의해대각선으로 2칸, [ 규칙 1] 에의해오른쪽으로 2칸, [ 규칙 2] 에의해아래로 1칸을이동해야한다. 2단계대각선으로 2칸, 오른쪽으로 2칸, 아래로 1칸이동할확률을구한다. 비기면오른쪽으로 1칸, 아래로 1칸, 즉대각선으로 1칸이동하므로대각선으로 2칸이동할확률은 (&1/3&)^^2=1/9 갑이이기면오른쪽으로 1칸이동하므로오른쪽으로 2칸이동할확률은 (&1/3&)^^2=1/9 을이이기면아래로 1칸이동하므로아래로 1칸이동할확률은 (&1/3&)^^1=1/3 이때 ( 대각선, 대각선, 오른쪽, 오른쪽, 아래 ) 를일렬로나열하는방법의수는.t3 xy+1/3(x+y)+1/9=1/3.c3.c3 ᄀ low2!2! 5! `=30 34 바른답 알찬 해 (028~056) 일등확통 -04~06 강 ok.indd 오후 3:33

35 따라서구하는확률은 1/9 1/9 1/3 30=10/81 P(AcupB^C)=P(A)-P(AcupB) =P(A)-1/5=1/4 이므로 P(A)=9/20.t3 P(A^C)=1-P(A)=1-9/20=11/20 실전대비평가문제 II. 확률 pp. 66~ / / 수학적확률전략직선 y=b/a&x가선분 AB와만날조건을구한다. 서로다른두개의주사위를동시에던질때, 모든경우의수는 6 6=36 직선 y=b/a&x에서 y좌표가 1일때, x좌표는 a/b 이므로직선 y=b/a&x와선분 AB가만나는경우는 2-<a/b-<3일때이다. 이때 2-<a/b-<3을만족시키는순서쌍 (a, b) 는 (2, 1), (3, 1), (4, 2), (5, 2), (6, 2), (6, 3) 의 6가지이므로구하는확률은 6/3 6 = 1/6 233 수학적확률전략네명이가위바위보를하는모든경우의수는중복순열이다. 4명이가위바위보를하는모든경우의수는가위, 바위, 보 3개중에서 4개를택하는중복순열의수와같으므로 _3PAI_4=3^4=81 4명중이기는 2명을선택하는경우의수는 _4C_2=6이고, 이 2명이이기는경우는 ( 가위, 가위, 보, 보 ), ( 바위, 바위, 가위, 가위 ), ( 보, 보, 바위, 바위 ) 의 3가지이므로 4명중 2명이이기는경우의수는 6 3=18 따라서구하는확률은 1 8/8 = 2/9 234 여사건의확률전략 P(A^ChapB^C)=P((AcupB)^C) 과 P(A^C)=1-P(A) 임을이용한다. P(A^ChapB^C)=P((AcupB)^C) =1-P(AcupB)=4/5 이므로 P(AcupB)=1/5 235 여사건의확률 전략 첫째날과여섯째날에모두여학생이봉사활동을하게될확 률을구하여여사건의확률을이용한다. 전체경우의수는 6! 첫째날과여섯째날에남학생이봉사활동을하지않는경우 의수는여학생 4 명중 2 명이첫째날과여섯째날에봉사활 동을하고나머지 4 명의학생이둘째날부터다섯째날까지 하루씩봉사활동을하는경우의수와같으므로 _4P_2 4! 따라서남학생이첫째날과여섯째날에봉사활동을하지않 을확률은 _4&P_2& 4! 6! = 4 3 4! = 2/5 6! 이므로구하는확률은 1-2/5 = 3/5 236 조건부확률 전략 주어진조건을표로나타내고조건부확률을이용한다. 주어진조건을표로나타내면다음과같다. 지각한학생 지각하지않은학생 버스로등교 3/5 1/2 0 3/5 1 9/2 0 걸어서등교 2/5 1/1 5 2/5 1 4/1 5.t3 ( 구하는확률 ) ( 버스로등교한학생중지각한학생의비율 ) = ( 지각한학생의비율 ) 3/5 1/2 0 & = = 9/1 7 3/5 1/20+2/5 1/ 확률의곱셈정리전략 1부터 10까지의자연수중소수는 2, 3, 5, 7의 4개이므로 4번째까지소수를 3개뽑고 5번째에나머지소수 1개를뽑아야함을이용한다. 1부터 10까지의자연수중소수는 2, 3, 5, 7의 4개이므로 4번째까지소수를 3개뽑고 5번째에나머지소수 1개를뽑으면시행이멈춘다. 4번째까지소수를 3개뽑는경우의수는 ( 소수, 소수, 소수, 소수가아닌수 ) 를일렬로나열하는방법의수와같으므로 up4! 3!`=4 실전대비 Ⅱ 단원평가문제 35 해 (028~056) 일등확통 -04~06 강 ok.indd 오후 3:33

36 바른답 알찬 또, 그각각의확률은 4/10 3/9 2/8 6/7=1/35 따라서구하는확률은 III 통계 (4 1/35) 1/6=2/105 다른 (_4C_3 _6C_1 4!) _1C_1 =2/10 5 _10P_5 05 확률분포 238 독립과종속전략두사건 A, B가서로독립이면 P(AcupB)=P(A)P(B) 임을이용한다. TV 프로그램 A, B를시청하는사람들중에서임의로한명을뽑을때, 그사람이 A 프로그램을시청하는사건을 A, 20세미만인사건을 E라하면 P(A)=250/400=5/8, P(E)=160/400=2/5& P(AcupE)=a/40 이때두사건 A, E가서로독립이므로 P(AcupE)=P(A)P(E), 즉 교과서에서뽑은기본문제 240 기댓값 : 1, 분산 : 1/2 241 ⑴ 참조 ⑵ 7/2 ⑶ 3 5/ ⑴ 평균 : 17, 분산 : 18 ⑵ 평균 : -9, 분산 : ⑴ 20 ⑵ 5 0/3 ⑶ 5rt6~ ⑴ ⑵ /2 pp. 70~ 확률변수 X 의기댓값과분산을각각 E(X), V(X) 라하면 E(X)=0 1/ /2+2\1/4=1 V(X)=E(X^2)-{E(X)}^2=0^2 1/4+1^2&\1/2+2^2 1/4-1^2=1/2 a/40 = 5/8 2/5 = 1/4 따라서 a=100, b=60, c=150, d=90 이므로 a b/c d = up `= 4/9 241 ⑴ X 합계 P(X=x) 1/6 1/6 1/6 1/6 1/6 1/6 1 1등급비법두사건 A, B가서로독립이다. NLO P(AcupB)=P(A)P(B) NLO P(B)=P(B A)=P(B A^C) NLO A와 B^C, A^C와 B, A^C와 B^C도서로독립이다. ⑵ E(X)=1 1/ / / / / /6 = 7/2 ⑶ V(X)=E(X^2)-{E(X)}^2 E(X)=1^2 1/6+2^2 1/6+3^2 1/6+4^2 1/6+5^2 1/6 239 독립시행의확률전략 a_i~(1-<i-<6) 의값은 0 또는 1이므로 S_3=2이고 S_6=3이되는경우는동전을 6번던지는시행에서 3번째까지앞면이 2번나오고, 4번째부터 6번째까지앞면이 1번나오는경우임을이용한다. 동전의앞면이나올확률은 1/2 S_3=2이고 S_6=3이되는경우는동전을 6번던지는시행에서 3번째까지앞면이 2번나오고, 4번째부터 6번째까지앞면이 1번나오는경우이므로구하는확률은 _3C_2(1/2)^^2(1/2)^^1 _3&C_1(1/2)^^1(1/2)^^2 =3/8 3/8=9/64 +6^2 1/6-(7/2)^^2 = 3 5/ ⑴ E(3X-1)=3E(X)-1=3 6-1=17 V(3X-1)=3^2~&V(X)=9 2=18 ⑵ E(-2X+3)=-2E(X)+3=-2 6+3=-9 V(-2X+3)=(-2)^2&~V(X)=4 2=8 243 ⑴ E(X)=120 1/6=20 ⑵ V(X)=120 1/6 5/6=100/6=50/3 ⑶ sigma(x)=2v(x)x~=rt50/3~~= 5rt6~ 3 ~ 244 f(x)=k 의그래프와 x 축및두직선 x=0, x=2 로둘러싸인직사각형의 넓이가 1 이므로 f{x} k f{x}=k 2 k=1.t3 k = 1/2 O 2 x 36 바른답 알찬 해 (028~056) 일등확통 -04~06 강 ok.indd 오후 3:33

37 245 Z= X-10 ~ 으로놓으면확률변수 Z는표준정규분포 2 N(0, 1) 을따른다. ⑴ P(8 X 12)=P( =P(-1 Z 1) =2 P(0 Z 1) Z ) 2 = = ⑵ P(X->15)=P(Z-> ~) 2 =P(Z->2.5) =0.5-P(0 Z 2.5) = = 확률변수 X 가가지는값은 1, 2, 3 이고, 그확률은각각 P(X=1)= _4C_2 _3C_2 = 6/1 0, P(X=2)= _5C_3 _5C_3 = 3/1 0, P(X=3)= _2C_2 _5C_3 = 1/1 0.t3 P(X->2)=P(X=2)+P(X=3) =3/10+1/10=2/5 249 확률의총합은 1이므로 P(X=2)+P(X=3)+P(X=4)+.c3+P(X=9) = k k k c3+ k 9 8 =kñ1-1/2)+k(1/2-1/3)+k(1/3-1/4)+.c3+k(1/8-1/9) 05 확률분포 =kñ1-1/9)=8/9&k=1 기출문제 pp. 72~ / / / ⑴ 참조 ⑵ 3/2 ⑶ 9/2 0 ⑷ / / ⑴ 550 ⑵ 174.2`cm 확률의총합은 1 이므로 Ña+1/2)+a+1/4=1 2a=1/4.t3 a = 1/8.t3 P(X^2=1)=P(X=-1 또는 X=1) =P(X=-1)+P(X=1) = 5/8 + 1/4 = 7/8 247 확률변수 X 가가지는값은 0, 10, 100, 110 이므로 a=110 이고, X=10 이면 10 원짜리동전은앞면이나오고 100 원짜 리동전은뒷면이나오므로 P(X=10)=1/2\1/2=1/4.t3 b = 1/4 X=100 이면 10 원짜리동전은뒷면이나오고 100 원짜리동 전은앞면이나오므로 P(X=100)=1/2\1/2=1/4.t3 c = 1/4.t3 abc=110\1/4\1/4=55/8.t3 k = 9/8 참고 1 AB = 1 (;1/A :-;1/B :) ( 단, Anot=B) B-A 250 확률변수 X가가지는값은 1, 2, 3, 6이고, 그확률은각각 P(X=1)= 7-k 7-2k, P(X=2)= 16 16, P(X=3)= 7-3k 7-6k, P(X=6)= 확률의총합은 1이므로 7-k k k k 16 = k =1, 28-12k=16 16.t3 k=1 또, X^2&-2X-15<0에서 (X+3)(X-5)<0.t3-3<X<5.t3 P(X^2&-2X-15<0) =P(-3<X<5) =P(X=1)+P(X=2)+P(X=3) =1-P(X=6) =1-1/16=15/16 k 의값구하기 40% 확률변수 X 의값의범위구하기 30% 확률구하기 30% 확률변수 X의확률질량함수 P(X=x_i)=p_i (i=1, 2, 3,.c3, n) 에대하여 p_i 의일부를모르거나함수식에미정계수가있을때는 sigi = ^n`~p_i=p_1&+p_2&+p_3&+.c3+p_n=1 1 임을이용한다. 251 확률의총합은 1이므로 0.2+a+0.3=1.t3 a= 확률분포 37 해 (028~056) 일등확통 -04~06 강 ok.indd 오후 3:33

38 바른답 알찬 또, m=sigi= ^3``~ 1~x_i&p_i 이므로 m=1\0.2+2\0.5+3\0.3=2.1.t3 a+m= = 확률의총합은 1 이므로 a+1/4+b=1.t3 P(X-<2)=P(X=1)+P(X=2) = 1/8 + 3/4 = 7/8 255 확률변수 X가가지는값은 1000, 5000, 10000이므로 그확률은각각.t3 a=3/4-b 또, E(X)=5 이므로.c3.c3 ᄀ P(X=1000)=1/3\1/3\1/3=1/27 P(X=5000)=(1/3\1/3\2/3)\3+(1/3\2/3\2/3)\3 1\a+3\1/4+7\b=5.c3.c3 ᄂᄀ을ᄂ에대입하면 3/4-b+3/4+7b=5.t3 b = 7/ 확률변수 X가가지는값은 0, 1, 2이고, 그확률은각각 P(X=0)=2/5\1/4=1/10 = 2/3 P(X=10000)=2/3\2/3\2/3=8/27 확률변수 X의확률분포를표로나타내면다음과같다. X 합계 P(X=x) 1/2 7 2/3 8/2 7 1 P(X=1)=3/5\2/4+2/5\3/4=3/5 P(X=2)=3/5\2/4=3/10 확률변수 X 의확률분포를표로나타내면다음과같다..t3 E(X)=1000\1/ \2/ \8/27 = X 합계 P(X=x) 1/1 0 3/5 3/1 0 1.t3 E(X)=0\1/10+1\3/5+2\3/10=6/5 1등급비법먼저확률변수 X가가지는값에대하여그각각의확률을구한후 X의확률분포를표로나타낸다. 이때 E(X)=sigi=1 ^n`~x_i&p_i, V(X)=E(X^2)-{E(X)}^2, sigma(x)=1v(x)z 임을이용한다. 254 확률의총합은 1이므로 a+b+c=1.c3.c3 ᄀ E(X)=2이므로 1\a+2\b+3\c=2.t3 a+2b+3c=2.c3.c3 ᄂ sigma(x)=1/2 이므로 V(X)=(1/2)^^2=1/4 V(X)=E(X^2)-{E(X)}^2 에서 E(X^2)=V(X)+{E(X)}^2=1/4+4=17/4 이므로 1^2&\a+2^2&\b+3^2&\c=17/4.t3 a+4b+9c=17/4 ᄀ, ᄂ, ᄃ을연립하여풀면 a = 1/8, b = 3/4, c = 1/8.c3.c3 ᄃ 확률변수 X가가지는값구하기 30% X의값에따라각각의확률을구하여표로나타내기 40% X의기댓값구하기 30% 256 E(X)=1\3/10+2\1/2+3\1/5=19/10 이므로 E(10X)=10E(X)=10\19/10= Y= X+a 이므로 b E(Y)=E^( X+a ^)=1/b &E(X)+a/b b =3/b + a/b = 4.t3 3+a=4b.c3.c3 ᄀ V(Y)=V^( X+a ^)=#1/@b^2 $V(X) b = 16 b^2 =4 b^2=4.t3 b=2 (.T3 b>0) b=2를ᄀ에대입하면 3+a=8.t3 a=5.t3 a+b=7 258 E(Y)=4, E(Y^2)=28이므로 V(Y) =E(Y^2)-{E(Y)}^2=28-4^2=12 Y=1/2&X+5 이므로 38 바른답 알찬 해 (028~056) 일등확통 -04~06 강 ok.indd 오후 3:33

39 E(Y)=E(1/2&X+5Ò= 1/2 E(X)+5=4.t3 E(X)=-2 X 합계 P(X=x) 1/6 2/3 1/6 1 V(Y) =V(1/2&X+5Ò= 1/4 & V(X)=12.t3 V(X)=48.t3 E(X)+V(X)=-2+48= E(X)=a, E(X^2)=2a+3이므로 V(X) =E(X^2)-{E(X)}^2=-a^2&+2a+3 sigma(2x)=2sigma(x)=22-a^2&+2a+x3x =22-(ax-1x)^2&+4x 따라서 2X의표준편차는 a=1일때최댓값 4를갖는다. E(X)=0\1/6+1\2/3+2\1/6=1 V(X)=E(X^2)-{E(X)}^2 =0^2&\1/6+1^2&\2/3+2^2&\1/6-1^2 =4/3-1=1/3 sigma(x)=;#!rt3/3 $ :.t3 sigma(6x-8)=6sigma(x)=6\;#!rt3/3 $ :=2rt3 05 확률분포 260 ⑴ 확률변수 X 가가지는값은 0, 1, 2, 3 이고, 그확률은각각 P(X=0)= _3&C0&\_3&C_3 = 1/2 0 _6C_3 P(X=1)= _3&C_1&\_3&C_2 = 9/2 0 _6C_3 P(X=2)= _3&C_2&\_3&C_1 = 9/2 0 _6C_3 P(X=3)= _3&C_3&\_3&C0 = 1/2 0 _6C_3 확률변수 X 의확률분포를표로나타내면다음과같다. X 합계 P(X=x) 1/2 0 9/2 0 9/2 0 1/ E(X)=10, V(X)=8이므로 np=10.c3.c3 ᄀ np(1-p)=8.c3.c3 ᄂᄀ을ᄂ에대입하면 10(1-p)=8.t3 p = 1/5 p = 1/5 을ᄀ에대입하면 1/5&n=10.t3 n= 확률변수 X가이항분포 BÑn, 1/3 ) 을따르므로 ⑵ E(X)=0\1/20+1\9/20+2\9/20+3\1/20 = 3/2 ⑶ V(X)=E(X^2)-{E(X)}^2 =0^2&\1/20+1^2&\9/20+2^2&\9/20+3^2&\1/20-(3/2)^^2 =27/10-9/4=9/20 E(X)=n\1/3=n/3 E(2X+5)=13이므로 2E(X)+5=13 2\n/3&+5=13.t3 n= 서로다른두개의동전을동시에던질때모두앞면이나올확률은 1/4 이므로확률변수 X는이항분포 BÑ64, 1/4 ) 을따른다. ⑷ V(10X+3)=100V(X)=100\9/20=45.t3 E(X)=64\1/4=16, V(X)=64\1/4\3/4=12.t3 E(X)+V(X)=16+12=28 X 의확률분포를표로나타내기 30% X 의평균구하기 20% X 의분산구하기 30% 10X+3 의분산구하기 20% 261 확률변수 X 가가지는값은 0, 1, 2 이고, 그확률은각각 P(X=0)= 2!&\2!& = 1/6 4! P(X=1)= 2!\2!\&_2&C_1&\2! = 2/3 4! P(X=2)= 2!&\2! = 1/6 4! 확률변수 X 의확률분포를표로나타내면다음과같다. 265 _1_6&C_x(1/8)^^x(7/8)^^1^^6-^^x은한번의시행에서일어날확률이 1/8 인어떤사건이 16번의독립시행에서 x번일어날확률이다. 따라서이사건이일어나는횟수를확률변수 X라하면 X는 이항분포 BÑ16, 1/8 ) 을따르므로 E(X)=16\1/8=2, V(X)=16\1/8\7/8=7/4.t3 ^16``k^2&_1_6&C_k(1/8)^^k(7/8)^^1^^6-^^k=sigk=0 ^16``k^2&~P(X=k) =E(X^2)=V(X)+{E(X)}^2 =7/4&+2^2=23/4 05. 확률분포 39 해 (028~056) 일등확통 -04~06 강 ok.indd 오후 3:33

40 바른답 알찬 확률변수 X 의확률이독립시행의확률로나타내어지면 X 는이 항분포를따른다. 270 확률밀도함수의그래프와 x 축으로둘러싸인부분의넓이가 1 이므로 1/2\6\k=1.t3 k = 1/3 266 확률변수 X 가이항분포 B(10, p) 를따르므로 P(X=x)=_10&C_x~&p^x&(1-p)^10-^x (x=0, 1, 2,.c3, 10) 이때 P(X=1)=5P(X=0) 이므로 P(2-<X-<4) 는오른쪽그림의색칠한부분의넓이와같으므로 P(2-<X-<4) =1/2\(1/6+1/3)\2=1/2 y y=f{x} O x _10&C_1~&p^1(1-p)^9=5_10&C0~&p0&(1-p)^10 10p(1-p)^9=5(1-p)^10 10p=5(1-p).t3 p = 1/3.t3 P(X=2)=_10&C_2(1/3)^^2(2/3)^^8 =5(2/3)^^8 271 f(x)=ax 의그래프와 x 축및직선 x=1 로둘러싸인부분의넓이가 1 이므로 1/2\1\a=1.t3 a=2 또, PÑ0-<X-<1/2)=P(1/2-<X-<bÒ 이려면오른쪽그림에서색칠한두부분 의넓이가같아야한다. 즉, f{x} 2b 2 1 f{x}=2x 267 주사위를던지는시행은독립시행이고, 주사위를한번던질 1/2\1/2\1=1/2\(1+2b)\Ñb-1/2) O 1-2 b1 x 때 4 의눈이나올확률은 1/6 이므로확률변수 X 는이항분포 BÑ10, 1/6 ) 을따른다..t3 E(4^X)=sigx=0 ^10``4^x&_10&C_x~(1/6)^^x(5/6)^^1^^0-^^x =sigx=0 ^10``_10&C_x~(4/6)^^x(5/6)^^1^^0-^^x =(4/6+5/6)^^1^^0=(3/2)^^1^^0 b^2=1/2.t3 b= rt2 2.t3 ab=2\ rt2 2 =rt2 (.T3 b>0) 272 함수 y=~f(x) 의그래프는오른쪽그림과같고, P(X-<a) 는그림의색칠 한부분의넓이와같으므로 P(X-<a) y 1 2-a y=f{x} O 1 a 2 x 268 E={ (1, 1), (1, 2), (1, 3), (1, 4), (2, 1), (2, 2), (2, 3), (2, 4), (3, 1), (3, 2), (3, 3), (3, 4), (4, 1), (4, 2), (4, 3)} =1-1/2\(2-a)\(2-a)=7/8 4a^2&-16a+15=0, (2a-3)(2a-5)=0.t3 a = 3/2 (.T3 a<2) 이므로 P(E)=15/36=5/ x^2&-6x+1=0 에서 따라서확률변수 X 는이항분포 BÑ24, 5/1 2 ) 를따르므로 (4x-1)(2x-1)=0.t3 x=1/4 또는 x=1/2 V(X)=24\5/12\7/12=35/6.t3 p+q=6+35=41 사건 E가일어날확률구하기 40% X의분산구하기 40% p+q의값구하기 20% P(X-<2), P(X-<4) 의값이이차방정식 8x^2&-6x+1=0의두근이고, P(X-<2)-<P(X-<4) 이므로 P(X-<2)=1/4, P(X-<4)=1/2.t3 P(4-<X-<5)=1-P(X-<4) =1-1/2=1/2 274 정규분포곡선은직선 x=m 에대하여대칭이고, P(X-<17)=P(X->23) 이므로 269 P(0-<X-<1) 은 ~f(x)=1/2&x의그래프와 x축및직선 x=1로둘러싸인부분의넓이와같으므로 P(0-<X-<1)=1/2\1\1/2 f{x} O 1 1 f{x}= -x 2 2 x m= = m 의값이일정할때, sigma 의값이클수록가운데부분의높이는낮아지면서그래프의모양은양쪽으로퍼지고 sigma 의 값이작을수록가운데부분의높이는높아지면서그래프 = 1/4 의모양은뾰족하게된다. 40 바른답 알찬 해 (028~056) 일등확통 -04~06 강 ok.indd 오후 3:33

41 276 P( Z -<1.84)=P(-1.84-<Z-<1.84) =2~P(0-<Z-<1.84) =2\0.4671= Z= X-5 ~ 로놓으면확률변수 Z는표준정규분포 N(0, 1) 2 을따르므로 P(3-<X-<k)=PÑ 3-5 k-5 -<Z-< 2 2 Ò =PÑ-1-<Z-< k-5 2 Ò =P(0-<Z-<1)+PÑ0-<Z-< k-5 2 Ò 279 확률변수 X 는정규분포 N( 3/2, 2^2Ò 을따르므로 X - 3/2 Z= 2 ~ 으로놓으면확률변수 Z는표준정규분포 N(0, 1) 을따른다. H(0)=P(0-<X-<1) =P^^( 0-3/2 1-3/2 -<Z-< ^^) 2 2 =P(-0.75-<Z-<-0.25) =P(0.25-<Z-<0.75) =P(0-<Z-<0.75)-P(0-<Z-<0.25) = = 확률분포 =0.34+PÑ0-<Z-< k-5 2 Ò=0.82.t3 PÑ0-<Z-< k-5 2 `Ò=0.48 이때 P(0-<Z-<2)=0.48이므로 k-5 =2.t3 k=9 2 오른쪽그림에서 P(0-<X-<1)=P(2-<X-<3) 이므로 H(0)=H(2).t3 H(0)+H(2) =2H(0) =2\0.1747= x 278 학생들의키를확률변수 X라하면 X는정규분포 N(170, 5^2) 을따르므로 Z= X-170 ~ 으로놓으면확률변수 5 Z는표준정규분포 N(0, 1) 을따른다. ⑴ P(167.4-<X-<175.2) =PÑ <Z-< Ò 5 =P(-0.52-<Z-<1.04) =P(0-<Z-<0.52)+P(0-<Z-<1.04) = =0.55 따라서 1000\0.55=550( 명 ) 이므로구하는학생수는 550 이다. ⑵ 키가 200 번째로큰학생의키를 k`cm 라하면 P(X->k)= =0.2 P^(Z-> k-170 Ò= P^(0-<Z-< k-170 Ò=0.2 5.t3 P^(0-<Z-< k-170 Ò=0.3 5 이때 P(0-<Z-<0.84)=0.3 이므로 k-170 =0.84.t3 k= 따라서키가 200 번째로큰학생의키는 174.2`cm 이다. 확률변수 X 를표준화하기 20% 키가 167.4`cm 이상 175.2`cm 이하인학생수구하기 40% 키가 200 번째로큰학생의키구하기 40% 평균이 m 이고표준편차가 sigma 인정규분포의확률밀도함수의그래 프는직선 x=m 에대하여대칭임을이용한다. 280 확률변수 X 가이항분포 BÑ64, 1/2 ) 을따르므로 E(X)=64\1/2=32, V(X)=64\1/2\1/2=16 따라서 X 는근사적으로정규분포 N(32, 4^2) 을따르므로 Z= X-32 로놓으면확률변수 Z는표준정규분포 N(0, 1) 4 을따른다. P(28-<X-<36)=PÑ =P(-1-<Z-<1) =2P(0-<Z-<1) =2\0.34=0.68 -<Z-< Ò 확률변수 X 가이항분포 BÑ100, 1/5 ) 을따르므로 E(X)=100\1/5=20, V(X)=100\1/5\4/5=16 따라서 X는근사적으로정규분포 N(20, 4^2) 을따르므로 Z= X-20 ~ 으로놓으면확률변수 Z는표준정규분포 N(0, 1) 4 을따른다. PÑ X/10-1/5 <1/10)=P(-1/10< X < 1/1 0 ) =P(-5/2< X-20 < 5/2 ) 4 =P(-2.5<Z<2.5) =2~P(0-<Z<2.5) =2\0.4938= 확률분포 41 해 (028~056) 일등확통 -04~06 강 ok.indd 오후 3:33

42 바른답 알찬 282 확률변수 X 는이항분포 B^(48, 1/4 ) 을따르므로 E(X)=48\1/4=12, V(X)=48\1/4\3/4=9 따라서확률변수 X 는근사적으로정규분포 N(12, 3^2) 을따 르므로 Z= X-12 ~ 로놓으면확률변수 Z는표준정규분포 3 N(0, 1) 을따른다. P(6-<X-<21)=PÑ =P(-2-<Z-<3) -<Z-< Ò 3 =P(0-<Z-<2)+P(0-<Z-<3) = = _4_50&C_x(1/3)^^x(2/3)^^45^^0-^^x 은한번의시행에서일어날확률이 1/3 인어떤사건이 450 번의독립시행에서 x 번일어날확률이다. 따라서이사건이일어나는횟수를확률변수 X 라하면 X 는 이항분포 B(450, 1/3 ) 을따르므로 E(X)=450\1/3=150, V(X)=450\1/3\2/3=100 따라서확률변수 X 는근사적으로정규분포 N(150, 10^2) 을 285 확률변수 X 는이항분포 BÑ100, 1/2 ) 을따르므로 E(X)=100\1/2=50, V(X)=100\1/2\1/2=25 따라서확률변수 X는근사적으로정규분포 N(50, 5^2) 을따르므로 Z= X-50 ~ 으로놓으면확률변수 Z는표준정규분포 5 N(0, 1) 을따른다. P(X->55-4k)=0.9772에서 PÑZ-> 55-4k-50 Ò=PÑZ->1-4/5&kÒ= PÑ1-4/5&k-<Z-<0Ò+0.5= t3 PÑ1-4/5&k-<Z-<0Ò= 이때 P(0-<Z-<2)=P(-2-<Z-<0)= 이므로 1-4/5&k=-2.t3 k = 1 5/4.t3 16k=16\15/4=60 P(Z->a)=0.9772에서 >0.5이므로 P(Z->a)=0.5+P(a-<Z-<0) 임을이용한다. 따르므로 Z= X-150 ~ 으로놓으면확률변수 Z는표준정규 10 분포 N(0, 1) 을따른다..t3 ( 주어진식 ) = P(X=140)+P(X=141)+P(X=142)+.c3 +P(X=175) =P(140-<X-<175) =P( ~-<Z-< ~) 10 =P(-1-<Z-<2.5) =P(0-<Z-<1)+P(0-<Z-<2.5) = = 불량품의개수를확률변수 X라하면 X는이항분포 B(400, 0.1) 을따르므로 E(X)=400\0.1=40, V(X)=400\0.1\0.9=36 따라서확률변수 X는근사적으로정규분포 N(40, 6^2) 을따르므로 Z= X-40 ~ 으로놓으면확률변수 Z는표준정규분 6 포 N(0, 1) 을따른다..t3 P(X-<49)=PÑZ-< Ò 6 =P(Z-<1.5)=0.5+P(0-<Z-<1.5) = = 불량품의개수를확률변수 X라하고이항분포와정규분포의관계를이용하여 X를표준화하기 50% 불량품이 49개이하일확률구하기 50% 1 등급문제 pp. 81~ 이산확률변수와확률질량함수전략 P(X=1), 즉짝수의개수가 1일확률을구한다. 확률변수 X가가지는값은 0, 1, 2이고, P(X=1) 은짝수의개수가 1일확률이므로 r1par 동전은뒷면이나오고주사위 1개를던져나온눈의수가짝수일확률은 1/2 \ 1/2 = 1/4 r2par 동전은앞면이나오고주사위 2개를던져나온눈의수가짝수 1개, 홀수 1개일확률은 1/2 \ ( 1/2 \ 1/2 + 1/2 \ 1/2 ) = 1/4 r1par, r2par 에서 P(X=1)=1/4+1/4=1/2 287 이산확률변수와확률질량함수전략확률의총합이 1임을이용하여 F(x) 와 G(x) 사이의관계식을구한다. 42 바른답 알찬 해 (028~056) 일등확통 -04~06 강 ok.indd 오후 3:33

43 P(0-<X-<9)=1이므로 P(0-<X-<9)=P(0-<X-<x)+P(x<X-<9).t3 F(x)+G(x)=1 ㄱ. F(4)+G(4)=1이므로 G(4)=1-F(4) ㄴ. P(4-<X-<7) =P(0-<X-<7)-P(0-<X-<3) =F(7)-F(3) ㄷ. P(4-<X-<7) =F(7)-F(3) (.T3 ㄴ ) ={1-G(7)}-{1-G(3)} (.T3 F(x)+G(x)=1) =G(3)-G(7) 이상에서옳은것은ㄱ, ㄷ이다..t3 E(X^2)=V(X)+{E(X)}^2 =25/4+Ñ-5/2&)^^2=25/2 290 연속확률변수와확률밀도함수전략확률밀도함수의그래프와 x축으로둘러싸인부분의넓이가 1임을이용한다. ㄱ. 확률밀도함수의그래프와 x축으로둘러싸인부분의넓이가 1이므로 1/2\4\k=1.t3 k = 1/2 ㄴ. P(-1-<X-<0) 은오른쪽그림의색칠한부분의넓이와같으므로 P(-1-<X-<0) =1/2\(1/4+1/2)\1 y O 2 x 05 확률분포 288 이항분포 전략확률변수 X 가이항분포 B(n, p) 를따를때, P(X=x)=_n&C_x&p^x(1-p)^n-^x (x=0, 1, 2,.c3, n) 임을이용한다. 확률변수 X 가이항분포 BÑ18, 1/3 ) 을따르므로 P(X-<1)=P(X=0)+P(X=1) =_18&C_0(1/3)^^0(2/3)^^18&+_18&C_1&(1/3)^^1(2/3)^^17 =10\(2/3)^^18&.t3 P(X->1 X-<1)= P(X=1) P(X-<1) 9\(2/3)^^18& = 10\(2/3)^^18& = 9/1 0 = 3/8 ㄷ. P(-1-<X-<1~ ~0-<X-<2)= P(0-<X-<1) P(0-<X-<2) ~ 이므로 오른쪽그림에서 P(0-<X-<2) 는색칠한부분의넓이와같고, y 1-2 P(0-<X-<1) 은빗금친부분의넓이와같다. -2 O 1 2 x.t3 ~P(-1-<X-<1~ ~0-<X-<2) = P(0-<X-<1) P(0-<X-<2) 1/2\(1/4+1/2)\1 = 1/2 \ 2 \ 1/2 = 3/4 이상에서옳은것은ㄱ, ㄴ이다. 289 이산확률변수의기댓값, 분산 + 이항분포전략 E(aX+b)=aE(X)+b, V(aX+b)=a^2&V(X) 를이용하여 E(X^2) 의값을구한다. 한개의주사위를 5번던졌을때 4의눈이나오는횟수를확률변수 Y라하면 Y는이항분포 BÑ5, 1/6 ) 을따른다. X는원점 O와점 P 사이의거리이므로 X=2Y-(5-Y).t3 X=3Y-5 이때 E(Y)=5\1/6=5/6, V(Y)=5\1/6\5/6=25/36 이므로 E(X)=E(3Y-5)=3E(Y)-5 =3\5/6-5=-5/2 V(X)=V(3Y-5)=3^2&V(Y) =9\25/36=25/4 291 정규분포와표준정규분포전략확률변수 X가정규분포 N(m, sigma^2) 을따를때, 확률변수 Z= X-m 은표준정규분포 N(0, 1) 을따름을이용한다. sigma 제품 A의무게를확률변수 X라하면 X는정규분포 N(m, 1) 을따르므로 Z_X=X-m으로놓으면확률변수 Z_X 는표준정규분포 N(0, 1) 을따른다. 따라서제품 A의무게가 k 이상일확률은 P(X->k)=P(Z_X->k-m) 또, 제품 B의무게를확률변수 Y라하면 Y는정규분포 N(2m, 2^2) 을따르므로 Z_Y= Y-2m ~ 으로놓으면 2 확률변수 Z_Y 는표준정규분포 N(0, 1) 을따른다. 따라서제품 B의무게가 k 이하일확률은 P(Y-<k)=PÑZ_Y-< k-2m Ò 2 =PÑZ_Y->- k-2m Ò 확률분포 43 해 (028~056) 일등확통 -04~06 강 ok.indd 오전 9:59

44 바른답 알찬 이때제품 A 의무게가 k 이상일확률과제품 B 의무게가 k 이하일확률이같으므로 P(Z_X->k-m)=P^(Z_Y->- k-2m ^)에서 2 k-m=- k-2m 2 2k-2m=-k+2m, 3k=4m.t3 ;k/m := 4/3 292 정규분포와표준정규분포 전략 f(x) 가 ~f(40-x)=f(x) 를만족시키므로함수 ~f(x) 의그래 프는직선 x=20 에대하여대칭이다. f(x) 가모든실수 x 에대하여 ~f(40-x)=f(x) 이므 로확률밀도함수 ~f(x) 의그래프는직선 x=20 에대하여대 칭이다. 따라서 E(X)=20 이므로확률변수 X 는정규분포 N(20, 5^2) 을따른다. 이때 Z= X-20 ~ 으로놓으면확률변수 Z는표준 5 정규분포 N(0, 1) 을따르므로 P(X-<15)=P^(Z-< ^) 5 =P(Z-<-1) =0.5-P(0-<Z-<1) = = P(X->30)=PÑZ-> ^) 5 =P(Z->2) =0.5-P(0-<Z-<2) = = 따라서 2 차기록측정대상자가될확률은 P(X-<15)+P(X->30)= = 이므로 2 차기록측정대상자수는 10000\0.1815=1815 참고 f(40-x)=f(x) 에 x 대신 20+x 를대입하면 f(40-20-x)=f(20+x), 즉 ~f(20-x)=f(20+x) 이므 로함수 ~f(x) 의그래프는직선 x=20 에대하여대칭이다. 293 정규분포와표준정규분포 전략 기업들의점수를확률변수 X 라하고 X 를표준화하여상위 1`% 이내에속하기위한최저점수를구한다. 기업들의점수를확률변수 X 라하면 X 는정규분포 N(160, 10^2) 을따르므로 Z= X-160 ~ 으로놓으면확률변 10 수 Z 는표준정규분포 N(0, 1) 을따른다. 상위 1`% 이내에속하는기업의최저점수를 a 점이라하면 P(X->a)=0.01 PÑZ-> a-160 ^)= PÑ0-<Z-< a-160 ^)= PÑ0-<Z-< a-160 ^)= 이때 P(0-<Z-<2.5)=0.49 이므로 a-160 =2.5.t3 a= A 기업이경영관리분야에서 85 점, 경영합리화분야에서 60 점을받았으므로공정거래기여분야에서받은점수를 x 점이 라하면 x->185.t3 x->40 따라서 A 기업의공정거래기여분야점수는최소 40 점이다. 기업들의점수를확률변수 X 라하고 X 를표준화하기 20% 표창을수여받기위한최저점수구하기 50% 공정거래기여분야의최소점수구하기 30% 294 이항분포와정규분포의관계 전략 주어진식을이용하여 n, p 의값을구한후이항분포와정규분 포의관계를이용한다 /9 이므로 이항분포 B(n, p) 를따르는확률변수 X 의분산이 np(1-p)=100/9.c3.c3 ᄀ 또, P(X=x)=_nC_x&p^x(1-p)^n^-^x(x=0, 1, 2,.c3, n) 이고, P(X=n-1)=16P(X=n) 이므로 _n&c_n-1 &p^n^-1(1-p)1=16_n&c_n& p^n(1-p)^0.t3 n(1-p)=16p ᄂ을ᄀ에대입하면 16p^2=100/9 p= 5/6 를ᄂ에대입하면.c3.c3 ᄂ.t3 p=5/6 (.T3 p>0) n\1/6=16\5/6.t3 n=80 확률변수 X 가이항분포 B^(80, 5/6 & ^) 를따르므로 E(X)=80\5/6=200/3, V(X)=80\5/6\1/6=100/9 따라서 X 는근사적으로정규분포 N^( 2 0 0/3, ^(10/3^)^^2^) 을따르 X /3 므로 Z= ~ 으로놓으면확률변수 Z는표준정규분 1 0/3 포 N(0, 1) 을따른다 /3 P(X->60)=P^^(Z-> ^^) 1 0/3 =P(Z->-2) =0.5+P(0-<Z-<2) = = 바른답 알찬 해 (028~056)-04~06 강 ok.indd 오후 2:15

45 295 이항분포와정규분포의관계 전략 계란 1 개의무게를확률변수 X 라하고계란이특란일확률을 구한후이항분포와정규분포의관계를이용한다. 계란 1 개의무게를확률변수 X 라하면 X 는정규분 포 N(50, 5^2) 을따르므로 Z_X= X-50 ~ 으로놓으면확률변 5 수 Z_X 는표준정규분포 N(0, 1) 을따른다. 이때계란이특란일확률은 P(X->60)=PÑZ_X-> Ò 5 =P(Z_X->2) =0.5-P(0-<Z_X-<2) = =0.02 임의로택한 2500 개의계란중특란의개수를확률변수 Y 라 하면 Y 는이항분포 B(2500, 0.02) 를따르므로 E(Y)=2500\0.02=50, V(Y)=2500\0.02\0.98=49 따라서 Y 는근사적으로정규분포 N(50, 7^2) 을따르므로 Z_Y= X-50 ~ 으로놓으면 Z_Y 는표준정규분포 N(0, 1) 을 7 따른다..t3 P(Y->57)=P^(Z_Y-> Ò 7 =P(Z_Y->1) =0.5-P(0-<Z_Y-<1) = =0.16 r2par 동전 2개를던져모두앞면이나오고주머니에서꺼낸 2 개의구슬중 1개만검은구슬일확률은 (1/2\1/2)\(3/10\7/9+7/10\3/9)=7/60 r1par, r2par 에서 P(X=1)=3/20+7/60=4/15 P(X=0)=1-{P(X=1)+P(X=2)} =1-(4/15+1/60)=43/60 확률변수 X의확률분포를표로나타내면다음과같다. X 합계 P(X=x) 4 3/6 0 4/1 5 1/ 단계 E(X), V(X) 의값을구한다. E(X)=0\43/60+1\4/15+2\1/60 = 3/1 0 V(X)=E(X^2)-{E(X)}^2 =0^2&\43/60+1^2&\4/15+2^2&\1/60-(3/10)^^2 = 7 3/3 0 4단계 V(30X) 의값을구한다..t3 V(30X)=30^2&~V(X) =900\73/30= 확률분포 1등급비법임의로택한계란 1개의무게가 60`g 이상일확률은 0.02이므로확률변수 Y는이항분포 B(2500, 0.02) 를따른다. 296 이산확률변수의평균, 분산, 표준편차 1단계검은구슬의개수를구한다. 확률변수 X 가가지는값은 0, 1, 2 이고, P(X=2)=1/60 에 서검은구슬이 2 개이려면동전 2 개를던졌을때모두앞면 이나오고주머니에서꺼낸 2 개의구슬이모두검은구슬이 어야한다. 10 개의구슬중검은구슬의개수를 x 라하면흰구슬의개 수는 10-x 이므로 (1/2\1/2)\ x 10 \ x-1 9 x^2&-x-6=0, (x+2)(x-3)=0.t3 x=3 (.T3 x->0) x(x-1) = 1/6 0, = 1/ 따라서주머니속에는흰구슬 7 개, 검은구슬 3 개가들어있다. 2 단계 P(X=0), P(X=1) 을구하여확률분포표를만든다 r1par 동전 2 개를던져앞면이 1 개나오고주머니에서꺼낸 1 개 의구슬이검은구슬일확률은 (1/2\1/2+1/2\1/2)\3/1 0 = 3/ 이항분포 1 단계주어진사건이이항분포를따름을알고, E(X) 와 E(X^2) 의값 을구한다. 흰구슬 4 개와검은구슬 1 개가들어있는상자에서한개의 구슬을꺼낼때, 흰구슬을꺼낼확률은 4/5 이므로확률변수 X 는이항분포 BÑ250, 4/5 ) 를따른다. E(X)=250\4/5=200 E(X^2)=V(X)+{E(X)}^2 =250\4/5\1/5+200^2 = 단계 f(x) 의최솟값을구한다. f(x)=sigk=0 ^250&~(x-k)^2&~P(X=k) =x^2~sigk=0 ^250`P(X=k)-2x~sigk=0 ^250`kP(X=k)+sigk ^250`k^2&~P(X=k) = 0 =x^2 1-2x E(X)+E(X^2) =x^2&-400x =(x-200)^2&+40 따라서 ~f(x) 는 x=200 일때최솟값 40 을갖는다. 05. 확률분포 45 해 (028~056) 일등확통 -04~06 강 ok.indd 오후 3:34

46 바른답 알찬 06 통계적추정 교과서에서뽑은기본문제 pp. 84~85 ⑵ 모비율 p 에대한신뢰도 99`% 의신뢰구간은 \ t p b~ p \0.2 b~ ⑴ 참조 ⑵ E(X^-)=4, V(X^-)=4/3 299 ⑴ 100 ⑵ 4 ⑶ ⑴ <m-< ⑵ <m-< N(0.3, ) 303 ⑴ p ⑵ p ⑴ X^ 합계 P(X^-=x^-) 1/9 2/9 1/3 2/9 1/9 1 ⑵ E(X^-)=2 1/ / / / /9 =4 V(X^-)=2^2 1/9+3^2 2/9+4^2 1/3+5^2 2/9+6^2 1/9-4^2 = 4/3 299 m=100, sigma^2=16, n=4 이므로 ⑴ E(X^- )=m=100 ⑵ V(X^- )=#/n!sigma^^2 $ =16/4=4 ⑶ sigma(x^- )=3V(X^- )c=rt4~=2 300 x^-=65, sigma=4, n=400 이므로 ⑴ 모평균 m 에대한신뢰도 95`% 의신뢰구간은 rt400~~ m rt400~~.t m ⑵ 모평균 m 에대한신뢰도 99`% 의신뢰구간은 rt400~~ m rt400~~.t m n=50, X=21 이므로표본비율 p^^ 은 p^^ =;X/n := 2 1/5 0 = 모비율이 0.3 이고, 표본의크기가 100 이므로표본비율 p^^은근사적으로정규분포 N(0.3, 0.3\0.7 ), 즉 N(0.3, ) 100 을따른다. 303 p^^ =0.8, n=100 이므로 ⑴ 모비율 p 에대한신뢰도 95`% 의신뢰구간은 \ t <p-< b~ p \0.2 b~ 100 기출문제 / / pp. 86~ <m-< ⑴ 4.4-<m-<5.6 ⑵ 제품의무게를확률변수 X 라하면 E(X)=20, V(X)=16, n=16 이므로 E(X^-)=20, V(X^-)=16/16=1 V(X^-)=E( X^- ~^2)-{E(X^-)}^2 에서 E( X^- ~^2) =V(X^-)+{E(X^-)}^2 =1+400= 장미꽃의길이를확률변수 X 라하면 E(X)=50, V(X)=4, n=100 이므로 E(X^-)=50, V(X^-)=4/10=1/25 따라서구하는값은 E(X^-)V(X^-)=50\1/25=2 306 주머니에서임의로 1 개의공을꺼낼때, 공에적힌숫자를확률변수 X 라하면 X 의확률분포는다음표와같다. X 합계 P(X=x) 3/2 0 1/5 3/1 0 1/5 3/2 0 1 E(X)=0\3/20+1\1/5+2\3/10+3\1/5+4\3/20 =2 V(X)=0^2&\3/20+1^2&\1/5+2^2&\3/10+3^2&\1/5 = 8/5 이때표본의크기가 10 이므로 8/5 V(X^-)= 10 = 4/2 5.t3 sigma(x^-)=3v(x^-)c~=44/2 5 r~= 2/5 +4^2&\3/20-2^2 46 바른답 알찬 해 (028~056) 일등확통 -04~06 강 ok.indd 오후 3:34

47 307 카드에적힌숫자를확률변수 X 라하면 X 의확률분포는다음표와같다. P(X^--<9)=P^^(Z-< /2 ~^^)=P(Z-<-2) X 합계 P(X=x) 1/2 1/3 1/6 1 E(X)=1\1/2+2\1/3+3\1/6=5/3 V(X)=E(X~^2)-{E(X)}^2 =1^2&\1/2+2^2&\1/3+3^2&\1/6-(5/3)^^2=5/9 따라서 X^- 의분산은 5/9 V(X^-)= = 1/9 5 =0.5-P(0-<Z-<2) = = 모집단이정규분포 N(m, m^2 ~) 을따르고, 표본의크기가 이므로표본평균 X^- 는정규분포 N^^(m, 즉 N(m, ( m )^^2`) 을따른다. 40 m^ ^^), 따라서 Z= X^--m ~ 으로놓으면확률변수 Z는표준정규분포 m/ 통계적추정 X 의확률분포를표로나타내기 30% ~E(X), V(X) 의값구하기 40% V(X^-) 의값구하기 30% 308 확률의총합이 1 이므로 1/4+a+1/2=1.t3 a = 1/4 E(X)=(-1)\1/4+0\1/4+1\1/2=1/4 V(X)=E(X^2)-{E(X)}^2 =(-1)^2&\1/4+0^2&\1/4+1^2&\1/2-(1/4)^^2=11/16 이때표본의크기가 4 이므로 E(X^-)=1/4, V(X^-)= = 1 1/6 4 V(X^-)=E( X^- ~^2)-{E(X^-)}^2~ 에서 E( X^- ~^2)=V(X^-)+{E(X^-)}^2=11/64+(1/4)^^2=15/ m=10, sigma^2=25, n=100 이므로 E(X^-)=m=10, V(X^-)= sigma^2 = 25/10 = 1/4, n sigma(x^-)=3v(x^- ) c=rt1/4~=1/2 X^- 를표준화하면 Z= X^--10 ~=2(X^--10) 1/2 따라서옳지않은것은 4 이다. 310 모집단이정규분포 N(10, 3^2) 을따르고, 표본의크기가 36 이므로표본평균 X^- 는정규분포 N(10, (1/2)^^2) 을따른다. 따라서 Z= X^--10 ~ 으로놓으면확률변수 Z는표준정규분포 1/2 N(0, 1) 을따르므로구하는확률은 N(0, 1) 을따르므로 P(m-<X^--<82)=P^^(0-<Z-< 82-m m/4 0 ^^) 이때 P(m-<X^--<82)=P(0-<Z-<1) 이므로 82-m m/4 0 ~ =1, 82-m=m/40.t3 m= E(X^-)=100, V(X^-)= V(X) =36이므로 4 E(X)=100, V(X)=144 따라서확률변수 X 는정규분포 N(100, 12^2) 을따른다. Z= X-100 ~ 으로놓으면 12 P(88-<X-<130)=P( =P(-1-<Z-<2.5) -<Z-< ) 12 =P(0-<Z-<1)+P(0-<Z-<2.5) = = 상담시간을확률변수 X 라하면 X 는정규분포 N(20, 5^2) 을따르고, 표본의크기가 16 이므로표본평균 X^- 는정규분포 N(20, #~~!5^^2 /1 6 ), 즉 N(20, (5/4)^^2) 을따른다. 따라서 Z= X^--20 ~ 으로놓으면확률변수 Z는표준정규분포 5/4 N(0, 1) 을따르므로구하는확률은 P(19-<X^--<22)=P^^( ~-<Z-<~ ~^^) 5/4 5/4 =P(-0.8-<Z-<1.6) =P(0-<Z-<0.8)+P(0-<Z-<1.6) = = 통계적추정 47 해 (028~056) 일등확통 -04~06 강 ok.indd 오후 3:34

48 바른답 알찬 314 표본평균 x^-=67, 모표준편차 sigma=4, 표본의크기 n=100 이므로모평균 m 에대한신뢰도 99`% 의신뢰구간은 ~ rt100~ ~-<m-<67+3~ 4 rt100~.t <m-< 표본의크기가 n, 표본평균 x^-=11, 모표준편차 sigma=5 이므로모평균 m 에대한신뢰도 95`% 의신뢰구간은 ~#5/rtn$:-<m-< ~#5/rtn$: 이때 <m-<11.49 이므로 ~#5/rtn$:=10.51, ~#5/rtn$:= ~#5/rtn$:=0.49, rtn~=20.t3 n= 딸기의무게를확률변수 X 라하고 X 가정규분포 N(m, sigma^2) 을따른다고하자. 표본평균이 20, 표본표준편차 가 5 이므로모평균 m 을신뢰도 95`% 로추정한신뢰구간은 ~#5/rtn$:-<m-< ~#5/rtn$: 이때 <m-<a 이므로 &#5/rtn$:=19.02, &#5/rtn$:=a 두식을연립하여풀면 n=100, a=20.98.t3 n+a= ⑴ 표본의크기 100 이충분히크므로모표준편차대신표본표준편차 3 을사용할수있고, 표본평균이 5 이므로모평 균 m 에대한신뢰도 95`% 의신뢰구간은 5-2\ 3 rt100~ ~-<m-<5+2\ 3 rt100~.t3 4.4-<m-<5.6 ⑵ 표본의크기를 n 이라하면모평균 m 에대한신뢰도 95`% 의신뢰구간은 X^- - 2\#3/rtn$:-<m-<X^-+2\#3/rtn$: -#6/rtn$:-<m-X^- -<#6/rtn$:.t3 m-x^- -<#6/rtn$: 모평균 m 과표본평균 X^- 의차가 1 분이하이어야하므로 #6/rtn$:-<1, rtn~->6.t3 n->36 따라서표본의크기의최솟값은 36 이다. 신뢰도 95`% 의신뢰구간구하기 50% 표본의크기의최솟값구하기 50% 318 학생 150 명중에서여름휴가장소로바다를선호하는학생의비율을 p^^ 이라하면표본의크기 150 이충분히크므로 p^^ 의분 포는근사적으로정규분포 N(0.4, 0.4\0.6 ~) 을따르고, 150 Z=~ p^^-0.4 p^^-0.4 ~ 는근사적으로표준정규분포 0.4\0.6 ~=~ 0.04 '~~ 150 N(0, 1) 을따른다. 따라서구하는확률은 P(p^^->69/150)=P(p^^->0.46) =P(Z-> ) 0.04 =P(Z->1.5) =0.5-P(0-<Z-<1.5) = = 명중에서 7월에태어난학생의비율을 p^^이라하면표본의크기 100이충분히크므로 p^^의분포는근사적으로정규분포 N(0.2, 0.2\0.8 ~) 을따르고, 100 Z= p^^ \0.8 = p^^-0.2 ~ 는근사적으로표준정규분포 0.04 '~~ 100 N(0, 1) 을따른다. 따라서구하는확률은 P(16/10-<p^^-<30/10^)=P(0.16-<p^^-<0.3) =P( <Z-< ~~) =P(-1-<Z-<2.5) =P(0-<Z-<1)+P(0-<Z-<2.5) = = 주민 100명중에서반대하는주민의비율을 p^^이라하면표본의크기 100이충분히크므로표본비율 p^^의분포는근사적으로정규분포 N(0.1, 0.1\0.9 ~) 를따르고, 100 Z= p^^ \0.9 = p^^-0.1 ~ 은근사적으로표준정규분포 0.03 '~~ 100 N(0, 1) 을따른다. 이때반대하는주민이 k명이하일확률이 0.16이므로 P(p^^-<k/10)=0.16 k/ P^^(Z-< ^^)=0.16.c3.c3 ᄀ 0.03 한편, P(0-<Z-<1)=0.34에서 P(Z-<-1)=P(Z->1) =0.5-P(0-<Z-<1) = =0.16.c3.c3 ᄂ 48 바른답 알찬 해 (028~056) 일등확통 -04~06 강 ok.indd 오후 3:34

49 ' 321 Z=~ k/ ᄀ, ᄂ에서 =-1이므로 0.03 k/1 0 = 0.07.t3 k=7 표본비율 p^^ 의분포구하기 30% k 의값구하기 70% p^^-p 로놓으면 p^^(1-p^^) Z는근사적으로표준정규분포 ~ ~ '~~ n N(0, 1) 을따르므로 P( p^^-p -<0.16 p^^(1-p^^) ~) \0.9 n b~-<p-0.1-<25 0.1\0.9 b~ n p-0.1 -<25 0.1\0.9 b~ n 이때모비율 p~ 와표본비율 p^^~ 의차가 0.1 이하가되어야하므로 \0.9 b~-<0.1 n rtn~->6.t3 n->36 따라서표본은최소 36 명이상이어야한다. p^^의값구하기 10% p 에대한신뢰도 95`% 의신뢰구간을간단히나타내기 50% 표본의크기의최솟값구하기 40% 06 통계적추정 =P(-0.16 p^^(1-p^^) ~-<p^^-p-<0.16 p^^(1-p^^) ~) p^^ (1-p^^ ) ~ 0.16 p^^ (1-p^^ ) ~ =P 5 p^^(1-p^^ ) ~ -<Z-<~ p^^(1-p^^ ) ~6 ' n n =P(-0.16RTn~&-<Z-<0.16RTn~&) =2P(0-<Z-<0.16RTn~&)-> t3 P(0-<Z-<0.16rtn~&)-> 즉, 0.16rtn~->2 에서 rtn~->12.5.t3 n-> 따라서 n 의최솟값은 157 이다. 322 표본비율은 7 5/3 0 =0.25 이고, 표본의크기 300 은충분히크므 로모비율 p 에대한신뢰도 99`% 의신뢰구간은 \ t <p-<0.315 b~-<p-< \0.75 b~ 표본비율이 0.8, 표본의크기가 n 이므로모비율 p 에대한신뢰도 95`% 의신뢰구간은 \0.2 n a= \0.2 n 이때 b-a=0.098 이므로 2\ \0.2 b~=0.098 n rtn~=~ 2\1.96\0.4 = t3 n=256 b~-<p-< \0.2 b~ n b~, b= \0.2 b~ n 325 ㄱ. V(p^^ )= pq n 의분산은작아진다. (q=1-p) 이므로 n 의값이커지면표본비율 ㄴ. ~p>0, q>0(q=1-p) 이므로 rtpq-< p+q 2 = 1/2 sigma(p^^ )=4 pq pq r~ 에서 4 n n r~-< 1 2rtn~ 즉, 표본비율의표준편차의최댓값은 1 2rtn~ ~ 이다. ㄷ. P(0-<Z-<k)=alpha/20 라하면신뢰도 alpha`% 로추정한모비율 p에대한신뢰구간의길이는 2k4 p^^ q^^ r~ (q^^ =1-p^^ ) 이므로 n b-a=2k4 p^^ q^^ n r 이 때 alpha의값이커지면 k의값도커지므로 n과 p^^ 의값이 일정하면 b-a 의값은커진다. 이상에서옳은것은ㄱ, ㄴ이다. 326 표본의크기 100 이충분히크므로모표준편차대신표본표준편차 10 을사용할수있다. 따라서모평균 m 에대한신뢰도 95`% 의신뢰구간의길이는 2\2 10 rt100~~ =4 327 표본비율은 144/400 = 0.36 이고, 표본의크기 400 이충분히크 므로모비율 p 에대한신뢰도 95`% 의신뢰구간의길이는 2\ \ b~= 구하는표본의크기를 n 이라하면표본비율 p^^은 p^^ =10/10=0.1 이므로모비율 p 에대한신뢰도 95`% 의신뢰구간은 \0.9 n b~-<p-< \0.9 b~ n sigma 328 신뢰도 95`% 의신뢰구간의길이는 2\1.96~ rtn~ 6 이때 sigma=6이므로 2\1.96~ rtn~ -<2 rtn~->11.76.t3 n-> 따라서구하는최솟값은 139이다. 06. 통계적추정 49 해 (028~056) 일등확통 -04~06 강 ok.indd 오전 10:01

50 바른답 알찬 329 ㄱ. 표본의크기를 n 이라하면표본평균 X^- 의분산은 V(X^-)=~ sigma^2 ~ 이므로 V(X^-) 는표본의크기에반비례한다. n ㄴ. 크기가 n 인표본의표본평균 X^- 의값을 x^- 라하면 모평균 m 에대한신뢰도 95`% 의신뢰구간은 sigma x^ ~ rtn~ -<m-<x^ ~ sigma rtn~ ~ 모평균 m 에대한신뢰도 99`% 의신뢰구간은 sigma x^ ~ rtn~ -<m-<x^ ~ sigma rtn~ ~ 따라서신뢰도 99`% 의신뢰구간은신뢰도 95`% 의신뢰 구간을포함한다. ㄷ. 표본의크기를 n 이라하면 P( Z -<k)=alpha/10 일때, 모평균 m 에대한신뢰도 alpha`% sigma 의신뢰구간의길이는 2k~ ~ 이고신뢰도가일정하므로 rtn~ k 의값은일정하다. 따라서표본의크기가작을수록신뢰구간의길이는길어진 다. 이상에서옳은것은ㄱ, ㄴ이다. 2\a+4\(1/3-a)+6\1/2+8\1/6=5.t3 a = 1/3 V(X)=E(X^2)-{E(X)}^2 =2^2&\1/3+4^2&\0+6^2&\1/2+8^2&\1/6-5^2=5 이때표본의크기가 2이므로 V(X^-)=5/2 V(X^-)=E(X^-~^2)-{E(X^-)}^2에서 E(X^-~^2)=V(X^-)+{E(X^-)}^2 =5/2+5^2=55/2 V(2X^-+1)=2^2 V(X^-)=4\5/2=10.t3 E(X^-~^2)+V(2X^-+1) =55/2+10=75/2 a의값구하기 20% V(X^-) 의값구하기 20% E(X^-~^2) 의값구하기 25% V(2X^-+1) 의값구하기 25% E(X^-~^2)+V(2X^-+1) 의값구하기 10% 1 등급문제 / 표본평균의평균, 분산, 표준편차 전략 E(X^-)=m, V(X^-)= sigma^2 ~ 임을이용한다. n V(X^-)=4 이므로 #/n$=4!4^^2.t3 n=4 또, E( X^- )=8 이고, V(X^-)=E( X^- ~^2)-{E( X^- )}^2 이므로 4=E( X^- ~^2)-64.t3 E(X^- ~^2)=68 따라서 X^- ~^2+2n 의평균은 E( X^- ~^2+2n)=E( X^- ~^2+8) =E( X^- ~^2)+8 =68+8= 표본평균의평균, 분산, 표준편차 전략 E(X^-)=E(X) 임을이용하여 a의값을구한다. E(X^-)=E(X)=5이므로 pp. 92~ 표본평균의분포 전략 모집단이정규분포를따르면표본의표본평균도정규분포를따 른다는것을이용하여표준화한다. ㄱ. V(X^-)=# /n!2^^2 $ = 4/n ㄴ. E(X^-)=10에서 X^- 는정규분포 N(10, #/n$)!2^^2 을따르고, E(X^-)=10 이므로표본평균 X^- 의정규분포곡선은오른쪽그림 과같이직선 x^-=10 에대하여대 칭이다..t3 P(X^--<10-a)=P(X^-->10+a) ㄷ. Z= X^--10 ~ 으로놓으면확률변수 Z는표준정규분포 #2/rtn$: N(0, 1) 을따르므로 P(X^-->a)=P^^(Z-> a-10 ^^)=P(Z-<b) #2/rtn$: 이때표준정규분포 N(0, 1) 을따르는확률변수 Z 의정 규분포곡선은직선 z=0 에대하여대칭이므로 ~ a-10 +b=0, a ~ b=0 #2/rtn$: rtn~~.t3 a+ 2 rtn~ ~b=10 이상에서ㄱ, ㄴ, ㄷ모두옳다. 10-a a x 50 바른답 알찬 해 (028~056) 일등확통 -04~06 강 ok.indd 오후 3:34

51 333 표본평균의분포 전략표본평균 X^- 의분포를구한후 X^- 를표준화한다. 전구의수명을확률변수 X 라하면모집단이정규분 포 N(1400, 100^2) 을따르고, 표본의크기가 n 이므로표본 평균 X^- 는정규분포 N(1400, ( 100 )^^2~) 을따른다. rtn~ Z=~ X^ 으로놓으면확률변수 Z는표준정규분포 100 ~ rtn~ N(0, 1) 을따르므로 P(X^--> rtn~ )=P~7Z->~ rtn~ ~8 100 rtn~ rtn~ =P~7Z->~ rtn~ =P(Z->1.65- RTn~ 2 ) 즉, P(Z->1.65- RTn~ )->0.95이므로 P(1.65- RTn~ 2 -<Z-<0)->0.95 P(0-<Z-< RTn~ )->0.45 이때 P(0-<Z-<1.65)=0.45 이므로 RTn~ >1.65 RTn~ ->3.3, RTn~->6.6 2.t3 n->43.56 따라서구하는 n 의최솟값은 44 이다. 12.9~ -<0.5, rtn~->25.8 rtn~.t3 n-> 따라서표본의크기를 666 이상으로해야한다. 335 표본비율의분포 전략 표본의크기 n 이충분히클때, 표본비율 p^^ 의분포는근사적으 로정규분포 N(p, p(1-p) ~) 를따르고, Z= p^^-p n p(1-p) ~ 는근사적 n 으로표준정규분포 N(0, 1) 을따름을이용한다. 100 가구중에서자택을소유한가구의비율을 p^^ 이라 하면표본의크기 100 이충분히크므로 p^^ 의분포는근사적으 로정규분포 N(p, p(1-p)~ ) 를따르고, 100 Z=~ p^^ -p p(1-p) 는표준정규분포 N(0, 1) 을따른다. ` '~~ 100 P(p^^ ->0.6)= 이므로 P~ 5 Z-> ~ 0.6-p 즉, ~ p(1-p) '~ ~ 6= =0.5+P(0-<Z-<2)~ 100 =0.5+P(-2-<Z-<0) =P(Z->-2) 0.6-p =-2이므로.c3.c3 ᄀ p(1-p) '~~ ~ p=-25 p(1-p) b b 100 양변을제곱하여정리하면 26p^2&-31p+9=0 (2p-1)(13p-9)=0 06 통계적추정 1등급비법확률변수 Z가표준정규분포 N(0, 1) 을따르고, P(Z->a)=p일때, p>0.5이면 a<0이므로 P(Z->a)=P(a-<Z-<0)+P(Z->0) =P(a-<Z-<0)+0.5.t3 p = 1/2 또는 p = 9/1 3 ᄀ에서좌변이음수가되려면 0.6-p<0이어야하므로 p>0.6 따라서 p = 9/1 3 이므로 26p=26\9/13= 모평균의추정 전략 신뢰도 99% 로추정한모평균 m 과표본평균 X^- 의차는 sigma m-x^- -<2.58~ ~ 임을이용한다. rtn~ 표본평균이 80`g 이고, 표본의크기가충분히크므로 모표준편차대신표본표준편차 5`g 을사용할수있다. 표본의크기를 n 이라하면신뢰도 99`% 로추정한모평균 m 의신뢰구간은 5~ ~ rtn~ -<m-< ~ 5~ rtn~.t3 m-80 -< 12.9~ rtn~ 이때모평균과표본평균의차 m-80 이 0.5`g 이하이려면 336 모평균의추정 + 신뢰구간의길이 1 단계주어진표준정규분포표를이용하여신뢰도가 98`% 일때의신 뢰구간의길이를구한다. P(0-<Z-<2.32)=0.49 에서 P(-2.32-<Z-<2.32)=0.98 모평균 m 에대한신뢰도 98`% 의신뢰구간이 x^--d-<m-<x^-+d 이므로신뢰구간의길이 2d 는 2d=2\2.32 sigma RTn~~ 2 단계주어진신뢰구간을이용하여신뢰도 alpha`% 를구한다. 모평균 m 에대한신뢰도 alpha`% 의신뢰구간이 x^--d/2-<m-<x^-+d/2 이므로신뢰구간의길이 d 는 06. 통계적추정 51 해 (028~056) 일등확통 -04~06 강 ok.indd 오후 3:34

52 바른답 알찬 sigma d=2.32~ RTn~ sigma.t3 d=2\1.16~ RTn~ 이때 P(0-<Z-<1.16)=0.38 이므로 P(-1.16-<Z-<1.16)=0.76 따라서신뢰구간의길이가 d 인신뢰구간의신뢰도는 76`% 이다..t3 alpha=76 P(-k-<Z-<k)=alpha/10 일때, 모평균 m 에대한신뢰도 alpha`% 의신 뢰구간의길이는 2\k\ sigma ~ 임을이용한다. rtn~ 337 모비율의추정 1 단계모비율 p 에대한신뢰도 95`% 의신뢰구간을이용하여표본비율 p^^ 을구한다. 표본비율이 p^^ 이고표본의크기가 n 이므로모비율 p 에대한 신뢰도 95`% 의신뢰구간은 p^^ & p^^ q^^ n ~ -<p-<p^^ & p^^ q^^ n ~ ( 단, q^^ =1-p^^ ) 이때 <p-< 이므로 p^^ & p^^ q^^ n ~ = p^^ & p^^ q^^ n ~ = ᄀ + ᄂ을하면 2p^^ &=1.6.t3 p^^ &=0.8.c3.c3 ᄀ.c3.c3 ᄂ 2 단계ᄀ또는ᄂ에표본비율 p^^ 의값을대입하여표본의크기 n 의값 을구한다. p^^ &=0.8이면 q^^ =1-0.8=0.2이므로이것을ᄂ에대입하면 \0.2 b~= n 1.96\0.4 = rtn~ rtn~=10.t3 n=100 3 단계 p^^ 과 n 의값을이용하여학생수를구한다. 여름방학동안의봉사활동시간이 20 시간이상인학생수 를 X라할때, 표본비율 p^^ &~ 은 100명의학생중에서여름방학 동안 20 시간이상봉사활동을한학생의비율이므로 0.8= X/1 0.t3 X=80 따라서구하는학생수는 80 이다. 338 이산확률분포와확률질량함수 전략확률변수 X 의확률의총합은 1 이고, P(X=x_i 또는 X=x_j)=P(X=x_i)+P(X=x_j) (inot=j) 임을이용 한다. a+b+c=1 확률의총합은 1 이므로.c3.c3 ᄀ a, b, c 가이순서대로등차수열을이루므로 b= a+c 2 P(X^2&-5X+6-<0) ~~.c3.c3 ᄂ =P((X-2)(X-3)-<0)=P(2-<X-<3) =P(X=2)+P(X=3)=1/2& b+c=1/2.c3.c3 ᄃ ᄀ, ᄂ, ᄃ을연립하여풀면 a = 1/2, b = 1/3, c = 1/6.t3 ~P(X=3)=1/6 339 이항분포의평균, 분산, 표준편차 전략 확률변수 X 가이항분포 B(n, p) 를따를때, 평균은 E(X)=np, 분산은 V(X)=np(1-p) 임을이용한다. 확률변수 X 가이항분포 B(9, p) 를따르므로 E(X)=9p, V(X)=9p(1-p) {E(X)}^2=V(X) 이므로 (9p)^2=9p(1-p) 90p^2&-9p=0 9p(10p-1)=0.t3 p = 1/1 0 (.T3 0<p<1) 340 정규분포 전략 정규분포 N(m, sigma^2) 을따르는확률변수 X 의확률밀도함수 ~f(x) 의그래프는직선 x=m 에대하여대칭임을이용한다. 확률변수 X 가정규분포 N(m, sigma^2) 을따르므로 P(X-<80)=0.5 에서 m=80 P(X->11/10&m)= 이므로.c3.c3 ᄀ P^(m-<X-<11/10&m)= = 이때 P(m-<X-<m+1.0sigma)= 이므로 11/10&m=m+1.0sigma ~.c3.c3 ᄂ ᄀ을ᄂ에대입하면 실전대비평가문제 ⅠⅠⅠ. 통계 pp. 94~95 11/10\80=80+sigma.t3 sigma=8.t3 P(X->96) =0.5-P(80-<X-<80+2.0\8) = (.T3 P(m-<X-<m+2.0sigma)=0.4772) = 바른답 알찬 해 (028~056) 일등확통 -04~06 강 ok.indd 오후 3:34

53 341 이항분포와정규분포의관계 전략 확률변수 X 가이항분포 B(n, p) 를따를때, n 이충분히크면 X 는근사적으로정규분포 N(np, npq) 를따름을이용한다. ( 단, p+q=1) 확률변수 X 가이항분포 B(450, p) 를따르므로 E(X)=450p, V(X)=450p(1-p) 이때 450 은충분히큰수이므로확률변수 X 는근사적으로 정규분포 N(450p, 450p(1-p)) 를따른다. 이때 P(X-<150)=0.5 에서 450p=150.t3 p = 1/3.t3 V(X)=450\1/3\2/3=100 즉, 확률변수 X 는근사적으로정규분포 N(150, 10^2) 을따른 다. Z= X-150 ~ 으로놓으면확률변수 Z는표준정규분포 10 N(0, 1) 을따르므로 P(X-<135)=PÑZ-< Ò 표본평균의분포 전략 =P(Z-<-1.5) =P(Z->1.5) =0.5-P(0-<Z-<1.5) = = 모집단이정규분포 N(m, sigma^2) 을따를때, 크기가 n 인표본의표 본평균 X^- 는정규분포 N^(m, sigma^2 n Ò 을따른다. 모집단이정규분포 N(75, 4^2) 을따르고표본의크기 가 n이므로표본평균 X^- 는정규분포 N^(75, ^( 4~ ^)^^2^)~ 을따른다. rtn~ 이때 Z=~ X^--75 로놓으면 Z는표준정규분포 N(0, 1) 을 4 ~ rtn~ 따르므로 P(73-<X^--<77)->0.96 에서 <Z-< P^^( #4/rtn$: #4/rtn$: ^^) =PÑ- rtn~ rtn~ -<Z-< 2 2 ^) =2P^(0-<Z-< rtn~ 2 ^)->0.96.t3 P^(0-<Z-< rtn~ 2 ^)->0.48 그런데 P(0-<Z-<2)=0.48 이므로 rtn~ ->2, rtn~->4 2.t3 n->16 따라서 n 의최솟값은 16 이다. 343 모평균의추정 전략 모평균 m 에대한신뢰도 95`% 의신뢰구간은 sigma x^ ~ rtn~ -<m-<x^-+1.96~ sigma ~ 임을이용한다. rtn~ 표본평균의값을 x^-, 표본의크기를 n 이라하면모평 균 m 에대한신뢰도 95`% 의신뢰구간은 sigma x^ ~ rtn~ ~-<m-<x^-+1.96~ sigma rtn~ ~ 이므로 sigma x^ ~ rtn~ ~=100.4 sigma x^ ~ rtn~ ~=139.6 ᄀ + ᄂ을하면 2x^-=240.t3 x^-=120 x^-=120 을ᄂ에대입하면 sigma 1.96~ rtn~ ~=19.6.c3.c3 ᄀ.c3.c3 ᄂ sigma.t3 rtn~ ~=10 따라서모평균 m 에대한신뢰도 99`% 의신뢰구간은 \10-<m-< \10 즉, 94.2-<m-<145.8 이므로신뢰도 99`% 의신뢰구간에속 하는자연수는 95, 96, 97,.c3, 145 의 51 개이다. 정규분포 N(m, sigma^2) 을따르는모집단에서크기가 n 인표본을임 의추출할때, 표본평균 X^- 의값 x^- 에대하여신뢰도 alpha`% 로모평균 m 을추정하면 sigma 1 신뢰구간은 m-x^- -<k~ rtn~ sigma 2 신뢰구간의길이는 2k~ ^(단, P( Z -<k)=alpha/10^) rtn~ 344 표본비율의분포 전략 표본비율 p^^ 이따르는정규분포 N^(p, pq ~Ò를구하고확률변수 n 를표준화한다. ( 단, q=1-p) 1600 명중에서국가공인자격증을가지고있는학생 의비율을 p^^ 이라하면표본의크기 1600 은충분히크므로 p^^ 의분포는근사적으로정규분포 N^(0.2, 0.2\0.8 Ò을따르고, 1600 p^^ &-0.2 p^^ &-0.2 Z= 4 0.2\0.8 = ~ 는근사적으로표준정규분포 0.01 v 1600 N(0, 1) 을따른다. 이때 P^(p^^ ->k/10^)=p(p^^->0.01\k) 이므로 =P^(Z-> 0.01\k-0.2 Ò 0.01 =0.5-P^(0-<Z-< 0.01\k-0.2 Ò 0.01 = 실전대비 Ⅲ 단원평가문제 53 해 (028~056)-04~06 강 ok.indd 오후 2:16

54 바른답 알찬 P(0-<Z-< 0.01\k-0.2 )= = 그런데 P(0-<Z-<2.4)= 이므로 0.01\k-0.2 = k=0.224.t3 k= 모비율의추정 전략 모비율 p 에대한신뢰도 95`% 의신뢰구간은 p^^ ~ p^^q^^ p^^q^^ ~-<p-<p^^ n n 임을이용한다. ( 단, q^^ =1-p^^ ) 표본비율 p^^ =75/30=0.25 이고표본의크기 75 가충분 히크므로모비율 p 에대한신뢰도 95`% 의신뢰구간은 \ & <p-< t <p-<0.299 b~-<p-< \0.75 b~ 바른답 알찬 해 (028~056) 일등확통 -04~06 강 ok.indd 오후 3:34

55 memo 해 (028~056) 일등확통 -04~06 강 ok.indd 오후 3:34

56 memo 해 (028~056) 일등확통 -04~06 강 ok.indd 오후 3:34

01

01 2019 학년도대학수학능력시험 9 월모의평가문제및정답 2019 학년도대학수학능력시험 9 월모의평가문제지 1 제 2 교시 5 지선다형 1. 두벡터, 모든성분의합은? [2 점 ] 에대하여벡터 의 3. 좌표공간의두점 A, B 에대하여선분 AB 를 로외분하는점의좌표가 일때, 의값은? [2점] 1 2 3 4 5 1 2 3 4 5 2. lim 의값은? [2점] 4. 두사건,

More information

고 학년도 9월고수학 1 전국연합학력평가영역문제지 1 1 제 2 교시 수학영역 5 지선다형 3. 두다항식, 에대하여 는? [ 점 ] 1. 의값은? ( 단, ) [ 점 ] 다항식 이 로인수분해될때, 의값은? ( 단,,

고 학년도 9월고수학 1 전국연합학력평가영역문제지 1 1 제 2 교시 수학영역 5 지선다형 3. 두다항식, 에대하여 는? [ 점 ] 1. 의값은? ( 단, ) [ 점 ] 다항식 이 로인수분해될때, 의값은? ( 단,, 고 208학년도 9월고수학 전국연합학력평가영역문제지 제 2 교시 수학영역 5 지선다형 3. 두다항식, 에대하여 는? [ 점 ]. 의값은? ( 단, ) [ 점 ] 2 3 2 3 4 5 4 5 2. 다항식 이 로인수분해될때, 의값은? ( 단,, 는상수이다.) [ 점 ] 4. 좌표평면위의두점 A, B 사이의거리가 일때, 양수 의값은? [ 점 ] 2 3 4 5 2

More information

제 2 교시 2019 학년도 3 월고 1 전국연합학력평가문제지수학영역 1 5 지선다형 1. 의값은? [2점] 일차방정식 의해는? [2 점 ] 두수, 의최대공약수는? [2 점 ] 일차함수 의그래프에서

제 2 교시 2019 학년도 3 월고 1 전국연합학력평가문제지수학영역 1 5 지선다형 1. 의값은? [2점] 일차방정식 의해는? [2 점 ] 두수, 의최대공약수는? [2 점 ] 일차함수 의그래프에서 제 2 교시 2019 학년도 3 월고 1 전국연합학력평가문제지 1 5 지선다형 1. 의값은? [2점] 1 2 3 4 5 3. 일차방정식 의해는? [2 점 ] 1 2 3 4 5 2. 두수, 의최대공약수는? [2 점 ] 1 2 3 4 5 4. 일차함수 의그래프에서 절편과 절편의합은? [3 점 ] 1 2 3 4 5 1 12 2 5. 함수 의그래프가두점, 를지날때,

More information

<30325FBCF6C7D05FB9AEC7D7C1F62E687770>

<30325FBCF6C7D05FB9AEC7D7C1F62E687770> 고1 2015학년도 9월고수학 1 전국연합학력평가영역문제지 1 1 제 2 교시 수학영역 1. 두복소수, 에대하여 의값은? ( 단, ) [2 점 ] 1 2 3 4 5 3. 좌표평면위의두점 P, Q 사이의거리는? [2 점 ] 1 2 3 4 5 2. 두다항식, 에대하여 를간단히하면? [2점] 4. 에서이차함수 의최댓값을, 최솟값을 이라할때, 의값은? [3점] 1

More information

31. 을전개한식에서 의계수는? 를전개한식이 일 때, 의값은? 을전개했을때, 의계수와상수항의합을구하면? 을전개했을때, 의 계수는? 를전개했을때, 상수항을 구하여라. 37

31. 을전개한식에서 의계수는? 를전개한식이 일 때, 의값은? 을전개했을때, 의계수와상수항의합을구하면? 을전개했을때, 의 계수는? 를전개했을때, 상수항을 구하여라. 37 21. 다음식의값이유리수가되도록유리수 의값을 정하면? 1 4 2 5 3 26. 을전개하면상수항을 제외한각항의계수의총합이 이다. 이때, 의값은? 1 2 3 4 5 22. 일때, 의값은? 1 2 3 4 5 27. 를전개하여간단히 하였을때, 의계수는? 1 2 3 4 5 23. 를전개하여 간단히하였을때, 상수항은? 1 2 3 4 5 28. 두자연수 와 를 로나누면나머지가각각

More information

스무살, 마음껏날아오르기위해, 일년만꾹참자! 2014학년도대학수학능력시험 9월모의평가 18번두이차정사각행렬 가 를만족시킬때, 옳은것만을 < 보기 > 에서있는대로고른것은? ( 단, 는단위행렬이다.) [4점] < 보기 > ㄱ. ㄴ. ㄷ. 2013학년도대학수학능력시험 16번

스무살, 마음껏날아오르기위해, 일년만꾹참자! 2014학년도대학수학능력시험 9월모의평가 18번두이차정사각행렬 가 를만족시킬때, 옳은것만을 < 보기 > 에서있는대로고른것은? ( 단, 는단위행렬이다.) [4점] < 보기 > ㄱ. ㄴ. ㄷ. 2013학년도대학수학능력시험 16번 친절한하영쌤의 수학 A형 약점체크집중공략오답률 Best 5 정복 하기! - 보충문제 행렬 2015학년도대학수학능력시험 9월모의평가 19번두이차정사각행렬 가 를만족시킬때, < 보기 > 에서옳은것만을있는대로고른것은? ( 단, 는단위행렬이고, 는영행렬이다.) [4점] < 보기 > ㄱ. 의역행렬이존재한다. ㄴ. ㄷ. 2015학년도대학수학능력시험 6월모의평가 19번두이차정사각행렬

More information

벡터(0.6)-----.hwp

벡터(0.6)-----.hwp 만점을위한 수학전문가남언우 - 벡터 1강 _ 분점의위치벡터 2강 _ 벡터의일차결합 3강 _ 벡터의연산 4강 _ 내적의도형적의미 5강 _ 좌표를잡아라 6강 _ 내적의활용 7강 _ 공간도형의방정식 8강 _ 구의방정식 9강 _2014년수능최고난도문제 좌표공간에 orbi.kr 1 강 _ 분점의위치벡터 01. 1) 두점 A B 이있다. 평면 에있는점 P 에대하여 PA

More information

완벽한개념정립 _ 행렬의참, 거짓 수학전문가 NAMU 선생 1. 행렬의참, 거짓개념정리 1. 교환법칙과관련한내용, 는항상성립하지만 는항상성립하지는않는다. < 참인명제 > (1),, (2) ( ) 인경우에는 가성립한다.,,, (3) 다음과같은관계식을만족하는두행렬 A,B에

완벽한개념정립 _ 행렬의참, 거짓 수학전문가 NAMU 선생 1. 행렬의참, 거짓개념정리 1. 교환법칙과관련한내용, 는항상성립하지만 는항상성립하지는않는다. < 참인명제 > (1),, (2) ( ) 인경우에는 가성립한다.,,, (3) 다음과같은관계식을만족하는두행렬 A,B에 1. 행렬의참, 거짓개념정리 1. 교환법칙과관련한내용, 는항상성립하지만 는항상성립하지는않는다. < 참인명제 > (1),, (2) ( ) 인경우에는 가성립한다.,,, (3) 다음과같은관계식을만족하는두행렬 A,B에대하여 AB=BA 1 가성립한다 2 3 (4) 이면 1 곱셈공식및변형공식성립 ± ± ( 복호동순 ), 2 지수법칙성립 (은자연수 ) < 거짓인명제 >

More information

이항정리 1. : 서로다른개에서순서를생각하지않고개를택하는것을개에서개를택하는이라한다. 의수 : 이의수를기호로로나타내며, 이의수는 P C ( 단, ) 참고 1. 순열은개에서개를뽑아서일렬로나열하는것이고, 은개에서개를뽑는것이다. (1) C 는 Combinat

이항정리 1. : 서로다른개에서순서를생각하지않고개를택하는것을개에서개를택하는이라한다. 의수 : 이의수를기호로로나타내며, 이의수는 P C ( 단, ) 참고 1. 순열은개에서개를뽑아서일렬로나열하는것이고, 은개에서개를뽑는것이다. (1) C 는 Combinat Ⅵ. 순열과 Map 01. 0 이항정리 - 1 - 01. 01. 0 이항정리 1. : 서로다른개에서순서를생각하지않고개를택하는것을개에서개를택하는이라한다. 의수 : 이의수를기호로로나타내며, 이의수는 P C ( 단, ) 참고 1. 순열은개에서개를뽑아서일렬로나열하는것이고, 은개에서개를뽑는것이다. (1) C 는 Combination( ) 의머리글자, (2) 은증명할때,

More information

필수예제 중복순열 02 같은 것이 있는 순열 6. 6. 모스 부호 ㆍ, - 를 사용하여 부호를 만들 때, ㆍ과 -에서 개를 뽑아 만들 수 있는 부호의 수를 필수예제 함수의 개수 7. 7. 두 집합 일 때, 다음을 (1) 에서 로의 함수의 개수 (2) 에서 로의 일대일함

필수예제 중복순열 02 같은 것이 있는 순열 6. 6. 모스 부호 ㆍ, - 를 사용하여 부호를 만들 때, ㆍ과 -에서 개를 뽑아 만들 수 있는 부호의 수를 필수예제 함수의 개수 7. 7. 두 집합 일 때, 다음을 (1) 에서 로의 함수의 개수 (2) 에서 로의 일대일함 01 중복순열 개념체크 중복순열을이용하는 정수 1-중복순열 (1) 중복순열 서로 다른 개에서 중복을 허락하여 개를 택하는 순열을 개에서 개를 택하는 중복순열이라 하고 기호 로 와 같이 나타낸다. (2) 중복순열의 수 개 (3) 중복순열의 계산법 에서 (4) 중복순열인 경우 은 받는 쪽 (고정 숫자) 는 주는 쪽 (선택 숫자) 으로 생각하자. 1 중복을 허락하는

More information

수리영역 5. 서로다른두개의주사위를동시에던져서나온두눈의수의곱 이짝수일때, 나온두눈의수의합이 또는 일확률은? 5) 의전개식에서상수항이존재하도록하는모든자 연수 의값의합은? 7) 다음순서도에서인쇄되는 의값은? 6) 8. 어떤특산

수리영역 5. 서로다른두개의주사위를동시에던져서나온두눈의수의곱 이짝수일때, 나온두눈의수의합이 또는 일확률은? 5) 의전개식에서상수항이존재하도록하는모든자 연수 의값의합은? 7) 다음순서도에서인쇄되는 의값은? 6) 8. 어떤특산 제 2 교시 2008 학년도 10 월고 3 전국연합학력평가문제지 수리영역 성명수험번호 3 1 먼저수험생이선택한응시유형의문제지인지확인하시오. 문제지에성명과수험번호를정확히기입하시오. 답안지에수험번호, 응시유형및답을표기할때는반드시 수험생이지켜야할일 에따라표기하시오. 단답형답의숫자에 0 이포함된경우, 0 을 OMR 답안지에반드시표기해야합니다. 문항에따라배점이다르니,

More information

<B1B9BEEE412E687770>

<B1B9BEEE412E687770> 201 학년도대학수학능력시험 6 월모의평가문제및정답 2016 학년도대학수학능력시험 6 월모의평가문제지 1 제 2 교시 5 지선다형 1. 두행렬 성분은? [2 점 ] 에대하여행렬 의 3. lim 의값은? [2점] 1 2 3 4 5 1 2 3 4 5 2. 의값은? [2점] 1 2 3 4 5 4. 공차가 인등차수열 에대하여 의값은? [3 점 ] 1 2 3 4 5

More information

일반각과호도법 l 삼각함수와미분 1. 일반각 시초선 OX 로부터원점 O 를중심으로 만큼회전이동한위치에동경 OP 가있을때, XOP 의크기를나타내는각들을 ( 은정수 ) 로나타내고 OP 의일반각이라한다. 2. 라디안 rad 반지름과같은길이의호에대한중심각의 크기를 라디안이라한

일반각과호도법 l 삼각함수와미분 1. 일반각 시초선 OX 로부터원점 O 를중심으로 만큼회전이동한위치에동경 OP 가있을때, XOP 의크기를나타내는각들을 ( 은정수 ) 로나타내고 OP 의일반각이라한다. 2. 라디안 rad 반지름과같은길이의호에대한중심각의 크기를 라디안이라한 일반각과호도법 l 1. 일반각 시초선 OX 로부터원점 O 를중심으로 만큼회전이동한위치에동경 OP 가있을때, XOP 의크기를나타내는각들을 ( 은정수 ) 로나타내고 OP 의일반각이라한다. 2. 라디안 rad 반지름과같은길이의호에대한중심각의 크기를 라디안이라한다. 3. 호도법과육십분법 라디안 라디안 4. 부채꼴의호의길이와넓이 반지를의길이가 인원에서중심각이 인 부채꼴의호의길이를

More information

6.6) 7.7) tan 8.8) 자연수 10.10) 부등식 두 의전개식에서 의계수는? ) 사건 에대하여 P P 일때, P 의값은? ( 단, 은 의여사건이다.) 일때, tan 의값은? log log 을만족시키

6.6) 7.7) tan 8.8) 자연수 10.10) 부등식 두 의전개식에서 의계수는? ) 사건 에대하여 P P 일때, P 의값은? ( 단, 은 의여사건이다.) 일때, tan 의값은? log log 을만족시키 1.1) 벡터 2.2) cos 함수 제 2 교시 2016 년 6 월고 3 모의고사문제지 성명수험번호 3 1 먼저수험생이선택한응시유형의문제지인지확인하시오. 문제지에성명과수험번호를정확히기입하시오. 답안지에수험번호, 응시유형및답을표기할때는반드시 수험생이지켜야할일 에따라표기하시오. 단답형답의숫자에 0 이포함된경우, 0 을 OMR 답안지에반드시표기해야합니다. 문항에따라배점이다르니,

More information

문항코드 EBS 수능완성수학영역수학 1 A 형 주어진그래프의꼭짓점에 를그림과같이 정하고꼭짓점사이의연결관계를행렬로나타내면다 음과같다. ( 나 ) 세수, 12, 는이순서대로등비수열을이룬다. 의값은? 문

문항코드 EBS 수능완성수학영역수학 1 A 형 주어진그래프의꼭짓점에 를그림과같이 정하고꼭짓점사이의연결관계를행렬로나타내면다 음과같다. ( 나 ) 세수, 12, 는이순서대로등비수열을이룬다. 의값은? 문 곽정원의수능필수아이템! 2,3 점은다내꺼 + 4 점도전 ~ 실전모의고사 1. 두행렬 의모든성분의합은? 1 9 2 10 3 11 4 12 5 13 배점 2 문항코드 3-182-365 기 따라서행렬 의모든성분의합은 7+(-4)+4+5=12 2. log l 의값은? 에대하여행렬 3. lim 의값은? 1 2 3 1 4 2 5 4 배점 2 문항코드 3-179-239

More information

최종 고등수학 하.hwp

최종 고등수학 하.hwp 철/벽/수/학 고등수학 (하) 제1부 평면좌표 1 ST 철벽 CONCEPT 01 두점사이의거리 q 수직선위의두점사이의거리 수직선위의두점 A, B 사이의거리는 AB w 좌표평면위의두점사이의거리좌표평면위의두점 A, B 사이의거리는 AB Q❶-1 다음두점사이의거리를구하여라. 풀이 ⑴ A, B ⑵ A, B ⑶ A B ⑷ A B 2 배상면쌤 ^ ^ Q❶-2 다음을만족하는

More information

<BCF6B8AEBFB5BFAA28B0A1C7FC295FC2A6BCF62E687770>

<BCF6B8AEBFB5BFAA28B0A1C7FC295FC2A6BCF62E687770> 제 2 교시 2013 학년도대학수학능력시험문제지 수리영역 ( 가형 ) 1 짝수형 5 지선다형 1. 두행렬, 모든성분의합은? [2 점 ] 에대하여행렬 의 3. 좌표공간에서두점 A, B 에대하여선분 AB 를 로내분하는점의좌표가 이다. 의값은? [2점] 1 2 3 4 5 1 2 3 4 5 2. sin 일때, sin 의값은? ( 단, 이다.) [2 점 ] 1 2 3

More information

곡선 7.7. 오른쪽그림과같이반지름의길이가각각 이고중심이같은세원으로이루어진과녁에총을쏠때, 색칠한부분을맞힐확률은? ( 단, 총알은과녁을벗어나지않고, 경계선에맞지않는다.) [3점] [PP 난이도중 ] [PP 18 문

곡선 7.7. 오른쪽그림과같이반지름의길이가각각 이고중심이같은세원으로이루어진과녁에총을쏠때, 색칠한부분을맞힐확률은? ( 단, 총알은과녁을벗어나지않고, 경계선에맞지않는다.) [3점] [PP  난이도중 ] [PP 18 문 등차수열 함수 2017 학년도수능대비 9 월모의고사 FINAL 1 회 ( 나형 ) 제 2 교시 1 1. lim 의값은? 1 2 [PP 07 0006@ 문과 @ 고 3@ 수열의극한 @ 난이도하 ] 3 [2 점 ] 4.4. [PP 05 0010@ 문과 @ 고 3@ 수열 @ 난이도중 ] 에대하여 일때, 의값은? [3점] 1 2 3 4 5 4 5 [PP 08 0007@

More information

함수레시피 1. 케이스분류의 3 대원칙 2. 사건과여사건 3. 확률과경우의수의중대한차이점 - E. T -

함수레시피 1. 케이스분류의 3 대원칙 2. 사건과여사건 3. 확률과경우의수의중대한차이점 - E. T - E.T s Eight Technics. Ver. 2019 Second Technic. 경우나누기 확률과경우의수단원은수학중유일하게 논리보다손이더먼저나가야하는단원이다. - E. T - 함수레시피 1. 케이스분류의 3 대원칙 2. 사건과여사건 3. 확률과경우의수의중대한차이점 - E. T - Second Technic. 경우의수 / 확률 E.T s Eight Technics.

More information

2019 학년도대학수학능력시험문제및정답

2019 학년도대학수학능력시험문제및정답 2019 학년도대학수학능력시험문제및정답 2019 학년도대학수학능력시험문제지 1 제 2 교시 홀수형 5 지선다형 1. 두벡터, 에대하여 벡터 의모든성분의합은? [2 점 ] 3. 좌표공간의두점 A, B 에대하여선분 AB 를 로내분하는점이 축위에있을때, 의값은? [2점] 1 2 3 4 5 1 2 3 4 5 2. lim 의값은? [2점] ln 4. 두사건, 에대하여

More information

<B1B9BEEE412E687770>

<B1B9BEEE412E687770> 2015 학년도대학수학능력시험문제및정답 2015 학년도대학수학능력시험문제지 1 제 2 교시 홀수형 5 지선다형 1. 의값은? [2점] 3. lim 의값은? [2점] 1 2 3 4 5 1 2 3 4 5 2. 두행렬 성분의합은? [2 점 ], 에대하여행렬 의모든 4. 다음그래프의각꼭짓점사이의연결관계를나타내는행렬의성분중 의개수는? [3점] 1 2 3 4 5 1 2

More information

7) 다음의 다음 9) 남학생과 9. zb 여학생 각각 명이 갖고 있는 여름 티 셔츠의 개수를 조사하여 꺾은선그래프로 나타낸 것 이다. 이 두 그래프의 설명으로 옳지 않은 것은? ㄱ. ㄴ. 회째의 수학 점수는 점이다. 수학 점수의 분산은 이다. ㄷ. 영어점수가 수학 점

7) 다음의 다음 9) 남학생과 9. zb 여학생 각각 명이 갖고 있는 여름 티 셔츠의 개수를 조사하여 꺾은선그래프로 나타낸 것 이다. 이 두 그래프의 설명으로 옳지 않은 것은? ㄱ. ㄴ. 회째의 수학 점수는 점이다. 수학 점수의 분산은 이다. ㄷ. 영어점수가 수학 점 1) 은경이네 2) 어느 3) 다음은 자연수 그림은 6) 학생 학년 고사종류 과목 과목코드번호 성명 3 2012 2학기 중간고사 대비 수학 201 대청중 콘텐츠산업 진흥법 시행령 제33조에 의한 표시 1) 제작연월일 : 2012-08-27 2) 제작자 : 교육지대 3) 이 콘텐츠는 콘텐츠산업 진흥법 에 따라 최초 제작일부터 년간 보호됩니다. 콘텐츠산업 진흥법

More information

PSFZWLOTGJYU.hwp

PSFZWLOTGJYU.hwp 학년도대수능 9 월모의평가 ( 수리영역 - 가형 AH AT sin 8. log 9 log. log log 일때, ( 분모 ( 분자 이어야한다. 즉, ( +a-b+a-b a - b - ᄀ +a+b - (-(-b (-( ++ -b + + - b -b 9 ᄂ ᄀ, ᄂ에서 a, b 8 a+ b 5. log log X AB -B ( ( - - ( - ( 5 - -8

More information

제 5 일 년 3월교육청 년 6월평가원 년 9월평가원 년 11월교육청 년경찰대 년 3월교육청 년 6월평가원 년경찰대 년수능 년 10월교육청

제 5 일 년 3월교육청 년 6월평가원 년 9월평가원 년 11월교육청 년경찰대 년 3월교육청 년 6월평가원 년경찰대 년수능 년 10월교육청 제 5 일 1. 2009년 3월교육청 2. 2014년 6월평가원 3. 2016년 9월평가원 4. 2015년 11월교육청 5. 2013년경찰대 6. 2007년 3월교육청 7. 2009년 6월평가원 8. 2011년경찰대 9. 2006년수능 10. 2006년 10월교육청 1. 수열 이, 일때, 옳은것만을 [ 보기 ] 에서있는대로고른것은? ( 단, 는 0이아닌실수이다.)

More information

문제지 제시문 2 보이지 않는 영역에 대한 정보를 얻기 위하여 관측된 다른 정보를 분석하여 역으로 미 관측 영역 에 대한 정보를 얻을 수 있다. 가령 주어진 영역에 장애물이 있는 경우 한 끝 점에서 출발하여 다른 끝 점에 도달하는 최단 경로의 개수를 분석하여 장애물의

문제지 제시문 2 보이지 않는 영역에 대한 정보를 얻기 위하여 관측된 다른 정보를 분석하여 역으로 미 관측 영역 에 대한 정보를 얻을 수 있다. 가령 주어진 영역에 장애물이 있는 경우 한 끝 점에서 출발하여 다른 끝 점에 도달하는 최단 경로의 개수를 분석하여 장애물의 제시문 문제지 2015학년도 대학 신입학생 수시모집 일반전형 면접 및 구술고사 수학 제시문 1 하나의 동전을 던질 때, 앞면이나 뒷면이 나온다. 번째 던지기 전까지 뒷면이 나온 횟수를 라 하자( ). 처음 던지기 전 가진 점수를 점이라 하고, 번째 던졌을 때, 동전의 뒷면이 나오면 가지고 있던 점수를 그대로 두고, 동전의 앞면이 나오면 가지고 있던 점수를 배

More information

<B4EBC7D0BCF6C7D02DBBEFB0A2C7D4BCF62E687770>

<B4EBC7D0BCF6C7D02DBBEFB0A2C7D4BCF62E687770> 삼각함수. 삼각함수의덧셈정리 삼각함수의덧셈정리 삼각함수 sin (α + β ), cos (α + β ), tan (α + β ) 등을 α 또는 β 의삼각함수로나 타낼수있다. 각 α 와각 β 에대하여 α >0, β >0이고 0 α - β < β 를만족한다고가정하 자. 다른경우에도같은방법으로증명할수있다. 각 α 와각 β 에대하여 θ = α - β 라고놓자. 위의그림에서원점에서거리가

More information

mathna_hsj.hwp

mathna_hsj.hwp 2008 학년도 6 월모의평가 ( 수리영역 - 가형 ) 정답및해설 1. 4 4 4. 2. 로놓으면 ᄀ - ᄂ 양변을제곱하면 3. 5 따라서 방정식ᄀ의근은이다. 일때 ( 분모 ) ( 분자 ) 이어야한다. 따라서 따라서 두식ᄀ ᄂ을동시에만족하는실수의값은구하는합은 ( 준식 ) 5 5. 는최고차항의계수가 1인삼차함수 로놓으면 - 1 - 따라서 ㄷ. 3 < 다른풀이

More information

01 경우의수

01 경우의수 제 1 장 순열과조합 01 경우의수 1. 경우의수 01 순열과조합 빠짐없이, 중복되지않게 사전식배열, 수형도 복잡한경우의수를셀때는점화식을이용하는경우도있다. (1) 합의법칙한사건 가 가지의방법으로일어나고, 다른사건 가 가지의방법으로일어난다고할때 또는 가일어나는경우의수는, 가동시에일어나지않을때 m+n 가지, 가동시에일어나는경우가 가지있을때 m+n-l 가지 (2)

More information

7.7) 정의역이 8.8) 연속확률변수 10.10) 원점을 좌표평면에서 인함수 의그래프가그림 과같다. 9.9 ) 함수 의그래프와함수 의 그래프가만나는점을 라할때, 옳은것만을 < 보기 > 에서있는대로고른것은? lim lim 의값은? < 보기 > ㄱ. ㄴ

7.7) 정의역이 8.8) 연속확률변수 10.10) 원점을 좌표평면에서 인함수 의그래프가그림 과같다. 9.9 ) 함수 의그래프와함수 의 그래프가만나는점을 라할때, 옳은것만을 < 보기 > 에서있는대로고른것은? lim lim 의값은? < 보기 > ㄱ. ㄴ 1.1) 2.2) 두 두 로그부등식 제 2 교시 2012 년 5 월고 2 모의평가문제지 성명수험번호 3 1 먼저수험생이선택한응시유형의문제지인지확인하시오. 문제지에성명과수험번호를정확히기입하시오. 답안지에수험번호, 응시유형및답을표기할때는반드시 수험생이지켜야할일 에따라표기하시오. 단답형답의숫자에 0 이포함된경우, 0 을 OMR 답안지에반드시표기해야합니다. 문항에따라배점이다르니,

More information

집합 집합 오른쪽 l 3. (1) 집합 X 의각원소에대응하는집합 Y 의원소가단하나만인대응을 라할때, 이대응 를 X 에서 Y 로의라고하고이것을기호로 X Y 와같이나타낸다. (2) 정의역과공역정의역 : X Y 에서집합 X, 공역 : X Y 에서집합 Y (3) 의개수 X Y

집합 집합 오른쪽 l 3. (1) 집합 X 의각원소에대응하는집합 Y 의원소가단하나만인대응을 라할때, 이대응 를 X 에서 Y 로의라고하고이것을기호로 X Y 와같이나타낸다. (2) 정의역과공역정의역 : X Y 에서집합 X, 공역 : X Y 에서집합 Y (3) 의개수 X Y 어떤 다음 X 대응 1. 대응 (1) 어떤주어진관계에의하여집합 X 의원소에집합 Y 의원소를짝지어주는것을집합 X 에서집합 Y 로의대응이라고한다. l (2) 집합 X 의원소 에집합 Y 의원소 가짝지어지면 에 가대응한다고하며이것을기호로 와같이나타낸다. 2. 일대일대응 (1) 집합 A 의모든원소와집합 B 의모든원소가하나도빠짐없이꼭한개씩서로대응되는것을집합 A 에서집합

More information

2020 학년도랑데뷰실전모의고사문제지 - 시즌 3 제 1 회 제 2 교시 수학영역 ( 나형 ) 1 5 지선다형 3. 그림은함수 를나타낸것이다 학년도 9월모의평가나형과싱크로율 99% 학년도수학영역대비랑데뷰실전모의고사가형-시즌1~ 시즌6, 나형-시즌

2020 학년도랑데뷰실전모의고사문제지 - 시즌 3 제 1 회 제 2 교시 수학영역 ( 나형 ) 1 5 지선다형 3. 그림은함수 를나타낸것이다 학년도 9월모의평가나형과싱크로율 99% 학년도수학영역대비랑데뷰실전모의고사가형-시즌1~ 시즌6, 나형-시즌 2020 학년도랑데뷰실전모의고사문제지 - 시즌 3 제 1 회 제 2 교시 1 5 지선다형 3. 그림은함수 를나타낸것이다. - 2020학년도 9월모의평가나형과싱크로율 99% - 2020학년도수학영역대비랑데뷰실전모의고사가형-시즌1~ 시즌6, 나형-시즌1~ 시즌2 ( 각시즌 4회분 ) 오르비전자책에서구매가능 - 오타, 오류수정파일은랑데뷰수학카페자료실에서무료다운로드가능

More information

1 peaieslvfp3 1. 두점사이의거리 수직선위의두점사이의거리를구할수있다. 좌표평면위의두점사이의거리를구할수있다. 수직선위의두점사이의거리 todrkrgo qhqtlek 오른쪽그림은충무로역을중심으로한서울시지하철 3`호선노선도의일부분이다. 충무로역을` 0, 을지로 3`

1 peaieslvfp3 1. 두점사이의거리 수직선위의두점사이의거리를구할수있다. 좌표평면위의두점사이의거리를구할수있다. 수직선위의두점사이의거리 todrkrgo qhqtlek 오른쪽그림은충무로역을중심으로한서울시지하철 3`호선노선도의일부분이다. 충무로역을` 0, 을지로 3` peaieslvfp. 두점사이의거리 수직선위의두점사이의거리를구할수있다. 좌표평면위의두점사이의거리를구할수있다. 수직선위의두점사이의거리 todrkrgo qhqtlek 오른쪽그림은충무로역을중심으로한서울시지하철 `호선노선도의일부분이다. 충무로역을` 0, 을지로 `가역을 ``로나타낼때, 다음물음에답하여라. 독립문 경복궁 안국종로 가을지로 가충무로동대입구약수금호옥수압구정잠원신사

More information

5. 두함수 log 에대하여옳은것을 < 보기 > 에서모두고르면?5 ) ㄱ. ㄴ. ㄷ. < 보기 > 1 ㄴ 2 ㄷ 3 ㄱ, ㄴ 4 ㄴ, ㄷ 5 ㄱ, ㄴ, ㄷ 7. 인실수 에대하여 log 의지표를 이라할때, 옳 은것을보기에서모두고르면? ( 단, 는 를넘지않는최대의정수이다.

5. 두함수 log 에대하여옳은것을 < 보기 > 에서모두고르면?5 ) ㄱ. ㄴ. ㄷ. < 보기 > 1 ㄴ 2 ㄷ 3 ㄱ, ㄴ 4 ㄴ, ㄷ 5 ㄱ, ㄴ, ㄷ 7. 인실수 에대하여 log 의지표를 이라할때, 옳 은것을보기에서모두고르면? ( 단, 는 를넘지않는최대의정수이다. 제 2 교시 2008 년 5 월고 3 모의고사문제지 성명수험번호 3 1 먼저수험생이선택한응시유형의문제지인지확인하시오. 문제지에성명과수험번호를정확히기입하시오. 답안지에수험번호, 응시유형및답을표기할때는반드시 수험생이지켜야할일 에따라표기하시오. 단답형답의숫자에 0 이포함된경우, 0 을 OMR 답안지에반드시표기해야합니다. 문항에따라배점이다르니, 각물음의끝에표시된배점을참고하시오.

More information

7. 다음그림과같이한변의길이 가 4 6 인마름모의넓이를구 하여라. 10. 다음그림과같이모선의길이가 6 cm 인원뿔의밑면의 둘레의길이가 6π cm 일때, 원뿔의높이와부피를구한 것은? 1 6 cm, 6 π cm 6 cm, 6π cm 8. 다음과같이한변의길이가 8 인정육 면

7. 다음그림과같이한변의길이 가 4 6 인마름모의넓이를구 하여라. 10. 다음그림과같이모선의길이가 6 cm 인원뿔의밑면의 둘레의길이가 6π cm 일때, 원뿔의높이와부피를구한 것은? 1 6 cm, 6 π cm 6 cm, 6π cm 8. 다음과같이한변의길이가 8 인정육 면 . 단원테스트 범위 : 피타고라스의정리 피타고라스의정리의활용 50 문항 / 저반 : 이름 : 출제자 : 박지연. 1. 다음그림에서 x 의값으로적절한것은? 4. 세변의길이가 6 cm, 5 cm, 10 cm 인삼각형은어떤삼 각형인가? 1 직각삼각형 이등변삼각형 직각이등변삼각형 4 예각삼각형 5 둔각삼각형 1 9 9 9 4 4 9 5 5 9. 삼각형의세변의길이가다음보기와같을때직각삼각

More information

1.1) 등비수열 전체집합 제 2 교시 나 형 2016 년 3 월고 3 모의고사문제지 수리영역 성명수험번호 3 1 먼저수험생이선택한응시유형의문제지인지확인하시오. 문제지에성명과수험번호를정확히기입하시오. 답안지에수험번호, 응시유형및답을표기할때는반드시 수험생이지켜야할일 에따

1.1) 등비수열 전체집합 제 2 교시 나 형 2016 년 3 월고 3 모의고사문제지 수리영역 성명수험번호 3 1 먼저수험생이선택한응시유형의문제지인지확인하시오. 문제지에성명과수험번호를정확히기입하시오. 답안지에수험번호, 응시유형및답을표기할때는반드시 수험생이지켜야할일 에따 1.1) 등비수열 전체집합 제 2 교시 2016 년 3 월고 3 모의고사문제지 성명수험번호 3 1 먼저수험생이선택한응시유형의문제지인지확인하시오. 문제지에성명과수험번호를정확히기입하시오. 답안지에수험번호, 응시유형및답을표기할때는반드시 수험생이지켜야할일 에따라표기하시오. 단답형답의숫자에 0 이포함된경우, 0 을 OMR 답안지에반드시표기해야합니다. 문항에따라배점이다르니,

More information

2018 학년도대학수학능력시험문제지 1 제 2 교시 홀수형 5 지선다형 1. 두벡터, 모든성분의합은? [2 점 ] 에대하여벡터 의 3. 좌표공간의두점 A, B 에대하여선분 AB 를 으로내분하는점의좌표가 이다. 의값은? [2점] ln

2018 학년도대학수학능력시험문제지 1 제 2 교시 홀수형 5 지선다형 1. 두벡터, 모든성분의합은? [2 점 ] 에대하여벡터 의 3. 좌표공간의두점 A, B 에대하여선분 AB 를 으로내분하는점의좌표가 이다. 의값은? [2점] ln 2018 학년도대학수학능력시험문제및정답 2018 학년도대학수학능력시험문제지 1 제 2 교시 홀수형 5 지선다형 1. 두벡터, 모든성분의합은? [2 점 ] 에대하여벡터 의 3. 좌표공간의두점 A, B 에대하여선분 AB 를 으로내분하는점의좌표가 이다. 의값은? [2점] 1 2 3 4 5 1 2 3 4 5 ln 2. lim 의값은? [2점] 4. 두사건 와 는서로독립이고

More information

5.5) cos 6.6) 두 coscos 일때, sinsin 의값은? [3점] ) 일때, 방정식 의모든해의합은? [3 점 ] 1 4 sin cos 의값은? [3점] 1 ln 2 ln 3 ln 4 ln 5 ln 8.8 ) 벡터 에대하여

5.5) cos 6.6) 두 coscos 일때, sinsin 의값은? [3점] ) 일때, 방정식 의모든해의합은? [3 점 ] 1 4 sin cos 의값은? [3점] 1 ln 2 ln 3 ln 4 ln 5 ln 8.8 ) 벡터 에대하여 1.1) 두 2.2) 방정식 좌표공간에서 두 제 2 교시 2016 년 9 월고 3 모의고사문제지 성명수험번호 3 1 먼저수험생이선택한응시유형의문제지인지확인하시오. 문제지에성명과수험번호를정확히기입하시오. 답안지에수험번호, 응시유형및답을표기할때는반드시 수험생이지켜야할일 에따라표기하시오. 단답형답의숫자에 0 이포함된경우, 0 을 OMR 답안지에반드시표기해야합니다.

More information

7. 인실수 에대하여 log 의지표를 이라할때, 옳 은것을보기에서모두고르면? ( 단, 는 를넘지않는최대의정수이다.) 7 ) ㄱ. log ㄴ. log 의지표는 이다. ㄷ. log log 이면 은 자리의정수 이다. 10. 다음은어느인터넷사이트의지도상단에있는버튼의기능을설명한

7. 인실수 에대하여 log 의지표를 이라할때, 옳 은것을보기에서모두고르면? ( 단, 는 를넘지않는최대의정수이다.) 7 ) ㄱ. log ㄴ. log 의지표는 이다. ㄷ. log log 이면 은 자리의정수 이다. 10. 다음은어느인터넷사이트의지도상단에있는버튼의기능을설명한 제 2 교시 2008 년 5 월고 3 모의고사문제지 성명수험번호 3 1 먼저수험생이선택한응시유형의문제지인지확인하시오. 문제지에성명과수험번호를정확히기입하시오. 답안지에수험번호, 응시유형및답을표기할때는반드시 수험생이지켜야할일 에따라표기하시오. 단답형답의숫자에 0 이포함된경우, 0 을 OMR 답안지에반드시표기해야합니다. 문항에따라배점이다르니, 각물음의끝에표시된배점을참고하시오.

More information

2017 학년도대학수학능력시험문제지 1 제 2 교시 홀수형 5 지선다형 3. sin 의값은? [2점] 1. 두벡터, 모든성분의합은? [2 점 ] 에대하여벡터 의 lim 의값은? [2점] ln 두사건 와 는

2017 학년도대학수학능력시험문제지 1 제 2 교시 홀수형 5 지선다형 3. sin 의값은? [2점] 1. 두벡터, 모든성분의합은? [2 점 ] 에대하여벡터 의 lim 의값은? [2점] ln 두사건 와 는 2017 학년도대학수학능력시험문제및정답 2017 학년도대학수학능력시험문제지 1 제 2 교시 홀수형 5 지선다형 3. sin 의값은? [2점] 1. 두벡터, 모든성분의합은? [2 점 ] 에대하여벡터 의 1 2 3 4 5 1 2 3 4 5 2. lim 의값은? [2점] ln 1 2 3 4 5 4. 두사건 와 는서로독립이고 P P 일때, PP 의값은? ( 단, 은

More information

5. 정적분 의값과반지름의길이가 인원의넓 이가같을때, 의값은? 7. 곡선 ln 와 축및 축으로둘러싸인도형의넓이 가 일때, 상수 의값은? ( 단, ) 에서정의된함수 의 그래프가오른쪽그림과같을때, 정적분 의값을구하면? 8. 함수 의

5. 정적분 의값과반지름의길이가 인원의넓 이가같을때, 의값은? 7. 곡선 ln 와 축및 축으로둘러싸인도형의넓이 가 일때, 상수 의값은? ( 단, ) 에서정의된함수 의 그래프가오른쪽그림과같을때, 정적분 의값을구하면? 8. 함수 의 1. lim sin 의값은? 3. 함수 cos cos ( ) 는 에서극솟값 를갖는다. 이때 의값은? 1 2 3 1 2 3 4 5 4 5 2. 아래쪽그림과같이중심이 C 이고반지름의길이가 인원이있다. 직선 가원점 O 를지나고기울기가양수인직선 과만나는점을 P 축과만나는점을 Q 라하고, 직선 이원과만나는원점이아닌점을 R 라하자. 직선 이 축의양의방향과이루는각의크기를

More information

경우의수 합의법칙과곱의법칙을이해하고, 이를 이용하여경우의수를구할수있다. 합의법칙 어느식당에는후식으로 컵케이크 가지와아이스크림 가지 한개의주사위를던질때, 다음을구하시오. ⑴ 짝수의눈이나오는경우의수 ⑵ 의배수의눈이나오거나 의약수의눈이나오는경우의수 가준비되어있다. 컵케이크또

경우의수 합의법칙과곱의법칙을이해하고, 이를 이용하여경우의수를구할수있다. 합의법칙 어느식당에는후식으로 컵케이크 가지와아이스크림 가지 한개의주사위를던질때, 다음을구하시오. ⑴ 짝수의눈이나오는경우의수 ⑵ 의배수의눈이나오거나 의약수의눈이나오는경우의수 가준비되어있다. 컵케이크또 야곱베르누이 (Bernoulli, J., 1654~1705) 스위스의수학자 이글은 1713 년에출간된확률론에관한저서 추측술 에서, 어떤일이일어날가 능성을정확하게예측하기위해서는가능한경우의수를아는것이중요함을강조한 것이다. 260 Ⅵ. 경우의수 경우의수 합의법칙과곱의법칙을이해하고, 이를 이용하여경우의수를구할수있다. 합의법칙 어느식당에는후식으로 컵케이크 가지와아이스크림

More information

두산동아-확통 완성본.hwp

두산동아-확통 완성본.hwp 1단원순열 ( 경우의수) 지윤이의필통에는서로다른볼펜 자루와서로다른 연필 자루가있다. 이때지윤이가필기구한자루를택하 는경우의수를구하여라. 1) 어느 분식점에서 는 오른쪽 차림표와 같이김밥 가지, 면 가지, 덮밥 가지를 판매하고있다. 이중 에서주문할음식한 가지를택하는경우의 수를구하여라. 2) 한개의주사위를두번던져나온눈의수를차례로 각각, 라고할때, 을만족하는, 의순

More information

3.2 함수의정의 Theorem 6 함수 f : X Y 와 Y W 인집합 W 에대하여 f : X W 는함수이다. Proof. f : X Y 가함수이므로 f X Y 이고, Y W 이므로 f X W 이므로 F0이만족된다. 함수의정의 F1, F2은 f : X Y 가함수이므로

3.2 함수의정의 Theorem 6 함수 f : X Y 와 Y W 인집합 W 에대하여 f : X W 는함수이다. Proof. f : X Y 가함수이므로 f X Y 이고, Y W 이므로 f X W 이므로 F0이만족된다. 함수의정의 F1, F2은 f : X Y 가함수이므로 3.2 함수의정의 Theorem 6 함수 f : X Y 와 Y W 인집합 W 에대하여 f : X W 는함수이다. Proof. f : X Y 가함수이므로 f X Y 이고, Y W 이므로 f X W 이므로 F0이만족된다. 함수의정의 F1, F2은 f : X Y 가함수이므로성립한다. Theorem 7 두함수 f : X Y 와 g : X Y 에대하여, f = g f(x)

More information

01. 순열 1. 경우의수 (1) 합의법칙두사건 와 가동시에일어나지않을때, 사건 가일어나는경우의수가, 사건 가일어나는경우의수가 이라하면사건 또는 가일어나는경우의수는 이다. 집합의개념을이용하여합의법칙을생각해보자. 두사건 가일어나는경우의집합을각각 라하면두사건 가일어나는경우

01. 순열 1. 경우의수 (1) 합의법칙두사건 와 가동시에일어나지않을때, 사건 가일어나는경우의수가, 사건 가일어나는경우의수가 이라하면사건 또는 가일어나는경우의수는 이다. 집합의개념을이용하여합의법칙을생각해보자. 두사건 가일어나는경우의집합을각각 라하면두사건 가일어나는경우 경우의수순열조합 01. 순열 1. 경우의수 (1) 합의법칙두사건 와 가동시에일어나지않을때, 사건 가일어나는경우의수가, 사건 가일어나는경우의수가 이라하면사건 또는 가일어나는경우의수는 이다. 집합의개념을이용하여합의법칙을생각해보자. 두사건 가일어나는경우의집합을각각 라하면두사건 가일어나는경우의수는각각 와같다. 또사건 또는사건 가일어나는경우는집합 로나타낼수있고, 두사건

More information

math_hsj_kK5LqN33.pdf.hwp

math_hsj_kK5LqN33.pdf.hwp 2016 학년도 3 월고 1 전국연합학력평가정답및해설 수학영역 정답 1 1 2 3 3 4 4 3 5 5 6 3 7 2 8 5 9 1 10 5 11 2 12 2 13 5 14 4 15 2 16 1 17 4 18 2 19 4 20 3 21 1 22 23 24 25 26 27 28 29 30 해설 1. [ 출제의도 ] 거듭제곱의뜻을알고식의값을계산한다. 2. [ 출제의도

More information

01. 확률의정의와기본성질 1. 확률의정의 (1) 시행과사건같은조건에서몇번이고반복할수있으며그결과가우연에의해서정해지는실험이나관찰을시행이라고하고, 시행의결과로일어나는것을사건이라고한다. 어떤시행에서일어날수있는모든결과의집합을그시행에대한표본공간이라하고, 사건은표본공간의부분집합이

01. 확률의정의와기본성질 1. 확률의정의 (1) 시행과사건같은조건에서몇번이고반복할수있으며그결과가우연에의해서정해지는실험이나관찰을시행이라고하고, 시행의결과로일어나는것을사건이라고한다. 어떤시행에서일어날수있는모든결과의집합을그시행에대한표본공간이라하고, 사건은표본공간의부분집합이 확률과조건부확률 01. 확률의정의와기본성질 1. 확률의정의 (1) 시행과사건같은조건에서몇번이고반복할수있으며그결과가우연에의해서정해지는실험이나관찰을시행이라고하고, 시행의결과로일어나는것을사건이라고한다. 어떤시행에서일어날수있는모든결과의집합을그시행에대한표본공간이라하고, 사건은표본공간의부분집합이다. 표본공간 의부분집합중에서한개의원소로이루어진사건을근원사건이라고한다. (2)

More information

기본도형과작도 1 강 - 연습문제 1. 오른쪽그림과같이직선l 위에점,, 가있을때, 옳지않은것은? 1 = 2 = 3 = 직선l 4 = 5 = l 2. 오른쪽그림에서 = = 이다. 다음( ) 안에알맞은수를쓰시오. 1 =( 2 =( 3 =( 4 =( ) ) ) ) 3. 한평

기본도형과작도 1 강 - 연습문제 1. 오른쪽그림과같이직선l 위에점,, 가있을때, 옳지않은것은? 1 = 2 = 3 = 직선l 4 = 5 = l 2. 오른쪽그림에서 = = 이다. 다음( ) 안에알맞은수를쓰시오. 1 =( 2 =( 3 =( 4 =( ) ) ) ) 3. 한평 기본도형과작도 1 강 - 점, 선, 면 사이버스쿨우프선생 www.cyberschool.co.kr 도형의기본요소 1. 점 : 크기가없다. 0 차원, 있는것처럼점을찍는다. 2. 선 : 점이움직인자취( 흔적), 1차원 3. 면 : 선이움직인자취, 2차원 교점 : ( 선 + 선), ( 선 + 면) 이만나는점 교선 : ( 면 + 면) 이만나는선 [ 예제 1] 삼각뿔에서교점과교선의수는?

More information

Ⅰ. 순열과조합 2 1. 경우의수 개념 001 합의법칙과곱의법칙 (1) 합의법칙 1 두사건 와 가일어나는경우의수가각각 과 이고, 와 가동시에일어나지않을때, 또는 가일어나는경우의수는 이다. 이것을합의법칙이라고한다. 2 합의법칙은세개이상의사건에대하여도성립한다. 3 보기 학

Ⅰ. 순열과조합 2 1. 경우의수 개념 001 합의법칙과곱의법칙 (1) 합의법칙 1 두사건 와 가일어나는경우의수가각각 과 이고, 와 가동시에일어나지않을때, 또는 가일어나는경우의수는 이다. 이것을합의법칙이라고한다. 2 합의법칙은세개이상의사건에대하여도성립한다. 3 보기 학 Ⅰ. 순열과조합 1. 경우의수 2. 순열 3. 중복순열 4. 같은것이있는순열 5. 원순열 6. 조합 7. 중복조합 8. 자연수의분할 9. 집합의분할과분할, 분배 10. 이항정리 Ⅰ. 순열과조합 2 1. 경우의수 개념 001 합의법칙과곱의법칙 (1) 합의법칙 1 두사건 와 가일어나는경우의수가각각 과 이고, 와 가동시에일어나지않을때, 또는 가일어나는경우의수는 이다.

More information

두산동아-확통 완성본.hwp

두산동아-확통 완성본.hwp 5단원조건부확률 예제 1 1. 조건부확률 부터 까지의자연수가각각적힌공이들 어있는주머니에서임의로공한개를꺼내려고한 다. 꺼낸공이홀수가적힌공일때, 그것이소수일 확률을구하여라. 179) 문제 2 어느회사에서직원들이출근할때이용하는 교통수단을조사하였더니대중교통을이용하는직원 이전체의 % 이고, 대중교통을이용하는남자직원 은전체의 % 이었다. 이회사직원중에서임의로 뽑은한명이대중교통을이용할때,

More information

01 경우의 수 1 사건과 경우의 수 ① 사건 : 같은 조건에서 여러 번 반복할 수 있는 실험이나 관찰에 의하여 나타 나는 결과 바빠 꿀팁! ② 경우의 수 : 어떤 사건이 일어나는 가짓수 실험 동전을 던진 다. 사건 경우 일어날 수 있는 모든 경우 백 앞면이 나온다.

01 경우의 수 1 사건과 경우의 수 ① 사건 : 같은 조건에서 여러 번 반복할 수 있는 실험이나 관찰에 의하여 나타 나는 결과 바빠 꿀팁! ② 경우의 수 : 어떤 사건이 일어나는 가짓수 실험 동전을 던진 다. 사건 경우 일어날 수 있는 모든 경우 백 앞면이 나온다. 첫째마당 확률 첫째마당에서는경우의수와확률을배울거야. 확률은일상생활에서자주사용돼. 대표적으로비올확률, 복권에당첨될확률, 시험문제를맞힐확률등은흔히접하는것들이지. 이렇게확률은우리생활속의문제를해결하는편리한수단이돼. 또한중학교 2학년의확률단원은고등학교에서배우는확률과통계와연결되는중요한단원이니, 잘익혀두자. 공부할내용! 14 일진도 20 일진도 스스로계획을세워봐! 01

More information

순열 명의학생이있다. (1) 이 10명을일렬로세우는모든방법의수를구하여라. (2) 이 10명중 3명을뽑아일렬로세우는방법의수는? (3) 이 10명중 n명을뽑아일렬로세울때, 그방법의수가 90가지라고한다. n의값을구하여라. 10) 15. 권의책이있다. (1) 이

순열 명의학생이있다. (1) 이 10명을일렬로세우는모든방법의수를구하여라. (2) 이 10명중 3명을뽑아일렬로세우는방법의수는? (3) 이 10명중 n명을뽑아일렬로세울때, 그방법의수가 90가지라고한다. n의값을구하여라. 10) 15. 권의책이있다. (1) 이 순열 1 인쇄는 공급용지에맞추어 로하세요.(B4) 기본정석에있는문제와유제 6. 다음등식을만족시키는 및 의값을구하여 라. 6) (1) 1. 다음각값을계산하여라. 1) (1) (2) (2) (3) (3) (4) 7. 다음등식을만족시키는 의값을구하여라. 7) (1) 2. 1, 2, 3, 4, 5 를써서만들수있는세자리정수 중에서각자리의숫자가다른것은모두몇개인 가? 2)

More information

두 두 두 두 두 lim 1. 수열의극한 수열의극한에대한기본성질 1. 수열의극한 Ⅰ 수열의극한 5. 수열, 에대하여 lim, lim 이성 립할때, lim 의값은? [3 점 ][2015(A) 7 월 / 교육청 5] 의값은? [2 점 ][200

두 두 두 두 두 lim 1. 수열의극한 수열의극한에대한기본성질 1. 수열의극한 Ⅰ 수열의극한 5. 수열, 에대하여 lim, lim 이성 립할때, lim 의값은? [3 점 ][2015(A) 7 월 / 교육청 5] 의값은? [2 점 ][200 두 두 두 두 두 1. 01 1. 수열의극한 수열의극한에대한기본성질 1. 수열의극한 Ⅰ 수열의극한 5. 수열, 에대하여, 이성 립할때, 의값은? 1 2 3 4 5 [3 점 ][2015(A) 7 월 / 교육청 5] 의값은? [2 점 ][2006( 나 ) 9 월 / 평가원 3] 1 2 3 4 5 6. 수열, 이, 를만족할 때, 의값을구하시오. [3 점 ][2005(

More information

3. 방정식 이나타내는도형은?3) 1 중심이 이고지름이 인원 3 중심이 이고지름이 인원 5 중심이 이고지름이 인원 2 중심이 이고지름이 인원 4 중심이 이고지름이 인원 4. 다음원의방정식의중심의좌표와반지름의길이를구하시오.4) 5. 원 에대한설명이다. < 보기 > 에서옳

3. 방정식 이나타내는도형은?3) 1 중심이 이고지름이 인원 3 중심이 이고지름이 인원 5 중심이 이고지름이 인원 2 중심이 이고지름이 인원 4 중심이 이고지름이 인원 4. 다음원의방정식의중심의좌표와반지름의길이를구하시오.4) 5. 원 에대한설명이다. < 보기 > 에서옳 원의정의 1. 원의정의 평면위의한정점에서거리가일정한점들의자취 평면위의한정점 로부터일정한거리 에있는점 의집합이라할때, 를점 를중심으로하고반지름의길이가 인원이라고한다. 2. 원의방정식 (1) 기본형 : 원점이중심이고반지름의길이가 인원의방정식 (2) 표준형 : 점 가중심이고반지름의길이가 인원의방정식 (3) 일반형 : ( 단, ) l 원의방정식 중심 :, 반지름 :

More information

NETUWQLVSOOQ.hwp

NETUWQLVSOOQ.hwp 확률의정의 1. 2. 용어의정의 (1) 시행 : 동일한조건에서반복이가능하고그결과가우연에의하여지배되는실험이나관찰을 시행이라고한다. (2) 사건 : 시행의결과를사건이라고한다. 1 근원사건 : 사건중에서더이상세분할수없는기본적인사건을근원사건이라고한다. 2 표본공간 : 근원사건전체의집합을표본공간또는전사건이라고한다. 전사건은반드시일어나는사건이다. 3 공사건 : 절대로일어나지않는사건을공사건이라고한다.

More information

Python과 함께 배우는 신호 해석 제 5 강. 복소수 연산 및 Python을 이용한 복소수 연산 (제 2 장. 복소수 기초)

Python과 함께 배우는 신호 해석 제 5 강. 복소수 연산 및 Python을 이용한 복소수 연산      (제 2 장. 복소수 기초) 제 5 강. 복소수연산및 을이용한복소수연산 ( 제 2 장. 복소수기초 ) 한림대학교전자공학과 한림대학교 제 5 강. 복소수연산및 을이용한복소수연산 1 배울내용 복소수의기본개념복소수의표현오일러 (Euler) 공식복소수의대수연산 1의 N 승근 한림대학교 제 5 강. 복소수연산및 을이용한복소수연산 2 복소수의 4 칙연산 복소수의덧셈과뺄셈에는직각좌표계표현을사용하고,

More information

<3235B0AD20BCF6BFADC0C720B1D8C7D120C2FC20B0C5C1FE20322E687770>

<3235B0AD20BCF6BFADC0C720B1D8C7D120C2FC20B0C5C1FE20322E687770> 25 강. 수열의극한참거짓 2 두수열 { }, {b n } 의극한에대한 < 보기 > 의설명중옳은것을모두고르면? Ⅰ. < b n 이고 lim = 이면 lim b n =이다. Ⅱ. 두수열 { }, {b n } 이수렴할때 < b n 이면 lim < lim b n 이다. Ⅲ. lim b n =0이면 lim =0또는 lim b n =0이다. Ⅰ 2Ⅱ 3Ⅲ 4Ⅰ,Ⅱ 5Ⅰ,Ⅲ

More information

5.5) 좌표평면 6.6) 그림과 그림과 수학영역경우의수 - 경로 위에서상하또는좌우방향으로한번에 만큼씩움 직이는점 P 가있다. 이때원점을출발한점 P 가 번움직여서최종위치가점 A 이되는경우의수를구하시오. [4 점 ][2004 년 3 월 ] 7.7 ) 같이바둑판모양의도로망

5.5) 좌표평면 6.6) 그림과 그림과 수학영역경우의수 - 경로 위에서상하또는좌우방향으로한번에 만큼씩움 직이는점 P 가있다. 이때원점을출발한점 P 가 번움직여서최종위치가점 A 이되는경우의수를구하시오. [4 점 ][2004 년 3 월 ] 7.7 ) 같이바둑판모양의도로망 수능 (94~17 학년도 ), 모의고사 (03~16 년 ) 단원 : 경우의수 ( 경로 ) 1. 아래그림과같은도로망이있다. 지점에서자동차가출발하 여 지점까지최단거리로갈때, 우회전하는회수를, 좌회전하는회수를 라하자. 도착 3. 어떤원자의전자들은에너지의증감에따라세가지상태 로바뀐다. 이때, 다음규칙이적용된다고하자. 규칙 1: 에너지가증가하면 상태의전자는 상태로올라가고,

More information

8. 수직선위에다음수들이대응할때, 원점에서가장멀리 위치한수는? 12. Å + 7 ã Å + 5 ã Å 16 ã + 3 을계산하여라 다음에서그결과가다른하나는? 1 3 보다 5 만큼큰수 9. 두정수 a, b

8. 수직선위에다음수들이대응할때, 원점에서가장멀리 위치한수는? 12. Å + 7 ã Å + 5 ã Å 16 ã + 3 을계산하여라 다음에서그결과가다른하나는? 1 3 보다 5 만큼큰수 9. 두정수 a, b 범위 : 소인수분해 정수와유리수 50 문항 / 중반 : 이름 : 중 1-1 수학중간고사대비 1. 다음중 81 의약수는? 1 2 2 4 3 5 4 6 5 9 6. 다음수들에대한설명으로옳은것은? 1 10, 1.2, 2, 2 5, 0, 4, 10 2 1 양수는 4 개이다. 2. 세수 2 7 2, 2 2 7 11, 5 11 2 의최소공배수는? 1 2 5 7 11 2

More information

Ⅰ 경우의수 개념 1 경우의수와순열 004 개념 2 여러가지순열 008 개념 3 조합과중복조합 012 개념 4 이항정리 020 Ⅱ 확률 개념 1 확률의정의및계산 024 개념 2 조건부확률 027 개념 3 독립과종속 028 Ⅲ 통계 개념 1 이산확률변수와확률분포 034

Ⅰ 경우의수 개념 1 경우의수와순열 004 개념 2 여러가지순열 008 개념 3 조합과중복조합 012 개념 4 이항정리 020 Ⅱ 확률 개념 1 확률의정의및계산 024 개념 2 조건부확률 027 개념 3 독립과종속 028 Ⅲ 통계 개념 1 이산확률변수와확률분포 034 Ⅰ 경우의수 개념 1 경우의수와순열 004 개념 2 여러가지순열 008 개념 3 조합과중복조합 012 개념 4 이항정리 020 Ⅱ 확률 개념 1 확률의정의및계산 024 개념 2 조건부확률 027 개념 3 독립과종속 028 Ⅲ 통계 개념 1 이산확률변수와확률분포 034 개념 2 이항분포 038 개념 3 연속확률변수와확률밀도함수 040 개념 4 정규분포 041

More information

FGB-P 학번수학과권혁준 2008 년 5 월 19 일 Lemma 1 p 를 C([0, 1]) 에속하는음수가되지않는함수라하자. 이때 y C 2 (0, 1) C([0, 1]) 가미분방정식 y (t) + p(t)y(t) = 0, t (0, 1), y(0)

FGB-P 학번수학과권혁준 2008 년 5 월 19 일 Lemma 1 p 를 C([0, 1]) 에속하는음수가되지않는함수라하자. 이때 y C 2 (0, 1) C([0, 1]) 가미분방정식 y (t) + p(t)y(t) = 0, t (0, 1), y(0) FGB-P8-3 8 학번수학과권혁준 8 년 5 월 9 일 Lemma p 를 C[, ] 에속하는음수가되지않는함수라하자. 이때 y C, C[, ] 가미분방정식 y t + ptyt, t,, y y 을만족하는해라고하면, y 는, 에서연속적인이계도함수를가지게확 장될수있다. Proof y 은 y 의도함수이므로미적분학의기본정리에의하여, y 은 y 의어떤원시 함수와적분상수의합으로표시될수있다.

More information

PARUEFQXXISK.hwp

PARUEFQXXISK.hwp 합의기호 1. 기호 의약속 끝항의번호 제 항 일반항 첫째항번호 2. 의성질 (1) (2) (는상수 ) (3) (5) ± ± ( 평행이동 ) ( 복호동순 ) (4) (는상수 ) 3. 4. 자연수의거듭제곱의합 (1) (2) (3) 분수수열의합 (1) (2) (3) (4) (5) (6) 계차수열 수열 에서 을계차라하고계차로이루어지는수열을계차수열이라한다. a n =

More information

03.순열과조합.hwp

03.순열과조합.hwp 순열 02 대칭성의 원리 01 합의 법칙과 곱의 법칙을 이용하여 경우의 수 구하기 1 합의 법칙 : 동시에 일어나지 않는 두 사건, 에 대 하여 두 사건, 가 일어나는 경우의 수가 각각, 일 때 또는 가 일어나는 경우의 수는 2 곱의 법칙 : 사건 가 일어나는 경우의 수가 이고, 그 각각에 대하여 사건 가 일어나는 경의 수가 일 때, 두 사건, 가 동시에 일어나는

More information

도형의닮음 1 강 - 닮은도형과닮음중심 사이버스쿨우프선생 닮음도형 : 일정한비율로확대또는축소하였을때닮음모양의도형 기호 : ABCD A'B'C'D' [ 예제 1 ] 그림에서와같이두닮은도형 ABCD 와 A'B'C'D' 에서대응점, 대

도형의닮음 1 강 - 닮은도형과닮음중심 사이버스쿨우프선생   닮음도형 : 일정한비율로확대또는축소하였을때닮음모양의도형 기호 : ABCD A'B'C'D' [ 예제 1 ] 그림에서와같이두닮은도형 ABCD 와 A'B'C'D' 에서대응점, 대 도형의닮음 1 강 - 닮은도형과닮음중심 사이버스쿨우프선생 www.cyberschool.co.kr 닮음도형 : 일정한비율로확대또는축소하였을때닮음모양의도형 기호 : '''' [ 예제 1 ] 그림에서와같이두닮은도형 와 '''' 에서대응점, 대응변을말하여라. ' ' ' ' [ 풀이] 대응점 : 와 ', 와 ', 와 ', 와 ' 대응변 : 와 '', 와 '', 와 '',

More information

Intensive Math Class I 공간기하벡터 강사최석호 1. 단면은수직으로 A, B 두평면사이각의코사인값을구하시오

Intensive Math Class I 공간기하벡터 강사최석호 1. 단면은수직으로 A, B 두평면사이각의코사인값을구하시오 Intensive Math Class I 공간기하벡터 강사최석호 1. 단면은수직으로 A, B 두평면사이각의코사인값을구하시오. 1. 2. 3. 4. 5. 6. 2. 꾹누르기 1. 그림과같은정육면체 ABCD EFGH에서모서리 BF를 로내분하는점을 I, 모서리 DH를 로내분하는점을 J라하자. 면 IGJ와 밑면 EFGH가이루는예각의크기를 라할때, cos 이다. 이때,

More information

1 경우의수 1 경우의수 일 ::1385 일 :: 경우의수 일 :: 집합,, 은다음과같다. [ 정답률 : 84%],, 집합 에서한개의원소를선택하여백의자리의수, 집합 에서한개의원소를선택하여십의자리의수, 집합 에서한개의원소를선택하여일의자리의수로

1 경우의수 1 경우의수 일 ::1385 일 :: 경우의수 일 :: 집합,, 은다음과같다. [ 정답률 : 84%],, 집합 에서한개의원소를선택하여백의자리의수, 집합 에서한개의원소를선택하여십의자리의수, 집합 에서한개의원소를선택하여일의자리의수로 1 경우의수 장미 일 ::8879 1 경우의수 1 경우의수 일 ::4716 일 ::2079 유형 01 [1] 합의법칙 경우의수 두사건, 가일어나는경우의수각각각, 이고두사건, 가동시에일어나지않을때, 사건 또는사건 가일어 나는경우의수는 이고이를합의법칙이라고한다. [2] 곱의법칙 사건 가일어나는경우의수가 이고사건 가일어나는경우 의수가 일때, 사건 와사건 가동시에일어나는경우의수

More information

제 5 일 년 7월교육청 년 10월교육청 년수능 년 6월평가원 년 9월평가원 년 9월평가원 년수능 년경찰대 년수능 년 10월교육청

제 5 일 년 7월교육청 년 10월교육청 년수능 년 6월평가원 년 9월평가원 년 9월평가원 년수능 년경찰대 년수능 년 10월교육청 제 5 일 1. 2016년 7월교육청 2. 2011년 10월교육청 3. 2016년수능 4. 2012년 6월평가원 5. 2010년 9월평가원 6. 2012년 9월평가원 7. 2006년수능 8. 2011년경찰대 9. 2006년수능 10. 2006년 10월교육청 1. 그림과같이중심이 O 이고반지름의길이가 인원의둘 레를 등분한점을,,, 이라하자. 호 ( ) 을이등분한점을

More information

< D312D3420BBEFB0A2C7FCC0C720BFDCBDC9B0FA20B3BBBDC E485750>

< D312D3420BBEFB0A2C7FCC0C720BFDCBDC9B0FA20B3BBBDC E485750> 1)1) 2)2) 3) 3) 4) 4) 5) 5) 1. zb 그림에서점 O는중옳은것은? ABC 의외심이다. 3. zb 그림에서점 I 는직각삼각형 ABC 의내심이다. 삼각형의세변의길이가각각 10 cm, 8cm, 6cm 일때, 색칠한부분의넓이는? 1 OD = OE = OF 2 OA = OB = OC 3 AD = AF 4 OCE = OCF 5 OBD OBE 1 (

More information

10-2 삼각형의닮음조건 p270 AD BE C ABC DE ABC 중 2 비상 10, 11 단원도형의닮음 (& 활용 ) - 2 -

10-2 삼각형의닮음조건 p270 AD BE C ABC DE ABC 중 2 비상 10, 11 단원도형의닮음 (& 활용 ) - 2 - 10 단원 : 도형의닮음 10-1 닮음도형 p265 ABC DEF ABC DEF EF B ABCD EFGH ABCD EFGH EF A AB GH ADFC CF KL 중 2 비상 10, 11 단원도형의닮음 (& 활용 ) - 1 - 10-2 삼각형의닮음조건 p270 AD BE C ABC DE ABC 중 2 비상 10, 11 단원도형의닮음 (& 활용 ) - 2 -

More information

두산동아-확통 완성본.hwp

두산동아-확통 완성본.hwp 4단원확률의뜻과활용 1. 확률의뜻 문제 1 원짜리동전 개를두번던지는시행에 대하여다음을구하여라. 143) ( 단, 앞면은 H, 뒷면은 T로나타낸다.) ⑴ 표본공간 문제 3 원짜리동전 개와 원짜리동전 개를동시에던질때, 모두앞면또는모두뒷면이나올확률을구하여라. 145) ⑵ 두번모두앞면이나오는사건 예제 1 어느가수선불프로그램에서는예선을통과한최종 명만생방송본선무대에진출할수있다고한다.

More information

집합 서로 확률과통계 1. 순열 순열의수, 에 대하여함수 가일대일대응이다. 이때, 을만족하는함 수 의개수는? [3점][2008( 가 ) 11월 / 교육청 ( 고2) 6] 다른네종류의모자 A, B, C, D 가각각 개씩모두 개 있다. 개

집합 서로 확률과통계 1. 순열 순열의수, 에 대하여함수 가일대일대응이다. 이때, 을만족하는함 수 의개수는? [3점][2008( 가 ) 11월 / 교육청 ( 고2) 6] 다른네종류의모자 A, B, C, D 가각각 개씩모두 개 있다. 개 서로 그림과 세 그림과 1. 순열 Ⅰ 순열과조합 1. 01 합의법칙과곱의법칙 다른네가지의색이있다. 이중네가지이하의색을이용하 여인접한행정구역을구별할수있도록모두칠하고자한다. 다섯개의구역을서로다른색으로칠할수있는모든경우의수는? ( 단, 행정구역에는한가지색만을칠한다.) [3점][2009( 가 ) 4월 / 교육청 13] 3.[ 그림 ] 과같이네개의방이통로로연결되어있을때,

More information

- A 2 -

- A 2 - - A 1 - - A 2 - - A 3 - - A 4 - - A 5 - - A 6 - 번호 정답 번호 정답 1 4 16 1 2 1 17 1 3 1 18 3 4 4 19 4 5 2 20 4 6 2 21 4 7 3 22 2 8 4 23 4 9 2 24 4 10 1 25 2 11 2 26 1 12 1 27 4 13 2 28 3 14 3 29 3 15 2 30 3

More information

2008 년도 3 월고 1 전국연합학력평가정답및해설 수리영역 정답

2008 년도 3 월고 1 전국연합학력평가정답및해설 수리영역 정답 2008 년도 3 월고 1 전국연합학력평가정답및해설 수리영역 정답 1 2 2 5 3 3 4 4 5 4 6 1 7 4 8 5 9 1 10 1 11 3 12 5 13 2 14 4 15 2 16 3 17 2 18 1 19 5 20 3 21 4 22 23 24 25 26 27 28 29 30 주어진연립부등식이해를가지려면ᄃ과ᄅ의공통범위가존재하여야한다. 따라서그림으로부터

More information

7.7) 오른쪽 8.8) A 10.10) 컴퓨터에 11.11) A 두 수학영역경우의수, 확률 그림과같이한변의길이가 인정삼각형 개로이루어진정육각형모양의도형이있다. 동전을던져앞면이나오면정삼각형의변을따라 만큼씩움직이고, 뒷면이나오면움직이지않는다. 갑과을이각각동전을한번씩던지고

7.7) 오른쪽 8.8) A 10.10) 컴퓨터에 11.11) A 두 수학영역경우의수, 확률 그림과같이한변의길이가 인정삼각형 개로이루어진정육각형모양의도형이있다. 동전을던져앞면이나오면정삼각형의변을따라 만큼씩움직이고, 뒷면이나오면움직이지않는다. 갑과을이각각동전을한번씩던지고 1.1) 2.2) 3.3) 어느 각 두 경찰대 (99~17 학년도 ), 사관학교 (02~17 학년도 ) 단원 : 경우의수, 확률 일때, 순서쌍 의개수는? [2000 년경찰대 ] 4.4 ) 자리숫자가 부터 까지인 자리수로된여행용가방의 비밀번호를잊어버렸다. 그런데, 비밀번호의일의자리숫자는, 백의자리숫자는 이고, 비밀번호가 로나누어떨어진다는것을알고있다. 이때, 비밀번호로가능한것은몇가지인가?

More information

Ⅰ 순열과조합. 순열 0 경우의수 p.7 ~ 9 7 ⑴ ⑵ 8 8 0! 꺼낸공에적힌수의차가 인경우 {, }, {, }, {, }, {, } SG 꺼낸공에적힌수의차가 인경우 {, }, {, } SG 는동시에일어날수없으므로구하는경우의수는 += 부터 0

Ⅰ 순열과조합. 순열 0 경우의수 p.7 ~ 9 7 ⑴ ⑵ 8 8 0! 꺼낸공에적힌수의차가 인경우 {, }, {, }, {, }, {, } SG 꺼낸공에적힌수의차가 인경우 {, }, {, } SG 는동시에일어날수없으므로구하는경우의수는 += 부터 0 수학영역 확률과 통계 정답과해설 Ⅰ 순열과조합. 순열 0 경우의수 p.7 ~ 9 7 ⑴ ⑵ 8 8 0! 꺼낸공에적힌수의차가 인경우 {, }, {, }, {, }, {, } SG 가지 @ 꺼낸공에적힌수의차가 인경우 {, }, {, } SG 가지!, @ 는동시에일어날수없으므로구하는경우의수는 += 부터 00까지의자연수중에서 으로나누어떨어지는수, 즉 의배수는 개 로나누어떨어지는수,

More information

2 5. 어느나라의올해물가지수는전년도에비해 % 상승하였다. 7. 서로다른세종류의과일이각각 개씩모두 개가들어있 이나라의물가지수가매년이러한비율로상승한다고할때, 물 가지수가처음으로올해의 배이상이되는해는앞으로몇년 후인가? ( 단, log, log 로계산한다.) [3 점] 는바

2 5. 어느나라의올해물가지수는전년도에비해 % 상승하였다. 7. 서로다른세종류의과일이각각 개씩모두 개가들어있 이나라의물가지수가매년이러한비율로상승한다고할때, 물 가지수가처음으로올해의 배이상이되는해는앞으로몇년 후인가? ( 단, log, log 로계산한다.) [3 점] 는바 2009학년도 3월고3 전국연합학력평가문제지 제 2 교시 가 형 성명수험번호 3 1 자신이선택한유형( 가 형/ 나 형) 의문제지인지확인하시오. 문제지의해당란에성명과수험번호를정확히쓰시오. 답안지의해당란에성명과수험번호를쓰고, 또수험번호와 답을정확히표시하시오. 단답형답의숫자에 0 이포함되면, 그 0 도답란에반드시 표시하시오. 문항에따라배점이다르니, 각물음의끝에표시된배점을

More information

그림은 집합 한 확률과통계 1. 순열 7. 어떤학생이작성한수행평가보고서의표지이다. 9.[ 그림 ] 과같이네개의방이통로로연결되어있을때, 어느한방 에서출발하여모든방을한번만방문하는방법의수는출발하는방의 경우의수가 ( 가지 ) 이고각경우에모든방을방문하는방법의수는 ( 가지 ) 이

그림은 집합 한 확률과통계 1. 순열 7. 어떤학생이작성한수행평가보고서의표지이다. 9.[ 그림 ] 과같이네개의방이통로로연결되어있을때, 어느한방 에서출발하여모든방을한번만방문하는방법의수는출발하는방의 경우의수가 ( 가지 ) 이고각경우에모든방을방문하는방법의수는 ( 가지 ) 이 다항식 세 그림과 그림과 집합 장미 1. 순열 Ⅰ 순열과조합 1. 01 합의법칙과곱의법칙 1. 순열 를전개하였을때 항의개수는? [3점][2005( 나 ) 7월 / 교육청 5] 1 2 3 4 5 4. 같이 개의섬이다리로연결되어있다. 흰색, 노란색, 파란 색깃발이각각 개씩총 개있을때, 이 개의깃발을섬에한개씩세우고자한다. 다리로연결된이웃한두섬에는같은색의깃발을세우지않는다고할때,

More information

등차수열 등차수열 등차수열 등차수열 등차수열 등차수열 등차수열 등차수열 첫째항이 수열 등차수열 등차수열 등차수열 수학 Ⅱ 1. 등차수열과등비수열 14. 이 이고, 일때, 의값을구 하시오. [3점][2011( 나 ) 9월 / 평가원 23] 21.개의실수,,,, 가이순서대

등차수열 등차수열 등차수열 등차수열 등차수열 등차수열 등차수열 등차수열 첫째항이 수열 등차수열 등차수열 등차수열 수학 Ⅱ 1. 등차수열과등비수열 14. 이 이고, 일때, 의값을구 하시오. [3점][2011( 나 ) 9월 / 평가원 23] 21.개의실수,,,, 가이순서대 등차수열 등차수열 등차수열 등차수열 등차수열 등차수열 공차가 등차수열 등차수열 첫째항과 등차수열 등차수열 등차수열 1. 등차수열과등비수열 Ⅲ 수열 01 등차수열의일반항 1. 등차수열과등비수열 7. 인등차수열 에대하여 의값은? [3점][2015(A) 6월 / 평가원 4] 1 2 3 4 5 1. 에대하여, 일때, 의값은? [3점][2014(A) 6월 / 평가원

More information

인천대건고등학교 3 학년반번호이름검사 (A,B,C- 채점및풀이과정여부 ): 순열 1. 이하의자연수 에대하여 개의수 을일렬로나열하여만든 자리의자연수를 이라할때, 다음조건을만족시키는자연수 의개수를 이라하자. ( 가 ) 일때, 1. 탁자 A 에서 명, 탁자 B

인천대건고등학교 3 학년반번호이름검사 (A,B,C- 채점및풀이과정여부 ): 순열 1. 이하의자연수 에대하여 개의수 을일렬로나열하여만든 자리의자연수를 이라할때, 다음조건을만족시키는자연수 의개수를 이라하자. ( 가 ) 일때, 1. 탁자 A 에서 명, 탁자 B 인천대건고등학교 3 학년반번호이름검사 (A,B,C- 채점및풀이과정여부 ): - 1-1. 순열 예제 2. 개의숫자 을일렬로나열할때, 다음조건을만족시키는경우의수는? ( 가 ) 홀수끼리는이웃하지않는다. ( 나 ) 짝수끼리는이웃하지않는다. ( 다 ) 은이웃하여나열한다. 1. 어른 명과어린이 명이함께놀이공원에가서어느놀이기구를타려고한다. 이놀이기구는그림과같이앞줄에 개,

More information

고난한입시의길, 당신이힘들고지쳐주저앉지않도록 때론앞에서끌고뒤에서밀며 그길같이걷겠습니다. 고지우배상 1 Round 1. 경우의수 Round 2. 확률 Round 3. 통계정답 3p 36p 59p 77p 2 UPSET 확률과통계 3 Round 1. 경우의수 Theme 1. 순열 4 UPSET 확률과통계 001 등식 를만족시키는자연수 의값을구하시오. [ 점 ] [2011

More information

2 제 7 장순열과조합 [ 문제 3] A, B,, H 지점이다음그림과같이도로로연결되어있을때, A 에서 B 에이르는방법의수를구하여라. ( 단, 한번지난지점은다시지날수없다.) 네개의도시 A, B, C,D 사이에아래그림과같은도 로가있다. 도시 A 에서도시 C 로가는방법의수는

2 제 7 장순열과조합 [ 문제 3] A, B,, H 지점이다음그림과같이도로로연결되어있을때, A 에서 B 에이르는방법의수를구하여라. ( 단, 한번지난지점은다시지날수없다.) 네개의도시 A, B, C,D 사이에아래그림과같은도 로가있다. 도시 A 에서도시 C 로가는방법의수는 1 경우의수 1 2 사건 A, B 가동시에일어나는경우 A B 3 사건 A 가일어나지않는경우 A c 제 7 장 순열과조합 학습목표 사건이일어나는모든경우의수를헤아리는것은 확률의가장기초적인단계입니다. 따라서경우의 수를효과적으로구할수있는방법가운데하나인 순열과조합을배우게됩니다. 1. 합의법칙과곱의법칙을이해하고, 이를이용 하여경우의수를구할수있다. 2. 순열의뜻을알고,

More information

4. [3 등급 60 초 ] 5. [3 등급 60 초 ] 6. [3 등급 60 초 ] 2

4. [3 등급 60 초 ] 5. [3 등급 60 초 ] 6. [3 등급 60 초 ] 2 Lui Intensive 천재의발상 공간벡터좌표 강사최석호 1. 단면은수직으로 A, B 두평면사이각의코사인값을구하시오. 1. [3 등급 45 초 ] 2. [3 등급 45 초 ] 3. [3 등급 45 초 ] * 등급 - 제한시간표시 [3 등급 90s] 3 등급에가장효과적인문항입니다. 90 초간생각후끝까지풀지말고강의를들어주세요. 등급및 제한시간표시는강의영상과차이가있을수있으며영상보다교재의등급시간을우선합니다.

More information

2020 학년도랑데뷰실전모의고사문제지 - 시즌 6 제 1 회 제 2 교시 수학영역 ( 가형 ) 1 5 지선다형 학년도 9월모의평가가형과싱크로율 99% 학년도수학영역대비랑데뷰실전모의고사가형-시즌1~ 시즌6, 나형-시즌1~ 시즌2 ( 각시즌 4회분 )

2020 학년도랑데뷰실전모의고사문제지 - 시즌 6 제 1 회 제 2 교시 수학영역 ( 가형 ) 1 5 지선다형 학년도 9월모의평가가형과싱크로율 99% 학년도수학영역대비랑데뷰실전모의고사가형-시즌1~ 시즌6, 나형-시즌1~ 시즌2 ( 각시즌 4회분 ) 2020 학년도랑데뷰실전모의고사문제지 - 시즌 6 제 1 회 제 2 교시 1 5 지선다형 - 2020학년도 9월모의평가가형과싱크로율 99% - 2020학년도수학영역대비랑데뷰실전모의고사가형-시즌1~ 시즌6, 나형-시즌1~ 시즌2 ( 각시즌 4회분 ) 오르비전자책에서구매가능 - 오타, 오류수정파일은랑데뷰수학카페자료실에서무료다운로드가능 (cafe.daum.net/baekipsi)

More information

1 1 장. 함수와극한 1.1 함수를표현하는네가지방법 1.2 수학적모형 : 필수함수의목록 1.3 기존함수로부터새로운함수구하기 1.4 접선문제와속도문제 1.5 함수의극한 1.6 극한법칙을이용한극한계산 1.7 극한의엄밀한정의 1.8 연속

1 1 장. 함수와극한 1.1 함수를표현하는네가지방법 1.2 수학적모형 : 필수함수의목록 1.3 기존함수로부터새로운함수구하기 1.4 접선문제와속도문제 1.5 함수의극한 1.6 극한법칙을이용한극한계산 1.7 극한의엄밀한정의 1.8 연속 1 1 장. 함수와극한 1.1 함수를표현하는네가지방법 1.2 수학적모형 : 필수함수의목록 1.3 기존함수로부터새로운함수구하기 1.4 접선문제와속도문제 1.5 함수의극한 1.6 극한법칙을이용한극한계산 1.7 극한의엄밀한정의 1.8 연속 2 1.1 함수를표현하는네가지방법 함수 f : D E 는집합 D 의각원소 x 에집합 E 에속하는단하나의원소 f(x) 를 대응시키는규칙이다.

More information

157. A B C 세나라에서각각대표 명씩을뽑 아총 명이원탁에앉아회의를하려고한다. A B 두나라의대표들만자국의대표끼리이웃하게앉는방법의수를구하여라. 157) 159. 두집합 에대하여 를만족시키는함수 의개수를구하여라. 159) 158. 좌표평면위의점들의집합 는정수 가있다.

157. A B C 세나라에서각각대표 명씩을뽑 아총 명이원탁에앉아회의를하려고한다. A B 두나라의대표들만자국의대표끼리이웃하게앉는방법의수를구하여라. 157) 159. 두집합 에대하여 를만족시키는함수 의개수를구하여라. 159) 158. 좌표평면위의점들의집합 는정수 가있다. 153. 아래그림과같이거리가 인두평행선위 에 의간격으로점이각각 개씩있을때, 네점을꼭짓점으로하여만들수있는사각형중에서넓이가 인것의개수를구하여라. 153) 155. 어느동물원에서오른쪽그림과같이번호가적혀있는 칸의동물우리에호랑이, 사자, 늑대, 여우, 원숭이, 곰을각각한마리씩넣으려고하는데호랑이와사자는이웃하지않게넣어야한다. 예를들어 의경우에는 와 가이웃하는우리이고, 은이웃하지않는우리이다.

More information

제 12강 함수수열의 평등수렴

제 12강 함수수열의 평등수렴 제 강함수수열의평등수렴 함수의수열과극한 정의 ( 점별수렴 ): 주어진집합 과각각의자연수 에대하여함수 f : 이있다고가정하자. 이때 을집합 에서로가는함수의수열이라고한다. 모든 x 에대하여 f 수열 f ( x) lim f ( x) 가성립할때함수수열 { f } 이집합 에서함수 f 로수렴한다고한다. 또 함수 f 을집합 에서의함수수열 { f } 의극한 ( 함수 ) 이라고한다.

More information

8. 8) 오른쪽그림과같은직사 )5명의학생 A, B, C, D, E를한줄로세울때, A 각형모양의길이있다. 최 와 E 가양끝에서는확률을구하여라. 단거리로점 A 에서 P 를 거쳐점 B 까지가는방법 은모두몇가지인지구하 여라 ) A, B, C, D, E

8. 8) 오른쪽그림과같은직사 )5명의학생 A, B, C, D, E를한줄로세울때, A 각형모양의길이있다. 최 와 E 가양끝에서는확률을구하여라. 단거리로점 A 에서 P 를 거쳐점 B 까지가는방법 은모두몇가지인지구하 여라 ) A, B, C, D, E 수학익힘책문제풀기 중 2-2: 01. 경우의수와확률 ( 기본부터심화까지 ) 4. 4)50 원짜리, 100 원짜리동전을사용하여 500 원을 지불하는방법은모두몇가지인가? ( 단, 두개의 동전은각각 1 개이상반드시사용한다.) 1. 1) 오른쪽그림과같이자음과모음이적힌카드에서각각한장씩을골라글자를만드는경우의수를구하여라. 두산동아 5. 5) 디딤돌진영이는 원, 원, 원짜리동전을각각

More information

(001~006)개념RPM3-2(부속)

(001~006)개념RPM3-2(부속) www.imth.tv - (~9)개념RPM-(본문).. : PM RPM - 대푯값 페이지 다민 PI LPI 알피엠 대푯값과산포도 유형 ⑴ 대푯값 자료 전체의 중심적인 경향이나 특징을 하나의 수로 나타낸 값 ⑵ 평균 (평균)= Ⅰ 통계 (변량)의 총합 (변량의 개수) 개념플러스 대푯값에는 평균, 중앙값, 최 빈값 등이 있다. ⑶ 중앙값 자료를 작은 값부터 크기순으로

More information

< D312D3220C0CCB5EEBAAFBBEFB0A2C7FC E485750>

< D312D3220C0CCB5EEBAAFBBEFB0A2C7FC E485750> 다음 1)1) 2)2) 다음 가 3) 3) 4) 4) 나 다 5) 5) 라 6) 6) 다음 7) 7) 8) 8) 다음 1. zb 다음그림과같이 AB = AC인 ABC 에서 BC = BD 이고, BDC = 65 일때, DAB - ABD 의크기는? AB = AD 1 BC = DC 2 ( 다 ) 3 1, 2, 3으로부터대응변의길이가같으므로 ABC ( 라 ) BAC

More information

Microsoft PowerPoint - 26.pptx

Microsoft PowerPoint - 26.pptx 이산수학 () 관계와그특성 (Relations and Its Properties) 2011년봄학기 강원대학교컴퓨터과학전공문양세 Binary Relations ( 이진관계 ) Let A, B be any two sets. A binary relation R from A to B, written R:A B, is a subset of A B. (A 에서 B 로의이진관계

More information

PowerPoint Presentation

PowerPoint Presentation 5 불대수 IT CookBook, 디지털논리회로 - 2 - 학습목표 기본논리식의표현방법을알아본다. 불대수의법칙을알아본다. 논리회로를논리식으로논리식을논리회로로표현하는방법을알아본다. 곱의합 (SOP) 과합의곱 (POS), 최소항 (minterm) 과최대항 (mxterm) 에대해알아본다. 01. 기본논리식의표현 02. 불대수법칙 03. 논리회로의논리식변환 04.

More information

집합 집합 확률과통계 1. 순열 6.,, 은다음과같다.,, 집합 에서한개의원소를선택하여백의자리의수, 집합 에서한개의원소를선택하여십의자리의수, 집합 에서한개의원소를선택하여일의자리의수로하는세자리의수를만들때, 각자리의수가모두다른세자리의수의개수는? [3점][2005( 나 )

집합 집합 확률과통계 1. 순열 6.,, 은다음과같다.,, 집합 에서한개의원소를선택하여백의자리의수, 집합 에서한개의원소를선택하여십의자리의수, 집합 에서한개의원소를선택하여일의자리의수로하는세자리의수를만들때, 각자리의수가모두다른세자리의수의개수는? [3점][2005( 나 ) 다항식 그림과 집합 장미 그림은 1. 순열 Ⅰ 순열과조합 1. 1. 순열 유형 1 합의법칙과곱의법칙 를전개하였을때 항의개수는? [3점][2005( 나 ) 7월 / 교육청 5] 1 2 3 4 5 4. 송이, 카네이션 송이, 백합 송이가있다. 이중 송이를 골라꽃병 A 에꽂고, 이꽃과는다른종류의꽃들중꽃병 B 에꽂을꽃 송이를고르는경우의수를구하시오. ( 단, 같은종류의꽃은서로구분하지않는다.)

More information

8. 8) 다음중용어의정의로옳은것은? 1 정사각형 : 네변의길이가같은사각형 2 정삼각형 : 세내각의크기가같은삼각형 3 이등변삼각형 : 두변의길이가같은삼각형 4 평행사변형 : 두쌍의대변의길이가각각같은사각형 5 예각삼각형 : 한내각의크기가 90 보다크고 180 보다작은삼각

8. 8) 다음중용어의정의로옳은것은? 1 정사각형 : 네변의길이가같은사각형 2 정삼각형 : 세내각의크기가같은삼각형 3 이등변삼각형 : 두변의길이가같은삼각형 4 평행사변형 : 두쌍의대변의길이가각각같은사각형 5 예각삼각형 : 한내각의크기가 90 보다크고 180 보다작은삼각 1. 1) 수학익힘책문제풀기 중 2-2: 02. 삼각형의성질 ( 기본부터심화까지 ) 다음명제의역이참인지거짓인지를말하여라. 5. 5), 는자연수이고, 문장,, 가각각다음과같을때, 다음기호를명제로나타낼때, 참인지거짓인지를말하여라. : 는짝수이고 는홀수이다. : 는홀수이다. : 는홀수이다. ⑴ ⑵ ⑶ ⑴ 이면 이다. ⑵ 이면 이다. ⑶ 12의배수는 6의배수이다.

More information

개념완성 확률과통계 VITAEDU-ACADEMY 노박사수학교실 제 1 장 경우의수 01 경우의수 개념완성 1. 경우의수 01 경우의수 빠짐없이, 중복되지않게 사전식배열, 수형도 복잡한경우의수를셀때는점화식을이용하는경우도있다. (1) 합의법칙 한사건 가 가지의방법으로일어나고, 다른사건 가 가지의방법으로일어난다고할때 또는 가일어나는경우의수는, 가동시에일어나지않을때,

More information

0 cm (++x)=0 x= R QR Q =R =Q = cm =Q =-=(cm) =R =x cm (x+) = +(x+) x= x= (cm) =+=0 (cm) =+=8 (cm) + =0+_8= (cm) cm + = + = _= (cm) 7+x= x= +y= y=8,, Q

0 cm (++x)=0 x= R QR Q =R =Q = cm =Q =-=(cm) =R =x cm (x+) = +(x+) x= x= (cm) =+=0 (cm) =+=8 (cm) + =0+_8= (cm) cm + = + = _= (cm) 7+x= x= +y= y=8,, Q . 09~ cm 7 0 8 9 8'-p 0 cm x=, y=8 cm 0' 7 cm 8 cm 9 'åcm 90 'åcm T T=90 T T =" 8 - =' (cm) T= T= _T _T _'_ T=8' (cm ) 7 = == =80 -_ =0 = = _=(cm) M = = _0= (cm) M M =" - = (cm) r cm rcm (r-)cm H 8cm cm

More information

MGFRSQQFNTOD.hwp

MGFRSQQFNTOD.hwp 접선의방정식과평균값의정리 1. 접선의기울기와미분계수 곡선 위의점 에서의접선의기울기는 2. 접선의방정식 (1) 접선의방정식 곡선 위의점 에서의접선의방정식은 ( 단, y 1 = f (x 1 ) ) (2) 법선의방정식 곡선 위의점 에서의법선의방정식은 3. 두곡선의공통접선 두곡선 가 (1) 점 에서접할조건 1 (2) 점 에서직교할조건 1 2 2 4. 롤(Rolle)

More information

남자 두 서로 다음 그림과 확률과통계 1. 순열 01 순열의수 01 중복순열 6. 을한번씩만사용하여만들수있는여섯자리자연수중에서일의자리의수와백의자리의수가모두 의배수인자연수의개수를구하시오. [3점][2005( 나 ) 6월 / 평가원 21] 9. 다른과일 개를 개의그릇 A,

남자 두 서로 다음 그림과 확률과통계 1. 순열 01 순열의수 01 중복순열 6. 을한번씩만사용하여만들수있는여섯자리자연수중에서일의자리의수와백의자리의수가모두 의배수인자연수의개수를구하시오. [3점][2005( 나 ) 6월 / 평가원 21] 9. 다른과일 개를 개의그릇 A, 그림은 장미 다항식 집합 1. 순열 Ⅰ 순열과조합 1. 01 합의법칙과곱의법칙 어떤학생이작성한수행평가보고서의표지이다. 3. 를전개하였을때 항의개수는? [3점][2005( 나 ) 7월 / 교육청 5] 1 2 3 4 5 머리말, 제목, 인적사항의글꼴을표에서각각한개씩선택하여바꾸려고할때, 글꼴이모두다른경우의수를구하시오. [3점][2006( 가 ) 4월 / 교육청 23]

More information

3. 원 위의점 P 가있다. 점 P 에서의접선이 축과만나는 점을 Q 라하고 인점을 R 라하자. 점 P 의 좌표를 라하고, OPQ 의넓이를, PRQ 의넓이를 라할때, 값은? ( 단, 점 P 는제 사분면위의점이다.) [ Level C+ ] lim 의 답

3. 원 위의점 P 가있다. 점 P 에서의접선이 축과만나는 점을 Q 라하고 인점을 R 라하자. 점 P 의 좌표를 라하고, OPQ 의넓이를, PRQ 의넓이를 라할때, 값은? ( 단, 점 P 는제 사분면위의점이다.) [ Level C+ ] lim 의 답 Intensive Math Daily Training - Day 1 강사최석호 1. 그림과같이 A B C D E의다섯개의영역에빨강, 노랑, 파랑, 초록의네가지색으로색칠을하려고한다. 네가지색중한색은두번사용하고나머지세가지색은한번씩만사용하여칠하는데, 인접한영역에는서로다른색을칠하기로할때, 색칠하는방법의수를구하시오. [ Level B+ ] 답 : 2. 그림과같이한변의길이가

More information

숫자 서로 서로 할머니 확률과통계 1. 순열 중복순열 중에서중복을허락하여네개를택해일렬로나 열하여만든네자리의자연수가 의배수인경우의수는? [3점][2017( 가 ) 수능 5] 사전식배열 03 두집단을배열하는순열, 아버지, 어머니, 아

숫자 서로 서로 할머니 확률과통계 1. 순열 중복순열 중에서중복을허락하여네개를택해일렬로나 열하여만든네자리의자연수가 의배수인경우의수는? [3점][2017( 가 ) 수능 5] 사전식배열 03 두집단을배열하는순열, 아버지, 어머니, 아 장미 세 1. 순열 Ⅰ 순열과조합 01 합의법칙과곱의법칙 01 순열의수 1. 송이, 카네이션 송이, 백합 송이가있다. 이중 송이를 골라꽃병 A 에꽂고, 이꽃과는다른종류의꽃들중꽃병 B 에꽂을꽃 송이를고르는경우의수를구하시오. ( 단, 같은종류의꽃은서로구분하지않는다.) [4점][2016( 가 ) 10월 / 교육청 26] 3.P 일때, 자연수 의값을구하시오. [3

More information

개념발상법 4 시그마의응용 1. 합의기호 1 의약속 제 항 일반항 2 의성질 ᄀ ᄂ ᄃ 는상수 ± ± ( 복호동순 ) ᄅ 는상수 ᄆ ( 평행이동 ) 3 자연수의거듭제곱 ᄀ ᄂ ᄃ 4 분수의합 ᄀ ᄂ ᄃ ᄅ

개념발상법 4 시그마의응용 1. 합의기호 1 의약속 제 항 일반항 2 의성질 ᄀ ᄂ ᄃ 는상수 ± ± ( 복호동순 ) ᄅ 는상수 ᄆ ( 평행이동 ) 3 자연수의거듭제곱 ᄀ ᄂ ᄃ 4 분수의합 ᄀ ᄂ ᄃ ᄅ 강좌명 EBS 수능특강수 1 서정원선생님의강의자료 2011. 4. 강의명 제 34 강 ~42 강 부제명여러가지 (1)~(9) 작성자성명 : 김마음 e-mail : maeum27@naver.com 개념발상법 4 시그마의응용 1. 합의기호 1 의약속 제 항 일반항 2 의성질 ᄀ ᄂ ᄃ 는상수 ± ± ( 복호동순 ) ᄅ 는상수 ᄆ ( 평행이동 ) 3 자연수의거듭제곱

More information

2. 곱의법칙 사건 가일어나는경우의수가, 그각각에대하여사건 가일어나는경우의수가 일때, 두사건, 가잇달아일어나는경우의수는 이다. 곱의법칙은셋이상의사건에대해서도성립한다. 생각열기 민서는영화예매사이트 A 와 B 중어느한곳에서영화가, 나, 다중하나를예매하려고한다. 예매하

2. 곱의법칙 사건 가일어나는경우의수가, 그각각에대하여사건 가일어나는경우의수가 일때, 두사건, 가잇달아일어나는경우의수는 이다. 곱의법칙은셋이상의사건에대해서도성립한다. 생각열기 민서는영화예매사이트 A 와 B 중어느한곳에서영화가, 나, 다중하나를예매하려고한다. 예매하 중단원 Ⅰ- 1. 순열소단원 1. 경우의수 (10~13 쪽 ) 자료번호 1 학습내용합의법칙, 곱의법칙, 경우의수학습자 2 학년반번이름 : 1. 합의법칙 합의법칙과곱의법칙을유도한다. 두사건, 가동시에일어나지않을때, 사건, 가일어나는경우의수가각각, 이면사건 또는사건 가일어나는경우의수는 이다. 합의법칙은셋이상의사건에대해서도성립한다. 생각열기 서로다른두개의주사위를동시에던질때,

More information